Questionnaires 1

Download as pdf or txt
Download as pdf or txt
You are on page 1of 143

MIDTERM

EXAM
V1
Question 1 1 / 1 pts

The following income are su bject to final tax, except:

Cash dividend received by a domestic corporation from a resident foreign corporation.

Interest income received by a resident foreign corporation from a Philippine bank.

Stock d ividends received by a domestic corporation from another do mestic corporation.

Royalty income received by a domestic corporation from another domestic corporation.

Question 2 1 / 1 pts

The M CIT shall not apply to the following Foreign Corporations, except:

RFC engaged in business as a Regional Operating Headquarters (ROHQ) for the year ending December 31. 2021.

Non-resident Foreign Corporation (NRFC) on gross income derived from lease of vessel

RFC engaged in business as international air carrier subject to 21/2% of its gross Philippine billings.

RFC engaged in importing Cranes and H eavy equipment into t he Philippines

MCIT is applicable to every corporation taxable to regular corporate income tax including non-profit, exempt, and special
corporations with respect to their taxable income subject to regular corporate income tax, but not to their income subject to special tax
rates.

Thus, international air carries, ROHQ, and NRFC are MCIT exempt entities.

Question 3 1 / 1 pts

W hich of t he following statements is not correct?

When the co-owners invest the income of the property co-owned in a business or in any income producing properties or activities
constituting themselves into a business partnership. su ch partnership is consequently subject to income tax as a corporation.

Pa rtnerships, no matter how created or o rganized arc considered corporation subje ct to corporate income tax.

A co-owner is subject to income tax on his share in the net income of the co- ownership actually or constructively received.

As a rule. co-ownership is not subject to income tax because the co-owners arc limited to the preservation and enjoyment of the property
and the collection of the income there from.

GPP is not subject to corporate income tax.


Que stion 4 1 / 1 pts

The payor of passive income subject to fi nal t ax is req uire to w ithhold t he fi nal t ax from the paymen: due to t he recipient. The
w ithholding of t he tax, has t he effect of

Consummating the transac:ion resulting in an income

A deduction in the recipient's income tax return

A credit from the recipient's taxable income

A final settlement of the tax liability on the income

Final tax withheld is not creditable and deductible against any tax due. But is a tax liability on the part of payor to
remit the taxes withheld.

► Que stion 5 OI 1 pts

Receipt of t he fo llowing by a minimum wage earner is subject to income tax, except

Holiday Pay

Allowance

Commission

Fringe benefit

Basic minimum wage, Holiday pay, Hazard Pay, Overtime pay, and Night Differential Pay are legally exempt benefits which should be
presented as non-taxable compensation and deducted in the amount of taxable compensation income.

Que stion 6 1 / 1 pts

Paolo Montalban, worked in the United States as an actor. Has been residing in t he St at es for the past 15 yea rs. He arrived in t he
Philippines on July 1, 2021 t o reside here permanently. W hich of :he following statements is correct wit h respect to Paolo's
classification for income tax purposes?

He will be cla~sified as neither resident nor nonresident citizen for the year 2021.

He will be cla~sified as nonresident citizen for the whole year of 2C21.

He will be cla~sified as nonresident citizen from Janua ry 1, 2021 until the date of his arrival in t he Philippines.

He will be cla~sified as resident citizen for the whole year of 2021.

A citizen who has been previously considered as nonresident citizen and who arrives in the Philippines at anytime during the taxable year
to reside permanently in the Philippines shall likewise be treated as a nonresident citizen for the taxable year in which he arrives in the
Philippines with respect to his income derived form sources abroad until the date of his arrival in the Philippines.
Que stion 7 1 / 1 pts

Beginning April 11, 2021, which of t he following stat ements is correct?

A taxable part1crship shall be subjected to the improperly accumulated earnings tax.

Partners of a taxable part nership arc considered as stockholders and profits d istributed to them by t he partnersh ip are considered as
dividends.

Partner's share in the distributable income after tax of a business partnership shall be added to the partner's gross income for purposes of
the latter's persoral income tax.

Partners share in the distributable income after tax of a business partnership shall be subjected to the 10% creditable withholding tax.

For purposes of computing the distributive share of the partners of a general professional partnership, the net
income of the partnership shall be computed in the same manner as a corporation.
Improperly accumulated earnings tax is no longer applicable; The 10% creditable withholding tax is applicable for
partners of GPP but not business partnership.

Que stion 8 1 / 1 pts

One of the following is not an accept ed basis of relief from the MCIT.

lntcrcorporate dispute

Legitimate business reverses

Prolonged labor dispute

Force majcurc problems

Upon submission of proof that the corporation sustained losses on account of prolonged labor dispute, force majeure problems, and
legitimate business reverses, the Secretary of Finance, upon the recommendation of the CIR, may suspend the imposition of MCIT.

Que stion 9 1 / 1 pts

The following are t axable w ith income derived from sources within t he Philippines only, except:

M icro Corporation Philippine Branch. Registered branch of Micro Corporation USA.

M icro Corporation USA. a corporation created under the laws of the United States of America.

Philippine Bearing Inc. a corporation created under the laws of the Philippines. 99% owned by Bearing USA. Inc . a corporation created under
the laws of t he USA.

Continental Air M icroAsia. a corporation created under the laws of Malaysia who has landing rights in the Philippines.
Question 10 1 / 1 pts

Jenaly_n is a compensation income earner from JMV Inc. She did not have another employer and her income t ax is correctly
w it hheld by her employer
DiegQ - Deriving purely compensation income from Frodo Inc, had only one employer during t he year and employer properly
w it hheld from him t he correct amount of tax. Diego is marri ed to Amelia. A melia owns and operat es a nail salon and foot spa.

Only Diego can avail of the substituted filing.

Neither Jcnalyn nor Diego arc allowed to avail of the substituted filing

Jcnalyn and Diego arc both allowed to avail of the substituted filing

Only Jenalyn can avail the substituted filing

Jenalyn qualified to avail the substituted filing because she met all the qualifications required by the law:
1. those receiving purely compensation income, regardless of the amount,
2. those with only one employer in the Philippines for the calendar year, and
3. those whose income taxes have been withheld correctly by the employer

Diego cannot avail because he needs to file income tax return (1701 and 1701Q) together with his wife who has a separate business income, thus unqualified for a reason
that there is business income which should only be compensation income.

Question 11 1 / 1 pts

W hich of the following t ransactions is subject t o 6% capita l gains tax:

Sale of real property utilized for office use

Sale of apartment houses

Sale of vacant lot by an employee

Sale of condominium units by a rea l estate dealer

The sale, exchange, and other disposition of real property classified as capital asset located in the Philippines is subject to a tax of 6% of the
selling price or the fair value, whichever is higher. Thus, the sale of vacant lot by an employee is a real property classified as capital asset
while the other three options are considered real property classified as ordinary assets.

Question 12 1 / 1 pts

For income tax purposes, the source of t he service income is relevant for t he taxpayer, who is a:

Korean citizen who is married to a Filipina citizen and residing in their family home located in Fort 3onifacio. Taguig City:

Filipino citizen residing in Makati C ty

Domestic corporation

Non-resident Filipino citizen working and resid ing in London, United Kingdom

A nonresident citizen is taxable only on income derived from sources within the Philippines; Services. - Compensation for labor or
personal services performed in the Philippines.
► Que stion 13 OI 1 pts

ABC, Inc. purchased a residential house and lot w ith a swimming pool in an upscale su bdivision and required t he company
president t o stay t here w it hout paying rent ; it reasoned out that t he company president must maint ain a certain image and be able
to ent ert ain guest s at t he house to promot e t he company's business. The compa ny president declared t hat because t hey are
childless, he and his w ife could very well live in a smaller house.

Was t here a taxa ble fringe benefit ?

There w as a taxable fringe benefit since the stay in the house was for free.

There was no taxable fringe benefit since it was for the convenience of the employer and was necessary for its business.

There was no taxable f ringe benefit because the company president was only required to stay there and did not demand free housing. Thus.
given for the advantage of the employer

There was a taxable fringe benefit because the house was very luxurious.

Que stion 14 1 I 1 pts

Share in t he net distributable income of a general professional partnership by a resident citizen is subject to:

6% capital gains tax

20% final withholding tax

Regular income tax at 0% to 35%

10% final withholding tax

Partner’s share in GPP is subject to graduated income tax rate from 0% to 35%. The 10% and 20% withholding tax
is not final tax but creditable withholding tax.

Que stion 15 1 I 1 pts

ABC Corp. secured an income t ax holiday for five (5) years as a pioneer indust ry. On t he 4 t h year of t he tax holiday, ABC Corp.
decla red a nd pa id cash d ividends t o it s stockho lders, all o f whom ar e individua ls. A re t he d ividends taxa b le?

The dividends arc taxable if they c~cccd 50% of ABC's retained earnings.

The dividends arc exempt if paid before the end of ABC's fiscal year.

The dividends are taxable; the tax exemption of ABC Corp. docs not ext end to its stockholders.

The dividends arc tax exempt because of ABC's income tax holiday.

ABC Corp. and its stockholders are separate tax entities under the NIRC. Consequently, ABC Corp.’s tax exemption does not extend to its
stockholders.

Under the NIRC, stockholders who receive dividends from a domestic corporation are subject to the following scheduler income tax rates:
10% for Filipino citizens and individual resident aliens; 20% for non-resident aliens engaged in trade or business; and 25%/15% for non-
resident foreign corporations. Thus, the stockholder’s claim for the tax exemption is unmeritorious.
Que stion 16 1 / 1 pts

A non-st ock, non-profit entity shut s down and transfers all its property in favor of another entity. It will not be subject to tax if it
meet s t he requirement s of the law, unless:

The transferee is a non-gosernment organization.

Not more than 30% of the donation is used for administrative purpose.

The transferee is another mn-stock. non-profit organization.

The transferee is an educational. charitable. religious. cultural or social welfare. philanthropic organization.

The other three options are considered exempt entities not subject to income tax.

► Que stion 17 OI 1 pts

Manuel Rodriguez, CPA practitioner, is also employed with a medium-sized manufacturing firm and receives the following for the
yea r 2019:

Salary- P 200,000
Overtime pay - PS0,000
13th month and other benefits - P20,000
Salary 200,000
Deposit of client - Pl00,000
Overtime pay 50,000
Retainer fee of P25,000/month - P300,000
Out -of -pocket charged to client - P400,000
Dividend income from domestic corporation - P30,000

How much is Manuel Rodriguez gross compensation income?

PS00.000

P270.000

P250.000

PSS0,000
Question 18 1 / 1 pts

W hich of t he following statement/s is/are correct ?

St at ement A - Non-resident citiizens are taxa ble on all income derived from sou rces wit hin the Philippines.
St at ement B - Resident citizens and domestic corporations are taxable on all income derived from sources w it hin or wit hout the
Philippines.
St at ement C - Foreign co rporations, w het her engaged or not in t rade or business in the Philippines are taxable from sources within
or without t he Philippines.
St at ement D - Aliens, whet her resident s or not in t rade or business in t he Philippines are taxable from sources w ithin or w ithout t he
Philippines.

Statements A and B are correct

Statements C and D arc correct

Statements A.C and D arc correct

Statements A. Band D arc correct

Question 19 1 / 1 pts

Mark Marquez, is a practicing lawyer, w ho is engaged in providing legal services to his client s. He is also a licensed real est ate
broker. Bot h businesses were registered wit h t he BIR and had t he following data made available for yea r 201 8 (a mounts are gross
of w ithhold ing tax):

► Revenu es from his profession as a lawyer amounti ng to P3,000,000;


► Commission received as a real estat e broker amounts t o P600,000;
► Cost of services and business relat ed expenses amount ed to P?00,000 and P400,000, respectively;
► Gain on sale of real property held fo r invest ment amount s to PS00,000 (selling price is P2,000,000);
► Wagering gain amounti ng to PS0,000 (net of wageri ng losses of Pl 00,000);
► Royalties from books published and so ld amounti ng t o Pl S0,000; and
► Interest income from banks amounting to P30,000.

Compute for t he t ax due of the taxpayer, assuming M r. Marquez opt ed to use itemized deduction in computi ng his income tax.

P826.000

P810.000

P666,000

P698.000

Revenues 3,000,000
Tax due = 2,550,000 – 2,000,000 = 550,000 x 32% = 176,000 +
Commission 600,000 490,000 = 666,000
Wagering gain 50,000
Total income 3,650,000
Less: Cost of services 700,000
Gross income 2,950,000
Less: Business expense 400,000
Net income 2,550,000
► Question 20 O I 1 pts

Mark Marquez, is a practicing lawyer, who is engaged in providing legal services to his clients. He is also a licensed real estate
broker. Both businesses were registered with the BIR and had the following data made available for year 2018 (amounts are gross
of withholding tax):

► Revenues from his profession as a lawyer amounting to P3,000,000;


► Commission received as a real estate broker amounts to P600,000;

► Cost of services and business related expenses amounted to P700,000 and P400,000, respectively;

► Gain on sale of real property held for investment amounts to PS00,000 (selling price is P2,000,000);

► Wagering gain amounting to PS0,000 (net of wagering losses of Pl00,000);

► Royalties from books publislhed and sold amounting to PlS0,000; and

► Interest income from banks amounting to P30,000.

How much is the final withholding income tax of Mr. Marquez, assuming 2/3 of interest income from banks were realized from time
deposits with maturity of 5 years?

P 17,000

✓ P137.000

P47.000

P 21.000

Question 21 1 I 1 pts

A taxpayer married, with three minor children, provided the following data for 2018:

Compensation income (10% represents SSS, Pag-lBIG, Philhealth & Union dues contributions) - P 275,000
Income from merchandising - P3,500,000
Other income (20% represents income from bank deposits abroad) - P250,000
Expenses (15% represents personal expenses) - P2,400,000
Income from treasury bills - Pl00,000

Additional information:
¼ of business income and deductible business expenses is from outside the Phil'ippines.

If the taxpayer is a resident citizen, his taxable income is:

P 1,957.500

P 1.350.000

P 1.542.000

P 1.625.000
Question 22 1 / 1 pts

A taxpayer married, with t hree minor children, provided t he following data for 2018:

Compensation income (10% represents SSS, Pag-lBIG, Philhealth & Union dues co nt ributions) - P 275,000
Income from merchandising - P3,500,000
Other income (20% represent s income from bank deposits abroad) - P250,000
Expenses (15% represents personal expenses) - P2,400,000
Income from treasury bills - Pl 00,000

Add itional information:


¼ of business income and deductible business expenses is from outside the Philippin es.

If t he taxpayer is a non-resident alien ETB, his taxable income is:

P 1.350.000

P 1.907.500

P 1,542.500

P 1.625.000

► Question 23 OI 1 pts

A taxpayer married, with t hree minor children, provid ed t he following data for 2018:

Compensation income ('10% represents SSS, Pag-lBIG, Philhealth & Union dues co nt ributions) - P275,000
Income from merchandising - P3,500,000
Other income (20% represent s income from bank deposits abroad) - P250,000
Expenses (15% represents personal expenses) - P2,400,000
Income from treasury bills - Pl 00,000

Add itional information:


¼ of business income and deductible business expenses is from outside the Phil'ippin es.

If t he taxpayer is a non-r esident alien not engaged in trade or business in the Philippines, his t axabl e income is:

P3.250.000

P3.172.500

P3,072.500

P4.075.000
► Que stion 24 OI 1 pts

A taxpayer married, with t hree minor child ren, provided th e following data for 2018 :

Compensation income (10% represents SSS, Pag-lBIG, Philhealth & U nio n dues co ntributions) - P275,000
Income from merchand ising - P3,500,000
Other income (20% represent s income from bank deposits abroad) - P250,000
Expenses (15% represents personal expenses) - P2,400,000
Income from t reasury bills - Pl 00,000

Add itional information:


¼ of business income and deductible business expenses is from outside the Philippines.

Assuming that the taxpayer is a non-resident c itizen, is he q ualified to ava il of t he 8% income tax rate based on gross sales and/ o r
gross receipts and oth er non-operati ng income?

Yes. because te is a mixed income earner (compensation and self-employment income).

No. because his 20 18 gross sales and/ or gross receipts exceeded the P3 M illion t hreshold as a compulsory VAT taxpayer. registered or not.

Yes. if he opts to be taxed at the 8% income tax rate duly signified in his 1st quarter income tax return. due to his actually failing to meet the
over P3 Million gross sales and/or gross receipts threshold as compulso)ryVAT taxpayer in 2018.

No. because there is no indication at all of his signifying his intention to be taxed at the 8% income tax rate in his 1st quarter of 2018
income tax return.

► Que stion 25 OI 1 pts

M r. I LUSTRE, a Fina ncial Compt roller of an Automotive firm, ea rned annual compensation income in 2019 of Pl ,500,000, incl usive
of 13th month pay and ot her benefits in the amount of P120,000 but net of mandatory contributions to SSS, Pag- lBIG and
Philhealth. Aside from employment income, he owns a medium-sized grocery store w ith gross sa les of P2,400,000. His cost of sa les
and operating expenses are Pl ,000,000 and P600,000, respectively, and w ith non-operating income of Pl 00,000.

W hich is t he better option t o choose from: (a) t he specia l income tax rat e of eight percent (8%); or (b) th e graduat ed income t ax
rates of 0% - 35% under Sec. 24 (A)(2)(a) of the Tax Code, as amended?

It's always the 8% income tax rate because of the twin benefits of not only avoiding the graduated income ta> rates under Sec. 24 (A)(2)(a)
but also the exemption from the 3% OPT under Sec. 116. both in the NIRC. as amended.

Indifferent. as the two options will result in the same total national internal revenue taxes liability anyway o)f the taxpayer.

8% income tax rate based on taxable income.

Progressive rates under Sec. 24 (A)(2)(a).


Que stion 26 1 / 1 pts

Mr. I LUSTRE, a Financial Compt roller of an Automotive firm, ea rned annual compensation income in 2019 of Pl ,500,000, incl usive
of 13th month pay and ot her benefits in the amount of P120,000 but net of mandatory contributions to SSS, Pag-lBIG and
Philhealth. Aside from employment income. he owns a medium-sized grocery store w ith gross sa les of P2.400.000. His cost of sa les
and operating expenses are Pl,000,000 and P600,000, respectively, and w ith non-operating income of Pl 00,000.

W hat if the taxpayer chose to be taxed the progressive income tax rat es of 0% - 35% under Sec. 24{A)(2)(a) of th e Tax Code, as
amended, the total income tax and bu siness tax due and paya ble for 2019 is:

P661.200

P 589,200

P664.200

P877.200

► Que stion 27 OI 1 pts

Ferdinand is a sole proprietor in t rading business, w ith his place of bu sin ess in a bu ilding w here he uses t he first floor as hi s store
and th e second floor as his residence. In 2019, he made t he following payments of taxes:

Regist ration for 2019 of the business wit h BIR - PS00


Income tax for 2018 - P200,000
Va lue-added tax (including int erest of P20,000 and surcharge of P 60,000) - P450,000
Fringe benefit tax t o his Manager - P35,000
Community Tax - P2,005
Real est ate tax on his land and building - P8,000
City business t axes and Mayor's permit - Pl 0,000
Motor vehicle registration fee to t he LTO (ca r is for non-bu siness use), includ ing Int erest of P 500 and surcharge for penalty of P
2,000 - P4,500

The deduction for taxes expense in 2019 income tax ret urn is:

P 53,505

P 51,505

P 71,505

P 55,505
► Que stion 28 O I 1 pts

W hich is not correct? The net operating loss ca rry-over (NOLCO) is:

Not available to a general professional partnership in the Ph ilippines.

Available to a domestic corporation.

Available to an individual in business in the Philippines.

Available to a registered general partnership in business in the Philippines.

Que stion 29 1 I 1 pts

Jerr y receiveu li re rulluwi1115 irrw rrre irr 2018:

Business income, Philippines - P300,000


Business income, Unit ed States - P250,000
Expenses, Philippines - P200,000
Expenses, United Stat es - P125,000
Interest on deposit w ith Metrobank - P3,000
Cash prize won in a local contest - P6,000
Cash prize won in a contest in U.S. - Pl0,000
W innings in lotto - P20,000
W innings in lotto in U.S. - PS0,000
Dividends from SMC, a domestic compa ny - P25,000
Interest on deposit in U.S. ($1 = P52) - $500

The taxable income if Jerry is a resident citizen, single, is:

P267.000

P317,000

P106.000

P88.500
Question 30 1 / 1 pts

A domestic corporation's computed NCIT and MCIT and creditable income taxes withheld at source from 1st to 4th quarters,
including excess MCIT and excess withholding taxes from prior year/s, are as follows:

Excess Excess
Quarter NCIT MCI T Taxes W ithheld MCIT/NCIT withholding tax
prior year prior year/s

1st Pl 00,000 P80,000 P20,000 P30,000 Pl0,000

2nd P120,000 P250,000 P30,000

3rd P250,000 Pl 00,000 P40,000

4th P200,000 Pl 00,000 P35,000

The income tax due at t he end of t he first quarter is:

PS0.000

P80.000

P70.000

P40,000

Question 31 1 / 1 pts

MICHAEL, single, had the following during the t axable year 2018:

Philippines USA
Gross Income from Business P800,000 P400,000

Business Expenses PS00,000 P200,000

Renta l Income, Gross P200,000 Pl 00,000

Interest Income on Bank Deposit Pl 00,000 P20,000

Roya lty Income P60,000 P40,000

Dividend Received P80,000 P30,000

If M ICHAEL is a non-resident citi zen, his taxable income is:

PS00,000

P450.000

P840.000

P870.000
► Question 32 OI 1 pts

A domestic corporation, in it s fourth yea r of operations, had:

Gross profit from sales - P 3,000,000


Expenses of operations - Pl ,000,000
Quarterly income t axes paid - PS00,000
Dividend from domestic corporation - P200,000
Interest on bank deposit - Pl00,000
Gain on sa le of ca pital asset:
Land and building outside t he Philippin es, on a selling price of PlS,000,000 and cost of Pl 0,000,000 - PS,000,000
Shares of stock of a domestic co rporation, not listed and t raded ir a local stock exchange, on a selling price of P350,000 and cost of
Pl 00,000 - P250,000

W hat is the income t ax still due fo r the year?

None of the Aoove.

P150.000

P100.000

P200.000

Question 33 1 I 1 pts

A corporation has the following income and expenses for 2018 (fifth yea r of operations):

Gross income, Philippines - P2,400,000


Gross income, U.S. - Pl ,600,000
Business expense, Philippines - Pl ,000,000
Business expense, U.S. - P800,000
Unallocated business expense - P480,000
Interest expense in con nection w ith U.S. business - Pl 00,000
Interest expense in con nection w ith Philippine business - P200,000
Interest expense in con nection w ith business in the Philippines and U.S. - P160,000

The co rporate income t ax due if a resident foreign corporation:

P 216.120

P 285.600

P48.000

P 244,800
Que stion 34 1 / 1 pts

SNOWDROP Corporation made the following payments for the 3rd quarter of 2018 in behalf of its Executive Vice President:

1) To SM Supermarket, groceries consumed by the officer's family - P97,500


2) To Saint Lou is University, tu ition of EVP's eldest son (one of the company scholars chosen after passing t he compa ny's qualifying
examination) - P30,000
3) Brand new ca r leased from Rent -A-Car for use of the officer (quarterly rent al) - P32,500

The fringe benefit tax for the quarter is:

PSS.168

P61,250

P67.012

P57.600

► Que stion 35 OI 1 pts

Glenn is a dealer of household 'urniture regularly selling on in stallment s. It made a sale of a bedroom set, wit h a cost of P4,000, for
Pl 0,000, receiving as downpayment Pl,000 on the dat e of sa le on December 15, 2018, and Pl ,500 t he day after, and mont hly
t hereafter at P2,500.

The gross income to report on :he inst allment method in 2019 is:

Zero

P4.500

P 1,500

P7.500
► Question 36 OI 1 pts

Francisco is a single propri etor in trading business, w ith his place of busin ess in a building w here he uses t he first floor as his store
and th e second floor as his resi dence. In 2018, he made t he following payments of taxes:

1) Registration fee for 2018 of the business wit h the Bureau of Internal Revenue - P 500
2) Income t ax of 2017 - P200,000
3) Va lue-added t axes (incl uding int erest of P20,000 and surcha rges of P60,000) - P420,000
4) Fringe benefit tax to his manager - P32,000
5) Community tax - P2,005
6) Rea l estate tax on his land and building - P7,000
7) City business taxes and city mayor's permit - P5,500
8) Mot or vehicle registration fee (ca r is for non-bu sin ess use) Including interest of P50 and surcharge for penalty of Pl ,500 - P3,050

The deduction for taxes is:

P 211.505

P638.055

P43.505

P 11,005

► Question 37 OI 1 pts

Mr. Garcia, a cemetery lot dealer sold real properties to different buyers as follows:

Cost
House & Lot P2,000,000 Pl ,250,000
Farm Lot P800,000 P300,000
Cemetery Lot P45,000 P20,000

The house and lot was M r. Garcia' principal residence that was sold to reacquire a new pri ncipa l residence noting that th e entire
proceeds thereof was utilized.

W hat is Mr. Garcia' capit al gains tax, assuming t hat all t he selling p rice are at fa ir market va lue?

P170.700

P48.000

P 50,700

P168,000
Question 38 1 / 1 pts

In 2020, DEL Corp. had t he following items of income and expenses:

Sales - PS00,000
Cost of services, sa lary of personnel direct ly engaged in business - P250,000
Dividends received - P25,000

The dividends were received from a domestic corporation. The general and administrative expenses i nclude factory facilities
expense and cost of manufactu ring supplies of P25,000 and PlS,000, respectively. W hat amount should be reported as gross
income for minimum corporat e income tax?

P235.000

P250.000

P210,000

P275.000

► Question 39 O/ 1 pts

Bench Corp., a calendar-year co rporation realized taxable income of P36,000 from its regular business operations fo r ca lendar-yea r
2020. In addition, Bench had t he following ca pit al gains and losses duri ng 2020.

Short-t erm capita l gain - PS,500


Short-t erm capita l loss - P(4,000)
Long-t erm ca pital gain - Pl,500
Long-t erm ca pital loss - P(3,500)

Bench did not rea lize any other capital gains or losses since it began operations.
W hat is Bench' total taxable income for 2020?

P38.500

P36.250

P40,500

P39.500
Que stion 40 1 / 1 pts

The Co ngress of th e Philippines enact ed a law im posing 5% tax o n a specific t ransaction. The t ax law
may be considered valid because:

It is equitable

It is for unity

It is generally payable in money

It is for public purpose

Que stion 41 1 / 1 pts

The assessment of BIR aga inst Nica nor became final and executory. The BIR conduct ed sum mary
proceedings to collect t he delinquent taxes. The BIR det ermined that Nicano r had real pro perties hence
t he same were levied. The Bl R's action is in consona nce with:

Power of Unity

Power of Eminent Domain

Power of Taxation

Police Power

Question 42 1 / 1 pts

W hich is FALSE in the application of the principle of Fiscal Adequacy in case of deficit?
I. The government may lend money
II. The government may impose new taxes
Ill. The government may sell st ate properties
IV. The government may suspend ca pital expenditures

IV

II

• I

Ill
Question 43 1 / 1 pts

During audit, t he BIR discovered t hat ABC Corp. does not keep books of account s. ABC Corp. in it s SEC
and BIR registration, it indicated t hat its accounting period is fisca l yea r. Which is T RUE?

The BIR can compel the ta>.payer to change its taxable period

The BIR needs to follow taxpayer's registered taxable year

The BIR can issue an assessment using the calendar year

The BIR. in issuing an assessment. can only consider the fiscal year period being applied by ABC Corp.

Question 44 1 / 1 pts

The BIR is under t he direct supervision of:

The Office of the President

The Department of Interior and Local Government

The Department of Finance

The Department of Trade and Industry

► Question 45 O I 1 pts

The following are powers of t he Commissioner, except:


I. To change venue of fi ling of ret urns
II. To extend period of filing of ret urns
Il l. To compel taxpayers to appea r for it s failure to provide requested documents
IV. To decide protested assessments

IV

Ill

II
► Que stion 46 O I 1 pts

S1: lnday, a model, is a BIR-registered professional. In 2021, she ventu red into vlogging w here she earned
Php 1,000,000 on top of her modelling income of Php 1,000,000.00. In 2021, lnday is classified as a M ixed
Income Earner.

S2: Nicanor is unemployed. He t ried moto-vlogging as a hobby. He was able to monetize Php 1,000 based
on t he views he was able to generat e on his YuTube Channel. Nicanor is required to register as a
professional, issue official receipt s and keep books of account s.

Only S2 is true

Both are False

Both arc True

Only S1 is true

Que stion 47 1 I 1 pts

ABC Corp. a PAGCOR-licensee t o operat e offline gaming in t he Ph ilippines employs aliens. The following
are t he t erms of employment of its Chinese employees:

Name Position Period of Employment Amount of Salary

Wa n President 1 Year Pl ,000,000

Ton Consu lt ant 6 Mont hs PS00,000

Sio Dealer 3 Mont hs P200,000

Mai Interpret er 1 Mont h Pl00,000

Wh ich is TRU E?

Wan is subject to withholding tax on compensation

All arc subject to 25% final tax

Sia and Mai arc exempted from taxation because their income is below Php 250.000

Ton is subject to expanded withholding tax


Question 48 1 / 1 pts

W hich is FALSE?
I. Nicanor, an employee, is not qualified t o use 8% preferential taxation.
II. Nicano r, a sari-sa ri store owner, is allowed t o use commut ed tax of 8%.
Il l. Nicanor, a social media influencer and apo logist was paid Php 5,000,000. He has t he o ption t o be
taxed at 8% or graduat ed tax rate.
IV. Nicano r, a VAT-regist ered t axpayer, w hose gross sa les is only Php 1,000,000.00 is not qualified to
use 8% rat e

Ill and IV

IV

Ill

11.111 and IV

Question 49 1 / 1 pts

W hich is FALSE?
I. Nicanor, a sa ri -sari sto re owner, opted commuted tax of 8%. He is allowed to cla im deduction of
Php 250,0:>0.00 o n his gross sales.
II. Nicano r. a socia l media influencer and apologist was pa id Php 5,000,000. He is allowed to claim
deduction of Php 250,000.00 o n his g'oss receipts.
Il l. Nicanor, a VAT-registered taxpayer. w hose gross sales is only Php 1,000,000.00 is allowed t o
claim deduction of Php 250,000.00 on his gross sa les.

Ill

II and Ill

II

1.11 and Ill


Question SO 1 / 1 pts

W hich is FALSE? Assume all incomes are derived from w ithin t he Philippines
I. Nicanor, a non-resident citizen w ho is engaged in t rade or business has option to use 8% or
graduated tax rate
II. Nick Aynor, a resident alien who is in t he practice of hi s professilon can opt to use 8% t axation
Il l. Nica Knorr, a non-resident alien who is engaged in t rade or business ca n use eit her graduated or
8% t axation
IV. Nick Nore, a non-resident alien w ho is not engaged in t ra de or business ca n use graduated tax
rate
V. Est at e of Nicanor ca n only use graduate tax

Ill and IV

II. Ill. IV and V

IV

IV and V

Question 51 1 / 1 pts

S1 - Nicanor, an NRC based in Canada opened a bank account in A BC Bank, a DC. The source of t he
interest income of Nicanor is derived from wit hin t he Philippines.
S2 - Nicanor, an RA, opened a bank account in ABC Bank, a DC. The source of t he int erest income of
Nicanor is derived from wit hin t he Philippin es.
S3 - Nicanor, an NRAETB opened a bank account in ABC Bank, a DC. The source of t he interest income of
Nicanor is derived from wit hin t he Philippin es.
S4 - Nicanor, an NRANETB opened a bank account in ABC Bank, .a DC. The source of t he interest income
of Nicanor is derived from w it hin t he Philippines.

W hich is FALSE?

53

54

51

None.

52
Que stion 52 1 / 1 pts

S1 - Nicanor, an NRC based in Ca nada opened a bank accou nt in ABC Bank, a NRFC. The income is subject
to 20% fl naI t ax.
S2 - Nicanor, an RA, opened a bank accou nt in ABC Bank, a DC. The w ithholding agent is t he bank.
S3 - Nicanor, an NRAETB opened a bank account in ABC Bank, a DC. The interest income is not required
to be repo rt ed by Nicanor.
S4 - Nicanor, an NRANETB opened a bank account in ABC Bank, an NRFC. The interest income is exempt
from taxation

W hich is FALSE?

52

S1

53

54

► Que stion 53 O I 1 pts

W hich is TRU E? A BC Corp declared dividends.

S1- If ABC Corp is DC the source of the income is derived from w ithin t he Philippines
S2 - If ABC Corp is RFC t he source of the income is derived from within the Philippines if t he dominant income
last yea r is Philippine income
S3 - If ABC Corp is NRFC t he source of t he income is derived from wit hin the Philippines if t he dominant income
last yea r is Philippine income
S4 - If ABC Corp is a foreign corporation, t he source of th e income is derived from w ithout t he Philippines

S1 &54

All arc True.

54

51

51. S2 & S3
► Que stion 54 OI 1 pts

W hich is FALSE in withholding tax syst em?


S1 - All passive income derived from within the Philippines are subject t o final wit hholding tax
S2 - Income payments are generally subject to withholding tax
S3 - If an income is exempted from taxation, it is not subject to wit hholding
S4 - A w ithhold ing agent is required to w it hhold and remit taxes on all its income payments

54

52 and S3

51. S2 and 54

51 and S4

S1

Que stion 55 1 I 1 pts

S1: ABC Corporation was identified as a t op w ithholding agent by t he BIR. In case th e company
purchases raw material goods for its production, the co mpany is required to w it hhold 1% expanded
w ithholding tax for ·ts purchases.

S2: ABC Corporation was identified as a t op w ithholding agent by t he BIR. In case th e company purchases raw
mat erial goods for its production, the company is required to w ithhold 2% expanded withholding t ax.

Only 52 is true

Both arc False.

Both arc True.

Only S1 is true
► Que stion 56 OI 1 pts

S1: ABC Corporation was identified as a top w ithholding agent by t he BIR. In case th e company engages
t he services of XYZ Corpo ration, a security agency company, to p·ovide secu rity guards to the former,
ABC Corporation is required to w ithhold 1% expanded wit hholding tax for t he amount billed by XYZ
Corporation (excluding t he sa laries of the guards).

S2: ABC Corporation was identified as a t op w ithholding agent by t he BIR. In case th e company engages t he
services of XYZ Corporation, a secu rity agency compa ny, to provide security guards t o t he former, ABC
Corporation is requ·red to w ithhold 2% expanded wit hholding tax for t he amount billed by XYZ Corpo ration
(excluding t he salaries of the guards).

Both arc False.

Both arc True.

Only S1 is true

Only S2 is true

► Question 57 OI 1 pts

ABC Corporation was identified as a top w it hholding agent by t he BIR. One day, the accounti ng
department, headed by Nicano', request ed t he company to t reat all of its staff a Jollibee mea l amounting
to Php 9,000 because it is already tax season, and the employees are working a minimum of 18-hours a
day. The compa ny grant ed th e request. W hich is FALSE?

The transaction is not subject to w ithholding tax on compensation even if it is for employee benefits.

The transaction is not subject to final withholding tax.

The transaction is exempt from expanded withholding tax.

The transaction is not subject to expanded withholding tax but subject to withholding tax on compensation.
Que stion 58 1 / 1 pts

ABC Corporation was identified as a t op w ithholding agent by the BIR. One day, t he accounti ng
department, headed by Nicanor, request ed the company t o t reat all of its staff a Jalibee meal amounting
to P 1,000 because it is already tax season, and the employees are working 18-hours a day. The company
grant ed the request Within t he taxable year, t he company t ra nsact ed w ith Jalibee five times for buying
Jollibee meals w ith a t ot al amount of Php 7,000. As a result of purchases duri ng t he taxable year, th e
compa ny is:

Required tow thhold expanded withholding tax of 2%

Required to provide tax credit certificate to Jalibcc

Required tow thhold expanded withholding tax of 1%

Not required to withhold expanded w ithholding tax

► Question 59 OI 1 pts

ABC College, a non-stock, non-profit educational institution, operat es a ca nteen, bookstore, and
dormitory inside its campus which are commercia l in nature. The income from the sa le of goods and/ or
services by the ca nt een, bookstore, and dormitory in side t he campus are act ually, direct ly, and excl usively
used for educational purposes. Using t he data provided, which is FALSE?

S1 - Income from relat ed activity is exempt from t axation


S2 - Income from unrelat ed activity is subject to regular tax
S3 - Income from unrelat ed activity is exempt from income t ax

All arc True.

S3

52

51
► Que stion 60 O I 1 pts

ABC Corp is required to file income tax ret urn s and the following are t he deadlines assuming it applies
ca lendar year basis, except:
S1 - 1st Quarter - May 31
S2 - 2nd Quart er - August 29
S3 - 3rd Quarter - November 29
S4 - A nnual - April 15

51

53

52

S4

► Question 61 O I 1 pts

Nicanor is filing his annual income tax ret urn . His comput ed income t ax due is Php 5,000 and he has tax
credits amounting t o Php 4,000. W hich is TRUE?

He is not allowed to pay on instal lment because the payable is below Php 1.000

He can pay the balance in installment. Php 500 upon filing and Php 500 before October 15

He will be subjected to penalties if he pays on April 15

He can pay not hing during the annual filing and pay the balance of Php 1,000 on or before August 15
Que stion 62 1 / 1 pts

In the deadline of fi ling of 2020 Annual Income Tax Returns, w hich is FALSE?

51 - ABC Corp. employing fisca l year ending January 31, t he deadline is May 15 of t he same year
52 - ABC Corp. employing fisca l year ending June 30, t he deadline is Octo ber 15 of t he same yea r
53 - ABC Corp. employing fisca l year ending October 31, t he deadline is Februa ry 15 of the same year
54 - ABC Corp. employing calendar, t he deadline is April 15 of the following yea r

S3

51

54

52

► Que stion 63 O I 1 pts

In capita l gains tax, which is FALSE?

51 - The seller is t he person liable to pay t he capital gains tax if there is no agreement
52 - The buyer is t he o ne liable if t here's an agreement but the tax so paid is part of t he
consideration

Only S2 is True

Only 51 is True

Both arc False

Both arc True


Question 64 1 / 1 pts

W hich is TRUE in fringe benefits t ax?

S1 - Supervisory employees are t hose who, in t he interest of t he employer, effectively recommend such
managerial actions or t he exercise of such aut hority is not merely routinary or clerica l in nat ure but requires the
use of independent judgment
S2 - Managerial employees are ones w ho are vested wit h powers o r prerogatives to lay down and execut e
management policies and/ or hire, suspend, lay-off, recall, discharge, assign or discipline employees
S3 - Rank-and-File employees are employees w ho are holding neither managerial nor supervisory position

51

A ll of the above

52

53

Question 65 1 / 1 pts

W hich is FALSE in fringe benefits tax?

S1 - Monetary va lue of fri nge benefit less fringe benefits is equa l to fringe benefit s tax.
S2 - Grossed-up monet ary va lue of fringe benefits less of monetary va lue of fri nge benefit s is equal to fringe
benefit s tax.
S3 - Fri nge benefit s tax plus monetary valu e of fringe benefit s is equal to grossed-up monetary va lue of t he fringe
benefit s.

S1

52

52 and S3 arc True.

53
Question 66 1 / 1 pts

W hich is TRU E?
S1 - All incomes are generally taxable.
S2 A ll passive incomes a rc generally su bject t o final tax.

52 is true

Both arc True

S1 is tr ue

Both arc False

► Question 67 OI 1 pts

W hich is FALSE?

S1 - Nicanor is a minimum wage earn er. His total compensation including overtime pay, night shift differential
and hazard pay amounted to Php 300,000. He is exempt from income t ax.
S2 - Nicanor's daily wage amount ed t o Php 600.00 per daywhic~ is beyond t he statut ory minimum wage of Php 537. His salary is
subject to w ithholding t ax on compensation.

52

Both are True

51

Both arc False


► Question 68 O I 1 pts

W ho is required to file Annua l Income Tax Return (AITR)?

No. of Employer Income Tax Due Tax W it hheld by Employer


Nicanor 1 Pl 0,000 P11,000
Jose 2 (m inimum wage ea rner) Lero Lero
Andres 1 Pl 0,000 P9,000
Goyo 2 Pl 0,000 Pl 0,000

Nicanor and Andres

A ll Except Jose

Andres and Jose

All of them arc required

Question 69 1 I 1 pts

A non -resident alien not engaged in trade or business (N RAN ETB) shall generally be su bject t o a 25% fi nal t ax on gross income
received from all sources within t he Philippines. However, a NRANETB sha ll be exem pt from t ax on:

De min imis prizes of less than Pl0.000.

Interest income from a long-term deposit or investment certificate issued by a bank in the Philippines. and held for a period exceeding 5
years.

Interest income paid by a depositary bank under the foreign currency deposit system.

Philippine Charity and Sweepstakes Office (PCSO) winnings less t han Pl0.000.

Question 70 1 I 1 pts

The following are t he feat ures of t he 8% optional income t ax in th e IT R, except:

The qualified taxpayer. who is also a partner in a general professional partnership, must signify his cho ice of the 8% tax rate in the first
quarterly ITR.

The qualified taxpayer is exempt from paying the 3% OPT under Section 116 of the Tax Code.

The qualified individual tax:>aycr is not required to file his financial statements with his Annual ITR.

The qualified taxpayer is still required to comply with bookkeeping and invoicing requirements.
MIDTERM
EXAM
V2
YELLOW HIGHLIGHT – SUGGESTED/CORRECT ANSWER

Question 1
The following income are subject to final tax, except:
a) Cash dividend received by a domestic corporation from a resident foreign corporation.
b) Interest income received by a resident foreign corporation from a Philippine bank.
c) Stock dividends received by a domestic corporation from another domestic corporation.
d) Royalty income received by a domestic corporation from another domestic corporation.

Normally stock dividends are not income for income taxation purposes, thus exempt from income
tax.

Question 2
The MCIT shall not apply to the following Foreign Corporations, except:
a) RFC engaged in business as international air carrier subject to 21/2% of its gross Philippine
billings.
b) RFC engaged in importing Cranes and Heavy equipment into the Philippines
c) Non-resident Foreign Corporation (NRFC) on gross income derived from lease of vessel
d) RFC engaged in business as a Regional Operating Headquarters (ROHQ) for the year
ending December 31, 2021.

MCIT is applicable to every corporation taxable to regular corporate income tax including non-
profit, exempt, and special corporations with respect to their taxable income subject to regular
corporate income tax, but not to their income subject to special tax rates.

Thus, international air carries, ROHQ, and NRFC are MCIT exempt entities.

Question 3
Which of the following statements is not correct?
a) A co-owner is subject to income tax on his share in the net income of the co- ownership
actually or constructively received.
b) Partnerships, no matter how created or organized are considered corporation subject to
corporate income tax.
c) When the co-owners invest the income of the property co-owned in a business or in any
income producing properties or activities constituting themselves into a business
partnership, such partnership is consequently subject to income tax as a corporation.
d) As a rule, co-ownership is not subject to income tax because the co-owners are limited to
the preservation and enjoyment of the property and the collection of the income there
from.

GPP is not subject to corporate income tax.


Question 4
The payor of passive income subject to final tax is require to withhold the final tax from the
payment due to the recipient. The withholding of the tax, has the effect of
a) A credit from the recipient’s taxable income
b) Consummating the transaction resulting in an income
c) A final settlement of the tax liability on the income
d) A deduction in the recipient’s income tax return

Final tax withheld is not creditable and deductible against any tax due. But is a tax liability on the
part of payor to remit the taxes withheld.

Question 5
Receipt of the following by a minimum wage earner is subject to income tax, except
a) Allowance
b) Commission
c) Holiday Pay
d) Fringe benefit

Basic minimum wage, Holiday pay, Hazard Pay, Overtime pay, and Night Differential Pay are
legally exempt benefits which should be presented as non-taxable compensation and deducted in
the amount of taxable compensation income.

Question 6
Paolo Montalban, worked in the United States as an actor. Has been residing in the States for the
past 15 years. He arrived in the Philippines on July 1, 2021 to reside here permanently. Which of
the following statements is correct with respect to Paolo’s classification for income tax purposes?
a) He will be classified as neither resident nor nonresident citizen for the year 2021.
b) He will be classified as nonresident citizen for the whole year of 2021.
c) He will be classified as resident citizen for the whole year of 2021.
d) He will be classified as nonresident citizen from January 1, 2021 until the date of his arrival
in the Philippines.

A citizen who has been previously considered as nonresident citizen and who arrives in the
Philippines at anytime during the taxable year to reside permanently in the Philippines shall
likewise be treated as a nonresident citizen for the taxable year in which he arrives in the
Philippines with respect to his income derived form sources abroad until the date of his arrival in
the Philippines.

Question 7
Beginning April 11, 2021, which of the following statements is correct?
a) A taxable partnership shall be subjected to the improperly accumulated earnings tax.
b) Partner’s share in the distributable income after tax of a business partnership shall be
added to the partner’s gross income for purposes of the latter’s personal income tax.
c) Partners share in the distributable income after tax of a business partnership shall be
subjected to the 10% creditable withholding tax.
d) Partners of a taxable partnership are considered as stockholders and profits distributed
to them by the partnership are considered as dividends.

For purposes of computing the distributive share of the partners of a general professional
partnership, the net income of the partnership shall be computed in the same manner as a
corporation.
Improperly accumulated earnings tax is no longer applicable; The 10% creditable withholding tax
is applicable for partners of GPP but not business partnership.
Question 8
One of the following is not an accepted basis of relief from the MCIT.
a) Intercorporate dispute
b) Prolonged labor dispute
c) Force majeure problems
d) Legitimate business reverses

Upon submission of proof that the corporation sustained losses on account of prolonged labor
dispute, force majeure problems, and legitimate business reverses, the Secretary of Finance, upon
the recommendation of the CIR, may suspend the imposition of MCIT.

Question 9
The following are taxable with income derived from sources within the Philippines only, except:
a) Philippine Bearing Inc, a corporation created under the laws of the Philippines. 99%
owned by Bearing USA, Inc, a corporation created under the laws of the USA.
b) Micro Corporation USA, a corporation created under the laws of the United States of
America.
c) Micro Corporation Philippine Branch. Registered branch of Micro Corporation USA.
d) Continental Air MicroAsia, a corporation created under the laws of Malaysia who has
landing rights in the Philippines.

Question 10
Jenalyn is a compensation income earner from JMV Inc. She did not have another employer and
her income tax is correctly withheld by her employer
Diego – Deriving purely compensation income from Frodo Inc, had only one employer during the
year and employer properly withheld from him the correct amount of tax. Diego is married to
Amelia. Amelia owns and operates a nail salon and foot spa.
a) Neither Jenalyn nor Diego are allowed to avail of the substituted filing
b) Only Jenalyn can avail the substituted filing
c) Only Diego can avail of the substituted filing.
d) Jenalyn and Diego are both allowed to avail of the substituted filing
Only Jenalyn qualified to avail the substituted filing because she met all the qualifications required
by the law:

1. those receiving purely compensation income, regardless of the amount,


2. those with only one employer in the Philippines for the calendar year, and
3. those whose income taxes have been withheld correctly by the employer

Diego cannot avail because he needs to file income tax return (1701 and 1701Q) together with
his wife who has a separate business income, thus unqualified for a reason that there is business
income which should only be compensation income.

Question 11
Which of the following transactions is subject to 6% capital gains tax:
Sale of real property utilized for office use
Sale of apartment houses
Sale of vacant lot by an employee
Sale of condominium units by a real estate dealer

The sale, exchange, and other disposition of real property classified as capital asset located in the
Philippines is subject to a tax of 6% of the selling price or the fair value, whichever is higher. Thus,
the sale of vacant lot by an employee is a real property classified as capital asset while the other
three options are considered real property classified as ordinary assets.

Question 12
For income tax purposes, the source of the service income is relevant for the taxpayer, who is a:
a) Filipino citizen residing in Makati City
b) Korean citizen who is married to a Filipina citizen and residing in their family home located
in Fort Bonifacio, Taguig City;
c) Non-resident Filipino citizen working and residing in London, United Kingdom
d) Domestic corporation

A nonresident citizen is taxable only on income derived from sources within the Philippines;
Services. - Compensation for labor or personal services performed in the Philippines.
Question 13
ABC, Inc. purchased a residential house and lot with a swimming pool in an upscale subdivision
and required the company president to stay there without paying rent; it reasoned out that the
company president must maintain a certain image and be able to entertain guests at the house
to promote the company’s business. The company president declared that because they are
childless, he and his wife could very well live in a smaller house.

Was there a taxable fringe benefit?


a) There was no taxable fringe benefit because the company president was only required to
stay there and did not demand free housing. Thus, given for the advantage of the
employer
b) There was a taxable fringe benefit since the stay in the house was for free.
c) There was a taxable fringe benefit because the house was very luxurious.
d) There was no taxable fringe benefit since it was for the convenience of the employer and
was necessary for its business.

First, the company president is not a rank-and-file employee. Thus, the housing benefit is subject
to fringe benefits tax pursuant to Section 33 of the NIRC and Section 2.33 (A) of the RR No. 03-98.
However, the housing benefit to the President may be for the convenience of the employer (PRT
Corp.) or necessary to its business.

Letter b’s explanation:


First, the company president is not a rank-and-file employee. Thus, the housing benefit is subject
to fringe benefits tax pursuant to Section 33 of the NIRC and Section 2.33 (A) of the RR No. 03-98.
Although the housing benefit to the President may be for the convenience of the employer (PRT
Corp.) or necessary to its business, still, it also inured to the benefit of the President as his stay
therein is for free. RR No. 03-98 also provides for the guidelines and valuation of fringe benefits
for purposes of computing the portion which shall be subject to fringe benefits tax in cases where
the fringe benefits entail joint benefits to the employer and employee. Thus, there was a taxable
fringe benefit.

Question 14
Share in the net distributable income of a general professional partnership by a resident citizen
is subject to:
a) 6% capital gains tax
b) Regular income tax at 0% to 35%
c) 10% final withholding tax
d) 20% final withholding tax

Partner’s share in GPP is subject to graduated income tax rate from 0% to 35%. The 10% and 20%
withholding tax is not final tax but creditable withholding tax.

Question 15
ABC Corp. secured an income tax holiday for five (5) years as a pioneer industry. On the 4th year
of the tax holiday, ABC Corp. declared and paid cash dividends to its stockholders, all of whom
are individuals. Are the dividends taxable?
a) The dividends are taxable; the tax exemption of ABC Corp. does not extend to its
stockholders.
b) The dividends are exempt if paid before the end of ABC’s fiscal year.
c) The dividends are tax exempt because of ABC’s income tax holiday.
d) The dividends are taxable if they exceed 50% of ABC’s retained earnings.
ABC Corp. and its stockholders are separate tax entities under the NIRC. Consequently, ABC Corp.’s
tax exemption does not extend to its stockholders.

Under the NIRC, stockholders who receive dividends from a domestic corporation are subject to
the following scheduler income tax rates: 10% for Filipino citizens and individual resident aliens;
20% for non-resident aliens engaged in trade or business; and 25%/15% for non-resident foreign
corporations. Thus, the stockholder’s claim for the tax exemption is unmeritorious.

Question 16
A non-stock, non-profit entity shuts down and transfers all its property in favor of another entity.
It will not be subject to tax if it meets the requirements of the law, unless:
a) The transferee is an educational, charitable, religious, cultural or social welfare,
philanthropic organization.
b) Not more than 30% of the donation is used for administrative purpose.
c) The transferee is another non-stock, non-profit organization.
d) The transferee is a non-government organization.

The other three options are considered exempt entities not subject to income tax.

Question 17
Manuel Rodriguez, CPA practitioner, is also employed with a medium-sized manufacturing firm
and receives the following for the year 2019:
Salary - P 200,000
Overtime pay - P50,000
13th month and other benefits - P20,000
Deposit of client - P100,000
Retainer fee of P25,000/month - P300,000
Out-of-pocket charged to client - P400,000
Dividend income from domestic corporation - P30,000
How much is Manuel Rodriguez gross compensation income?
a) P550,000
b) P250,000
c) P270,000
d) P500,000

Salary 200,000
Overtime pay 50,000

Question 18
Which of the following statement/s is/are correct?
Statement A - Non-resident citizens are taxable on all income derived from sources within the
Philippines.
Statement B - Resident citizens and domestic corporations are taxable on all income derived from
sources within or without the Philippines.
Statement C - Foreign corporations, whether engaged or not in trade or business in the
Philippines are taxable from sources within or without the Philippines.
Statement D - Aliens, whether residents or not in trade or business in the Philippines are taxable
from sources within or without the Philippines.

a) Statements C and D are correct


b) Statements A,C and D are correct
c) Statements A, B and D are correct
d) Statements A and B are correct

Question 19
Mark Marquez, is a practicing lawyer, who is engaged in providing legal services to his clients. He
is also a licensed real estate broker. Both businesses were registered with the BIR and had the
following data made available for year 2018 (amounts are gross of withholding tax):
➢ Revenues from his profession as a lawyer amounting to P3,000,000;
➢ Commission received as a real estate broker amounts to P600,000;
➢ Cost of services and business related expenses amounted to P700,000 and P400,000,
respectively;
➢ Gain on sale of real property held for investment amounts to P500,000 (selling price is
P2,000,000);
➢ Wagering gain amounting to P50,000 (net of wagering losses of P100,000);
➢ Royalties from books published and sold amounting to P150,000; and
➢ Interest income from banks amounting to P30,000.
Compute for the tax due of the taxpayer, assuming Mr. Marquez opted to use itemized deduction
in computing his income tax.
a) P810,000
b) P666,000
c) P826,000
d) P698,000

Revenues 3,000,000
Commission 600,000
Wagering gain 50,000___
Total income 3,650,000
Less: Cost of services 700,000__
Gross income 2,950,000
Less: Business expense 400,000__
Net income 2,550,000

Tax due = 2,550,000 – 2,000,000 = 550,000 x 32% = 176,000 + 490,000 = 666,000

Question 20
Mark Marquez, is a practicing lawyer, who is engaged in providing legal services to his clients. He
is also a licensed real estate broker. Both businesses were registered with the BIR and had the
following data made available for year 2018 (amounts are gross of withholding tax):

➢ Revenues from his profession as a lawyer amounting to P3,000,000;


➢ Commission received as a real estate broker amounts to P600,000;
➢ Cost of services and business related expenses amounted to P700,000 and P400,000,
respectively;
➢ Gain on sale of real property held for investment amounts to P500,000 (selling price is
P2,000,000);
➢ Wagering gain amounting to P50,000 (net of wagering losses of P100,000);
➢ Royalties from books published and sold amounting to P150,000; and
➢ Interest income from banks amounting to P30,000.
How much is the final withholding income tax of Mr. Marquez, assuming 2/3 of interest income
from banks were realized from time deposits with maturity of 5 years?
a) P137,000
b) P 47,000
c) P 17,000
d) P 21,000

Royalties 150,000 x 10% = 15,000


Interest 30,000 x 1/3 = 10,000 x 20% = 2,000
Gain on sale 2,000,000 x 6% = 120,000
Total final withholding tax 137,000

It must be noted that the 6% CGT is a final tax in nature.

Question 21
A taxpayer married, with three minor children, provided the following data for 2018:
Compensation income (10% represents SSS, Pag-IBIG, Philhealth & Union dues contributions) - P
275,000 Income from merchandising -
P3,500,000 Other income (20% represents income from bank
deposits abroad) - P250,000 Expenses (15% represents personal expenses) -
P2,400,000 Income from treasury bills -
P100,000 Additional
information: ¼ of business income and
deductible business expenses is from outside the Philippines. If the taxpayer is a resident citizen,
his taxable income is:
a) P 1,350,000
b) P 1,542,000
c) P 1,625,000
d) P 1,957,500
Compensation income 275,000 x .90 = 247,500
Business income 3,500,000
Deductible expense 2,400,000 x .85 = 2,040,000
Other income 250,000

Business income 3,500,000


Add: Other income 250,000
Total income 3,750,000
Less: Deductible expense 2,040,000
Net income 1,710,000
Add: Compensation income 247,500
Total taxable income 1,957,500

Question 22
A taxpayer married, with three minor children, provided the following data for 2018:
Compensation income (10% represents SSS, Pag-IBIG, Philhealth & Union dues contributions) - P
275,000 Income from merchandising -
P3,500,000 Other income (20% represents income from bank
deposits abroad) - P250,000
Expenses (15% represents personal expenses) - P2,400,000
Income from treasury bills - P100,000
Additional information:
¼ of business income and deductible business expenses is from outside the Philippines.
If the taxpayer is a non-resident alien ETB, his taxable income is:
a) P 1,907,500
b) P 1,542,500
c) P 1,350,000
d) P 1,625,000

Compensation income 275,000 x .90 = 247,500


Business income 3,500,000 x .75 = 2,625,000
Deductible expense 2,400,000 x .85 x .75 = 1,530,000
Other income 250,000 x .80 = 200,000

Business income 2,625,000


Add: Other income 200,000
Total income 2,825,000
Less: Deductible expense 1,530,000
Net income 1,295,000
Add: Compensation income 247,500
Total taxable income 1,542,500

Question 23
A taxpayer married, with three minor children, provided the following data for 2018:
Compensation income (10% represents SSS, Pag-IBIG, Philhealth & Union dues contributions) -
P275,000
Income from merchandising - P3,500,000
Other income (20% represents income from bank deposits abroad) - P250,000
Expenses (15% represents personal expenses) - P2,400,000
Income from treasury bills - P100,000
Additional information:
¼ of business income and deductible business expenses is from outside the Philippines.
If the taxpayer is a non-resident alien not engaged in trade or business in the Philippines, his
taxable income is:
a) P 3,072,500
b) P 4,075,000
c) P 3,172,500
d) P 3,250,000

Compensation income 275,000 x .90 = 247,500


Business income 3,500,000 x .75 = 2,625,000
Deductible expense 2,400,000 x .85 x .75 = 1,530,000
Other income 250,000 x .80 = 200,000

Business income 2,625,000


Add: Other income 200,000
Total income 2,825,000
Add: Compensation income 247,500
Income from treasury bills 100,000
Total taxable income 3,172,500

Question 24
A taxpayer married, with three minor children, provided the following data for 2018:
Compensation income (10% represents SSS, Pag-IBIG, Philhealth & Union dues contributions) -
P275,000
Income from merchandising - P3,500,000
Other income (20% represents income from bank deposits abroad) - P250,000
Expenses (15% represents personal expenses) - P2,400,000
Income from treasury bills - P100,000
Additional information:
¼ of business income and deductible business expenses is from outside the Philippines.
Assuming that the taxpayer is a non-resident citizen, is he qualified to avail of the 8% income tax
rate based on gross sales and/or gross receipts and other non-operating income?
a) Yes, because he is a mixed income earner (compensation and self-employment income).
b) No, because there is no indication at all of his signifying his intention to be taxed at the
8% income tax rate in his 1st quarter of 2018 income tax return.
c) Yes, if he opts to be taxed at the 8% income tax rate duly signified in his 1st quarter income
tax return, due to his actually failing to meet the over P3 Million gross sales and/or gross
receipts threshold as compulsory VAT taxpayer in 2018.
d) No, because his 2018 gross sales and/or gross receipts exceeded the P3 Million threshold
as a compulsory VAT taxpayer, registered or not.

Compensation income 275,000 x .90 = 247,500


Business income 3,500,000 x .75 = 2,625,000
Deductible expense 2,400,000 x .85 x .75 = 1,530,000
Other income 250,000 x .80 = 200,000

Business income 2,625,000


Add: Other income 200,000
Total income 2,825,000
Less: Deductible expense 1,530,000
Net income 1,295,000

Criteria for Availing this Option


According to Revenue Memorandum Order No. 23-2018, aside from gross sales/receipts and non-
taxable income not exceeding the VAT threshold (3 million PHP), here are other criteria that you
should follow:
• The individual taxpayer should either be a self-employed (single proprietor,
professional, or mixed income earner).
• The taxpayer shall be registered and subject to percentage tax (or non-VAT filer).
• Taxpayer should have expressed his/her intention of availing the 8% Income Tax Rate.

While those who are not qualified are the following:


• Corporation owning a business
• Compensation income earners
• VAT registered taxpayers (depending on the amount of gross sales/receipts)
• Taxpayers exempted from VAT, but exceeded the 3 million peso VAT threshold
• Taxpayers subject to other forms of Income Taxes
• Partners of GPP (General Professional Partnership).

Question 25
Mr. ILUSTRE, a Financial Comptroller of an Automotive firm, earned annual compensation income
in 2019 of P1,500,000, inclusive of 13th month pay and other benefits in the amount of P120,000
but net of mandatory contributions to SSS, Pag-IBIG and Philhealth. Aside from employment
income, he owns a medium-sized grocery store with gross sales of P2,400,000. His cost of sales
and operating expenses are P1,000,000 and P600,000, respectively, and with non-operating
income of P100,000.
Which is the better option to choose from: (a) the special income tax rate of eight percent (8%);
or (b) the graduated income tax rates of 0% - 35% under Sec. 24 (A)(2)(a) of the Tax Code, as
amended?

a) Progressive rates under Sec. 24 (A)(2)(a).


b) 8% income tax rate based on taxable income.
c) It’s always the 8% income tax rate because of the twin benefits of not only avoiding the
graduated income tax rates under Sec. 24 (A)(2)(a) but also the exemption from the 3%
OPT under Sec. 116, both in the NIRC, as amended.
d) Indifferent, as the two options will result in the same total national internal revenue taxes
liability anyway of the taxpayer.

Using the 8% income tax rate, the total tax due from income tax in lieu of graduated income tax
and percentage income tax is still lower than the total income tax due when using the progressive
tax rates and total percentage tax due.

Question 26
Mr. ILUSTRE, a Financial Comptroller of an Automotive firm, earned annual compensation income
in 2019 of P1,500,000, inclusive of 13th month pay and other benefits in the amount of P120,000
but net of mandatory contributions to SSS, Pag-IBIG and Philhealth. Aside from employment
income, he owns a medium-sized grocery store with gross sales of P2,400,000. His cost of sales
and operating expenses are P1,000,000 and P600,000, respectively, and with non-operating
income of P100,000.
What if the taxpayer chose to be taxed the progressive income tax rates of 0% - 35% under Sec.
24(A)(2)(a) of the Tax Code, as amended, the total income tax and business tax due and payable
for 2019 is:
a) P 877,200
b) P 589,200
c) P 664,200
d) P 661,200

Compensation income 1,500,000 – 90,000 = 1,410,000


Business income 2,400,000 + 100,000 = 2,500,000 – 1,000,000 – 600,000 = 900,000

Total tax due 1,410,000 + 900,000 = 2,310,000 – 2,000,000 = 310,000 x 32% =


99,200 + 490,000 = 589,200

Question 27
Ferdinand is a sole proprietor in trading business, with his place of business in a building where
he uses the first floor as his store and the second floor as his residence. In 2019, he made the
following payments of taxes:
• Registration for 2019 of the business with BIR - P500
• Income tax for 2018 - P200,000
• Value-added tax (including interest of P20,000 and surcharge of P 60,000) - P450,000
• Fringe benefit tax to his Manager - P35,000
• Community Tax - P2,005
• Real estate tax on his land and building - P8,000
• City business taxes and Mayor’s permit - P10,000
• Motor vehicle registration fee to the LTO (car is for non-business use), including
Interest of P 500 and surcharge for penalty of P 2,000 - P4,500

The deduction for taxes expense in 2019 income tax return is:
a) P 55,505
b) P 51,505
c) P 53,505
d) P 71,505

Registration 500
Fringe benefit tax 35,000
Community Tax 2,005
Real estate tax 4,000 (8,000 ÷ 2)
Mayor’s permit 10,000
Total deduction 51,505

Question 28
Which is not correct? The net operating loss carry-over (NOLCO) is:
a. Available to an individual in business in the Philippines.
b. Available to a domestic corporation.
c. Not available to a general professional partnership in the Philippines.
d. Available to a registered general partnership in business in the Philippines.

NOLCO refers to an operating loss from business or exercise of a profession. For individuals who
are mixed income earners, NOLCO is measured by separating compensation income from business
or professional income following the income classification and globalization rule.

Question 29
Jerry received the following income in 2018:

• Business income, Philippines - P300,000


• Business income, United States - P250,000
• Expenses, Philippines - P200,000
• Expenses, United States - P125,000
• Interest on deposit with Metrobank - P3,000
• Cash prize won in a local contest - P6,000
• Cash prize won in a contest in U.S. - P10,000
• Winnings in lotto - P20,000
• Winnings in lotto in U.S. - P50,000
• Dividends from SMC, a domestic company - P25,000
• Interest on deposit in U.S. ($1 = P52) - $500

The taxable income if Jerry is a resident citizen, single, is:


a) P317,000
b) P267,000
c) P88,500
d) P106,000

Business income, Ph 300,000 – 200,000 = 100,000


Business income, USA 250,000 = 125,000 = 125,000
Cash prize local 6,000
Cash prize in US 10,000
Winnings in lotto in US 50,000
Interest on deposit US 26,000_
Total taxable income 317,000

Question 30
A domestic corporation’s computed NCIT and MCIT and creditable income taxes withheld at
source from 1st to 4th quarters, including excess MCIT and excess withholding taxes from prior
year/s, are as follows:

Quarter NCIT MCIT Taxes Withheld Excess Excess


MCIT/NCIT withholding tax
prior year prior year/s

1st P100,000 P80,000 P20,000 P30,000 P10,000

2nd P120,000 P250,000 P30,000

3rd P250,000 P100,000 P40,000


4th P200,000 P100,000 P35,000

The income tax due at the end of the first quarter is:
a) P40,000
b) P70,000
c) P80,000
d) P50,000

NCIT 100,000
Less: Taxes withheld 20,000
Excess MCIT/NCIT prior year 30,000
Excess withholding tax prior year 10,000
Income tax due 40,000

Question 31
MICHAEL, single, had the following during the taxable year 2018:

Philippines USA

Gross Income from Business P800,000 P400,000

Business Expenses P200,000

P500,000

Rental Income, Gross P200,000 P100,000

Interest Income on Bank Deposit P100,000 P20,000

Royalty Income P60,000 P40,000

Dividend Received P80,000 P30,000


If MICHAEL is a non-resident citizen, his taxable income is:
a) P870,000
b) P500,000
c) P840,000
d) P450,000

Gross income from business, Ph 800,000


Business expenses, Ph (500,000)
Gross profit 300,000
Rental income, Ph 200,000
Taxable income 500,000

Question 32
A domestic corporation, in its fourth year of operations, had:
Gross profit from sales - P 3,000,000
Expenses of operations - P1,000,000
Quarterly income taxes paid - P500,000
Dividend from domestic corporation - P200,000
Interest on bank deposit - P100,000
Gain on sale of capital asset:
Land and building outside the Philippines, on a selling price of P15,000,000 and cost of
P10,000,000 - P5,000,000
Shares of stock of a domestic corporation, not listed and traded in a local stock exchange, on a
selling price of P350,000 and cost of P100,000 - P250,000
What is the income tax still due for the year?
a) P100,000
b) P200,000 3m - 1m = 2m x 30% = 600k - 500k = 100k
c) None of the Above.
d) P150,000

Question 33
A corporation has the following income and expenses for 2018 (fifth year of operations):
Gross income, Philippines - P2,400,000
Gross income, U.S. - P1,600,000
Business expense, Philippines - P1,000,000
Business expense, U.S. - P800,000
Unallocated business expense - P480,000
Interest expense in connection with U.S. business - P100,000
Interest expense in connection with Philippine business - P200,000
Interest expense in connection with business in the Philippines and U.S. - P160,000
The corporate income tax due if a resident foreign corporation:
a) P 216,120
b) P 244,800
c) P 285,600
d) P 48,000

Question 34
SNOWDROP Corporation made the following payments for the 3rd quarter of 2018 in behalf of
its Executive Vice President:
1) To SM Supermarket, groceries consumed by the officer’s family - P97,500
2) To Saint Louis University, tuition of EVP’s eldest son (one of the company scholars chosen after
passing the company’s qualifying examination) - P30,000
3) Brand new car leased from Rent-A-Car for use of the officer (quarterly rental) - P32,500
The fringe benefit tax for the quarter is:
a) P67,012
b) P61,250
c) P57,600
d) P55,168
Groceries or household expenses 97,500 ÷ 65% = 150,000 x 35% = 52,500
Brand new car leased 32,500 ÷ 2 = 16,250 ÷ 65% = 25,000 x 35% = 8,750
Fringe benefit tax 52,500 + 8,750 = 61,250

Question 35
Glenn is a dealer of household furniture regularly selling on installments. It made a sale of a
bedroom set, with a cost of P4,000, for P10,000, receiving as downpayment P1,000 on the date
of sale on December 15, 2018, and P1,500 the day after, and monthly thereafter at P2,500.
The gross income to report on the installment method in 2019 is:
a) P 1,500
b) P 4,500
c) P 7,500
d) Zero

Gross profit x (collection ÷ contract price)

Question 36
Francisco is a single proprietor in trading business, with his place of business in a building where
he uses the first floor as his store and the second floor as his residence. In 2018, he made the
following payments of taxes:
1) Registration fee for 2018 of the business with the Bureau of Internal Revenue - P 500
2) Income tax of 2017 - P200,000
3) Value-added taxes (including interest of P20,000 and surcharges of P60,000) - P420,000
4) Fringe benefit tax to his manager - P32,000
5) Community tax - P2,005
6) Real estate tax on his land and building - P7,000
7) City business taxes and city mayor’s permit - P5,500
8) Motor vehicle registration fee (car is for non-business use) Including interest of P50 and
surcharge for penalty of P1,500 - P3,050

The deduction for taxes is:


a) P 638,055
b) P 43,505
c) P 11,005
d) P 211,505

Registration 500
Fringe benefit tax 32,000
Community Tax 2,005
Real estate tax 3,500 (7,000 ÷ 2)
Mayor’s permit 5,500
Total deduction 43,505

Question 37
Mr. Garcia, a cemetery lot dealer sold real properties to different buyers as follows:

Selling Price Cost


House & Lot P2,000,000 P1,250,000
Farm Lot P800,000 P300,000
Cemetery Lot P45,000 P20,000

The house and lot was Mr. Garcia’ principal residence that was sold to reacquire a new principal
residence noting that the entire proceeds thereof was utilized.
What is Mr. Garcia’ capital gains tax, assuming that all the selling price are at fair market value?
a) P168,000
b) P 48,000
c) P170,700
d) P 50,700
Farm Lot 800,000 x 6% = 48,000
Question 38
In 2020, DEL Corp. had the following items of income and expenses:
Sales - P500,000
Cost of services, salary of personnel directly engaged in business - P250,000
Dividends received - P25,000
The dividends were received from a domestic corporation. The general and administrative
expenses include factory facilities expense and cost of manufacturing supplies of P25,000 and
P15,000, respectively. What amount should be reported as gross income for minimum corporate
income tax?
a) P210,000
b) P250,000
c) P275,000
d) P235,000

Sales 500,000
Less: Cost of services (including factory exp and mftg supplies) 290,000
Gross income from operation 210,000

Question 39
Bench Corp., a calendar-year corporation realized taxable income of P36,000 from its regular
business operations for calendar-year 2020. In addition, Bench had the following capital gains
and losses during 2020.
Short-term capital gain - P8,500
Short-term capital loss - P(4,000)
Long-term capital gain - P1,500
Long-term capital loss - P(3,500)
Bench did not realize any other capital gains or losses since it began operations.
What is Bench’ total taxable income for 2020?
a) P36,250
b) P40,500
c) P38,500
d) P39,500

Total taxable income = 36,000 + 8,500 – 4,000 + 1,500 – 3,500 = 38,500

The holding period rule does not apply to corporations. Regardless of the length of the holding
period, 100% of the capital gain or capital loss is recognized for corporate taxpayers.

Question 40
The Congress of the Philippines enacted a law imposing 5% tax on a specific transaction. The tax
law may be considered valid because:
a) It is for public purpose
b) It is generally payable in money
c) It is equitable
d) It is for unity
“Specific transaction”

Equal protection of the law – means equality before the law. However, such equality does not
deny to the state the power to recognize factual differences between individuals and classes. It
recognizes that inherent in the right to legislate is the right to classify, provided that it is a valid
and reasonable classification. If the groupings are characterized by substantial distinctions that
make real differences, one class may be treated and regulated differently from another.

Question 41
The assessment of BIR against Nicanor became final and executory. The BIR conducted summary
proceedings to collect the delinquent taxes. The BIR determined that Nicanor had real properties
hence
the same were levied. The BIR’s action is in consonance with:
a) Power of Eminent Domain
b) Power of Taxation
c) Police Power
d) Power of Unity

Question 42
Which is FALSE in the application of the principle of Fiscal Adequacy in case of deficit?
I. The government may lend money
II. The government may impose new taxes
III. The government may sell state properties
IV. The government may suspend capital expenditures
a) I
b) II
c) IV
d) III

Fiscal adequacy means the sources of revenue must be sufficient to meet government
expenditures and other public needs. In case of deficit, the government may no longer lend money
as it will become detrimental to its funds. The other options are ways of obtaining revenue or
sources of revenue.

Question 43
During audit, the BIR discovered that ABC Corp. does not keep books of accounts. ABC Corp. in
its SEC and BIR registration, it indicated that its accounting period is fiscal year. Which is TRUE?
a) The BIR needs to follow taxpayer’s registered taxable year
b) The BIR can issue an assessment using the calendar year
c) The BIR, in issuing an assessment, can only consider the fiscal year period being applied
by ABC Corp.
d) The BIR can compel the taxpayer to change its taxable period
Under the NIRC, the calendar year shall be used when the:
• Taxpayer’s annual accounting period is other than a fiscal year (longer than 12 months)
• Taxpayer has no annual accounting period (less than 12 months)
• Taxpayer does not keep books
• Taxpayer is an individual

Question 44
The BIR is under the direct supervision of:
a) The Department of Finance
b) The Department of Trade and Industry
c) The Department of Interior and Local Government
d) The Office of the President

Question 45
The following are powers of the Commissioner, except:
I. To change venue of filing of returns
II. To extend period of filing of returns
III. To compel taxpayers to appear for its failure to provide requested documents
IV. To decide protested assessments
a) I
b) IV
c) II
d) III

The NIRC did not mention a Commissioner’s power to extend period of filing of returns.

Question 46
S1: Inday, a model, is a BIR-registered professional. In 2021, she ventured into vlogging where
she earned Php 1,000,000 on top of her modelling income of Php 1,000,000.00. In 2021, Inday is
classified as a Mixed Income Earner.
S2: Nicanor is unemployed. He tried moto-vlogging as a hobby. He was able to monetize Php
1,000 based on the views he was able to generate on his YuTube Channel. Nicanor is required to
register as a professional, issue official receipts and keep books of accounts.
a) Only S2 is true
b) Both are True
c) Only S1 is true
d) Both are False

Both income of Inday are considered income from business/profession and not compensation
income, thus she is not a mixed income earner. Nicanor’s income from youtube is also income
from business/profession but not compensation income.

Question 47
ABC Corp. a PAGCOR-licensee to operate offline gaming in the Philippines employs aliens. The
following are the terms of employment of its Chinese employees:

Name Position Period of Employment Amount of Salary

Wan President 1 Year P1,000,000

Ton Consultant 6 Months P500,000

Sio Dealer 3 Months P200,000

Mai Interpreter 1 Month P100,000

Which is TRUE?
a) Ton is subject to expanded withholding tax
b) Wan is subject to withholding tax on compensation
c) Sio and Mai are exempted from taxation because their income is below Php 250,000
d) All are subject to 25% final tax

Question 48
Which is FALSE?
I. Nicanor, an employee, is not qualified to use 8% preferential taxation.
II. Nicanor, a sari-sari store owner, is allowed to use commuted tax of 8%.
III. Nicanor, a social media influencer and apologist was paid Php 5,000,000. He has the option to
be
taxed at 8% or graduated tax rate.
IV. Nicanor, a VAT-registered taxpayer, whose gross sales is only Php 1,000,000.00 is not qualified
to
use 8% rate
a) III and IV
b) III
c) IV
d) II, III and IV

The gross income of 5,000,000 exceeded the threshold of 3,000,000 to qualify for 8% income tax
option.

Question 49
Which is FALSE?
I. Nicanor, a sari-sari store owner, opted commuted tax of 8%. He is allowed to claim deduction
of Php 250,000.00 on his gross sales.
II. Nicanor, a social media influencer and apologist was paid Php 5,000,000. He is allowed to claim
deduction of Php 250,000.00 on his gross receipts.
III. Nicanor, a VAT-registered taxpayer, whose gross sales is only Php 1,000,000.00 is allowed to
claim deduction of Php 250,000.00 on his gross sales.
a) II
b) I, II and III
c) II and III
d) III

Gross income + non-operating income – 250,000 = Taxable income x 8% = Tax due


II is false because his income exceeded the VAT threshold of 3M.
III is false because he is a VAT-registered taxpayer.

Criteria for Availing this Option


According to Revenue Memorandum Order No. 23-2018, aside from gross sales/receipts and non-
taxable income not exceeding the VAT threshold (3 million PHP), here are other criteria that you
should follow:
• The individual taxpayer should either be a self-employed (single proprietor,
professional, or mixed income earner).
• The taxpayer shall be registered and subject to percentage tax (or non-VAT filer).
• Taxpayer should have expressed his/her intention of availing the 8% Income Tax Rate.

While those who are not qualified are the following:


• Corporation owning a business
• Compensation income earners
• VAT registered taxpayers (depending on the amount of gross sales/receipts)
• Taxpayers exempted from VAT, but exceeded the 3 million peso VAT threshold
• Taxpayers subject to other forms of Income Taxes
• Partners of GPP (General Professional Partnership).

Question 50
Which is FALSE? Assume all incomes are derived from within the Philippines
I. Nicanor, a non-resident citizen who is engaged in trade or business has option to use 8% or
graduated tax rate
II. Nick Aynor, a resident alien who is in the practice of his profession can opt to use 8% taxation
III. Nica Knorr, a non-resident alien who is engaged in trade or business can use either graduated
or 8% taxation
IV. Nick Nore, a non-resident alien who is not engaged in trade or business can use graduated tax
rate
V. Estate of Nicanor can only use graduate tax
a) IV and V
b) III and IV
c) II, III, IV and V
d) IV

Question 51
S1 – Nicanor, an NRC based in Canada opened a bank account in ABC Bank, a DC. The source of
the interest income of Nicanor is derived from within the Philippines.
S2 – Nicanor, an RA, opened a bank account in ABC Bank, a DC. The source of the interest income
of Nicanor is derived from within the Philippines.
S3 – Nicanor, an NRAETB opened a bank account in ABC Bank, a DC. The source of the interest
income of Nicanor is derived from within the Philippines.
S4 – Nicanor, an NRANETB opened a bank account in ABC Bank, a DC. The source of the interest
income of Nicanor is derived from within the Philippines.
Which is FALSE?
a) S1
b) None.
c) S3
d) S4
e) S2

Question 52
S1 – Nicanor, an NRC based in Canada opened a bank account in ABC Bank, a NRFC. The income
is subject
to 20% final tax.
S2 – Nicanor, an RA, opened a bank account in ABC Bank, a DC. The withholding agent is the bank.
S3 – Nicanor, an NRAETB opened a bank account in ABC Bank, a DC. The interest income is not
required
to be reported by Nicanor.
S4 – Nicanor, an NRANETB opened a bank account in ABC Bank, an NRFC. The interest income is
exempt
from taxation
Which is FALSE?
a) S2
b) S4
c) S3
d) S1

Territoriality principle, thus NRC is subject to final tax only on income within the Philippines.

Question 53
Which is TRUE? ABC Corp declared dividends.
S1- If ABC Corp is DC the source of the income is derived from within the Philippines
S2 – If ABC Corp is RFC the source of the income is derived from within the Philippines if the
dominant income last year is Philippine income
S3 – If ABC Corp is NRFC the source of the income is derived from within the Philippines if the
dominant income last year is Philippine income
S4 – If ABC Corp is a foreign corporation, the source of the income is derived from without the
Philippines
a) S1
b) All are True.
c) S1, S2 & S3
d) S4
e) S1 & S4

Question 54
Which is FALSE in withholding tax system?
S1 – All passive income derived from within the Philippines are subject to final withholding tax
S2 – Income payments are generally subject to withholding tax
S3 – If an income is exempted from taxation, it is not subject to withholding
S4 – A withholding agent is required to withhold and remit taxes on all its income payments
a) S1 and S4
b) S2 and S3
c) S1, S2 and S4
d) S4
e) S1

Not all passive income derived from within the Philippines are subject to final tax, only certain
passive income. Likewise, not all the income payments of a withholding agent is required to
withhold and remit taxes.

Question 55
S1: ABC Corporation was identified as a top withholding agent by the BIR. In case the company
purchases raw material goods for its production, the company is required to withhold 1%
expanded withholding tax for its purchases.

S2: ABC Corporation was identified as a top withholding agent by the BIR. In case the company
purchases raw material goods for its production, the company is required to withhold 2%
expanded withholding tax.
a) Only S2 is true
b) Both are True.
c) Both are False.
d) Only S1 is true

Goods – 1% expanded withholding tax


Services – 2% expanded withholding tax
Question 56
S1: ABC Corporation was identified as a top withholding agent by the BIR. In case the company
engages the services of XYZ Corporation, a security agency company, to provide security guards
to the former, ABC Corporation is required to withhold 1% expanded withholding tax for the
amount billed by XYZ Corporation (excluding the salaries of the guards).

S2: ABC Corporation was identified as a top withholding agent by the BIR. In case the company
engages the services of XYZ Corporation, a security agency company, to provide security guards
to the former, ABC Corporation is required to withhold 2% expanded withholding tax for the
amount billed by XYZ Corporation
(excluding the salaries of the guards).
a) Only S2 is true
b) Both are True.
c) Both are False.
d) Only S1 is true

Goods – 1% expanded withholding tax


Services – 2% expanded withholding tax

Question 57
ABC Corporation was identified as a top withholding agent by the BIR. One day, the accounting
department, headed by Nicanor, requested the company to treat all of its staff a Jollibee meal
amounting to Php 9,000 because it is already tax season, and the employees are working a
minimum of 18-hours a day. The company granted the request. Which is FALSE?

a) The transaction is not subject to withholding tax on compensation even if it is for


employee benefits.
b) The transaction is not subject to expanded withholding tax but subject to withholding tax
on compensation.
c) The transaction is not subject to final withholding tax.
d) The transaction is exempt from expanded withholding tax.

The transaction is not subject to final withholding tax, expanded withholding tax, and withholding
tax on compensation even if it is for employee benefits.

Question 58
ABC Corporation was identified as a top withholding agent by the BIR. One day, the accounting
department, headed by Nicanor, requested the company to treat all of its staff a Jalibee meal
amounting to P 1,000 because it is already tax season, and the employees are working 18-hours
a day. The company granted the request. Within the taxable year, the company transacted with
Jalibee five times for buying Jollibee meals with a total amount of Php 7,000. As a result of
purchases during the taxable year, the company is:
a) Required to provide tax credit certificate to Jalibee
b) Not required to withhold expanded withholding tax
c) Required to withhold expanded withholding tax of 1%
d) Required to withhold expanded withholding tax of 2%

The transaction is not subject to final withholding tax, expanded withholding tax, and withholding
tax on compensation even if it is for employee benefits.

Question 59
ABC College, a non-stock, non-profit educational institution, operates a canteen, bookstore, and
dormitory inside its campus which are commercial in nature. The income from the sale of goods
and/or services by the canteen, bookstore, and dormitory inside the campus are actually,
directly, and exclusively used for educational purposes. Using the data provided, which is FALSE?
S1 - Income from related activity is exempt from taxation
S2 - Income from unrelated activity is subject to regular tax
S3 - Income from unrelated activity is exempt from income tax
a) All are True.
b) S1
c) S2
d) S3

Unrelated activity here might include the income from canteen, bookstore, and dormitory which
is not subject to regular tax, thus exempt from income tax.

Question 60
ABC Corp is required to file income tax returns and the following are the deadlines assuming it
applies calendar year basis, except:
S1 – 1st Quarter – May 31
S2 – 2nd Quarter – August 29
S3 – 3rd Quarter – November 29
S4 – Annual – April 15
a) S1
b) S2
c) S4
d) S3

S1 exceeded the 60 day threshold as it was already 61 days on May 31.

Question 61
Nicanor is filing his annual income tax return. His computed income tax due is Php 5,000 and he
has tax credits amounting to Php 4,000. Which is TRUE?
a) He will be subjected to penalties if he pays on April 15
b) He can pay the balance in installment, Php 500 upon filing and Php 500 before October
15
c) He is not allowed to pay on installment because the payable is below Php 1,000
d) He can pay nothing during the annual filing and pay the balance of Php 1,000 on or before
August 15

When the tax due is in excess of 2,000 (tax due is 5,000 which is in excess of 2,000), individual
taxpayers (except corporations) may elect to pay the tax in two equal installments.
a. The first installment shall be paid at the time the return is filed.
b. The second installment is due on or before October 15 following the close of the
calendar year.

Question 62
In the deadline of filing of 2020 Annual Income Tax Returns, which is FALSE?
S1 – ABC Corp. employing fiscal year ending January 31, the deadline is May 15 of the same year
S2 – ABC Corp. employing fiscal year ending June 30, the deadline is October 15 of the same year
S3 – ABC Corp. employing fiscal year ending October 31, the deadline is February 15 of the same
year
S4 – ABC Corp. employing calendar, the deadline is April 15 of the following year
a) S2
b) S3
c) S4
d) S1

It is not February 15 of the same year. It should be February 15 next year.

Question 63
In capital gains tax, which is FALSE?
S1 - The seller is the person liable to pay the capital gains tax if there is no agreement
S2 - The buyer is the one liable if there’s an agreement but the tax so paid is part of the
consideration
a) Both are True
b) Both are False
c) Only S1 is True
d) Only S2 is True

Capital gains tax is a final tax in which the capital gains tax shall be withheld by the buyer against
the selling price of the seller and remit the same to the government. If there is no agreement, the
seller will be the one liable to pay the capital gains tax,

Question 64
Which is TRUE in fringe benefits tax?
S1 - Supervisory employees are those who, in the interest of the employer, effectively
recommend such
managerial actions or the exercise of such authority is not merely routinary or clerical in nature
but requires the
use of independent judgment
S2 - Managerial employees are ones who are vested with powers or prerogatives to lay down and
execute
management policies and/or hire, suspend, lay-off, recall, discharge, assign or discipline
employees
S3 – Rank-and-File employees are employees who are holding neither managerial nor supervisory
position
a) All of the above
b) S3
c) S1
d) S2

Definition of type of employees.

Question 65
Which is FALSE in fringe benefits tax?
S1 – Monetary value of fringe benefit less fringe benefits is equal to fringe benefits tax.
S2 – Grossed-up monetary value of fringe benefits less of monetary value of fringe benefits is
equal to fringe
benefits tax.
S3 – Fringe benefits tax plus monetary value of fringe benefits is equal to grossed-up monetary
value of the fringe
benefits.

a) S1
b) S2 and S3 are True.
c) S2
d) S3

Fringe benefits tax = (Monetary value ÷ gross-up rate) x fringe benefit tax rate

Question 66
Which is TRUE?
S1 – All incomes are generally taxable.
S2 – All passive incomes are generally subject to final tax.
a) S1 is true
b) Both are False
c) Both are True
d) S2 is true

All income where legal or illegal are generally taxable but not all passive income are generally
subject to final tax, only certain passive income.

Question 67
Which is FALSE?
S1 –Nicanor is a minimum wage earner. His total compensation including overtime pay, night
shift differential and hazard pay amounted to Php 300,000. He is exempt from income tax.
S2 – Nicanor’s daily wage amounted to Php 600.00 per day which is beyond the statutory
minimum wage of Php 537. His salary is subject to withholding tax on compensation.
a) S1
b) Both are False
c) Both are True
d) S2

Question 68
Who is required to file Annual Income Tax Return (AITR)?

No. of Employer Income Tax Due Tax Withheld by Employer

Nicanor 1 P10,000 P11,000


Jose 2 (minimum wage earner) Zero Zero

Andres 1 P10,000 P9,000


Goyo 2 P10,000 P10,000
a) All Except Jose
b) Nicanor and Andres
c) All of them are required
d) Andres and Jose

All except Jose are required to file annual income tax return since they did not meet the
qualification for substituted filing system. Jose, although a minimum wage earner is still required
to file income tax but exempt from income tax.

Question 69
A non-resident alien not engaged in trade or business (NRANETB) shall generally be subject to a
25% final tax on gross income received from all sources within the Philippines. However, a
NRANETB shall be exempt from tax on:
a) Philippine Charity and Sweepstakes Office (PCSO) winnings less than ₱10,000.
b) Interest income paid by a depositary bank under the foreign currency deposit system.
c) Interest income from a long-term deposit or investment certificate issued by a bank in
the Philippines, and held for a period exceeding 5 years.
d) De minimis prizes of less than ₱10,000.

Territoriality principle, thus NRANETB is subject to tax only on income within the Philippines.

Question 70
The following are the features of the 8% optional income tax in the ITR, except:
a) The qualified individual taxpayer is not required to file his financial statements with his
Annual ITR.
b) The qualified taxpayer, who is also a partner in a general professional partnership, must
signify his choice of the 8% tax rate in the first quarterly ITR.
c) The qualified taxpayer is exempt from paying the 3% OPT under Section 116 of the Tax
Code.
d) The qualified taxpayer is still required to comply with bookkeeping and invoicing
requirements.

A self-employed individual who is qualified and who availed of the 8% income tax rate option is:
a. Required to file the Quarterly Income Tax Return, unless exempted by any revenue
issuances
b. Required to file the Annual Income Tax Return [Financial Statement (FS) is not required
to be attached]
c. Not required to file the Quarterly Percentage Tax Return
d. Required to signify the intention to avail of the 8% income tax rate every taxable year
e. Required to maintain books of accounts and issue receipts/invoices
RESA
FIRST
PREBOARD
ReSA - THE REVIEW SCHOOL OF ACCOUNTANCY
CPA Review Batch 43  May 2022 CPALE  9 Feb 2022  6:00 PM – 9:00 PM

TAXATION FIRST PRE-BOARD EXAMINATION

INSTRUCTIONS: Select the correct answer for each of the questions. Mark only one
answer for each item by shading the box corresponding to the letter of your choice on
the answer sheet provided. STRICTLY NO ERASURES ALLOWED. Use pencil no. 2 only.

1. Which of the following is not a nonresident citizen?


D a. A citizen of the Philippines who establishes to the satisfaction of
the Commissioner the fact of his physical presence abroad with a
definite intention to reside therein.
b. A citizen of the Philippines who leaves the Philippines during the
taxable year to reside abroad, either as an immigrant or for
employment on a permanent basis.
c. A citizen of the Philippines who works and derives income from sources
abroad and whose employment thereat requires him to be physically
present abroad most of the time during the taxable year.
d. A citizen of the Philippines who goes on a business trip abroad and
stays therein most of the time during the year.

2. First statement: The husband and wife shall compute their individual income tax
separately based on their respective total taxable income.
Second statement: If any income cannot be definitely attributed to or identified
as income exclusively earned or realized by either of the spouses, the same shall
be divided equally between the spouses for the purpose of determining their
respective taxable income.
A a. Both statements are correct
b. Both statements are incorrect
c. Both statements are incorrect
d. Only the second statement is correct

3. Which of the following is not subject to tax as a corporation?


D a. Business partnerships
b. Joint stock companies
c. Insurance companies
d. General professional partnership

4. First statement: All corporations, whether domestic or foreign, shall be taxed


at 20% effectively July, 2020 if their net tax taxable income does not exceed
P5,000,000 and their total assets do not exceed P100,000,000.
Second statement: Effective January 1, 2021, non-resident foreign corporations
shall be taxed at 25% on their net income from sources within the Philippines.
B a. Both statements are correct
b. Both statements are incorrect
c. Only the first statement is correct
d. Only the second statement is correct

5. One of the following is not a requisite of a taxable income.


D a. There must be gain.
b. The gain must be realized or received.
c. The gain must not be excluded by law from taxation.
d. The gain must be that of resident or nonresident citizen.

6. A property was received as donation from Charlwin when its fair market value was
P300,000. Charlwin in turn received this property as donation from Jose when
its fair market value was P350,000. This property was purchased by Marceliano
for P200,000 and was donated to Jose. The property was sold for P500,000.

How much gain (loss) shall be recognized from the sale?


A a. P300,000
b. P200,000
c. P150,000
d. None of the choices

7. If an individual performs services for a creditor who in consideration thereof


cancels the debt, the cancellation of indebtedness may amount to a:
D a. gift.
b. capital contribution.
c. donation inter vivos.
d. payment of income.

Page 1 of 14 0915-2303213  [email protected]


TAXATION
ReSA Batch 43 - May 2022 CPALE Batch
09 Feb 2022  6:00 PM to 9:00 PM TAX First Pre-Board Exam
8. It is important to know the source of income for tax purposes (i.e., from within
and without the Philippines) because:
A a. some individuals and corporate taxpayers are taxed on their
worldwide income while others are taxable only upon income from
sources within the Philippines.
b. the Philippines imposes income tax only on income from sources
within.
c. some individual taxpayers are citizens while others are aliens.
d. export sales are not subject to income tax.

9. Gains, profits and income from the sale of real property are from sources
within the Philippines if:
A a. the real property sold is located in the Philippines.
b. the real property is sold in the Philippines.
c. the real property sold is located outside the Philippines.
d. the real property sold is owned by a resident citizen.

10. The term “capital asset” includes:


B a. stock in trade or other property included in the taxpayer’s inventory.
b. real property not used in the trade or business of taxpayer.
c. property primarily for sale to customers in the ordinary course of
his trade or business.
d. property used in the trade or business of the taxpayer and subject
to depreciation.

11. The widow of your best friend has just been paid P1,000,000 on account of the
life insurance policy of the deceased husband. She asks you whether she shall
declare the amount for income tax purposes or for estate tax purposes.
First advice: The proceeds of the life insurance paid to the beneficiary upon
the death of the insured are exempt from income tax and need not be declared for
income tax purposes.
Second advice: The proceeds of life insurance will have to be declared for estate
tax purposes if the designation of the beneficiary is revocable, otherwise, they
need not be declared.
A a. Both advices are right
b. First advice right; second advice wrong
c. Both advices are wrong
d. First advice wrong; second advice right

12. Ms. Elisya Montenegro entered her short story in a literary contest. She won in
the Short Story category, and received P500,000 for her prize. What was the tax
consequence of the literary prize?
B a. Exempt from income tax
b. Subject to final withholding tax
c. Subject to Section 24 (A)
d. Not subject to any internal revenue tax

13. Under this system, the amount of income tax withheld by the withholding agent is
constituted as a full and final payment of the income tax due from the payee on
the said income.
B a. Creditable withholding tax
b. Final withholding tax
c. Global tax system
d. Schedular tax system

14. Which of the following passive income is not subject to 20% final withholding
tax. As a general rule, which of the following passive income of resident citizen?
D a. Interest from any peso bank deposit
b. Yield or any other monetary benefit from deposit substitutes and
from trust funds and similar arrangements
c. Prizes (except prizes amounting to P10,000 or less)
d. Royalties on books, as well as other literary works and musical
compositions

Page 2 of 14 0915-2303213  [email protected]


TAXATION
ReSA Batch 43 - May 2022 CPALE Batch
09 Feb 2022  6:00 PM to 9:00 PM TAX First Pre-Board Exam
15. Which of the following fringe benefits shall be subject to the fringe benefit
tax?
D a. Benefits given to the rank-and-file employees, whether granted under
a collective bargaining agreement or not
b. Fringe benefits required by the nature of, or necessary to the trade,
business or profession of the employer
c. Fringe benefit given for the convenience or advantage of the employer
d. Membership fees, dues and other expenses borne by the employer for
the managerial and supervisory employee in social and athletic clubs
and similar organizations

16. First statement: The fringe benefit tax is imposed only when the employer is a
corporation.

Second statement: The fringe benefit tax shall be withheld and paid by the
employer in accordance with the provision of Section 57 (A).
D a. Both statements are correct
b. Both statements are incorrect
c. Only the first statement is correct
d. Only the second statement is correct

17. De minimis benefit pertaining to monetized value of vacation and sick leave
credits paid to government officials and employees is limited to:
D a. 10 days
b. 15 days
c. 20 days
d. None of choices

18. The gross estate of this decedent shall only be comprised of properties
situated in the Philippines.
D a. Filipino residing in the Philippines
b. American residing in the Philippines
c. Filipino residing in the USA
d. American residing in the USA

19. Mr. Julian Cruz procured a life insurance upon his own life. He designated his
estate’s executor as an irrevocable beneficiary. For estate tax purposes, the
proceeds of life insurance is:
A a. included in the gross estate of Mr. Julian Cruz because when the
executor of the estate is a beneficiary the proceeds are included in
the gross estate regardless of the designation.
b. not included in the gross estate of Mr. Julian Cruz because the
designation of the beneficiary is irrevocable.
c. included in the gross estate of Mr. Julian Cruz because proceeds of
life insurance are always subject to estate tax.
d. not included in the gross estate because, as a rule, proceeds of life
insurance are generally not subject to estate tax.

20. Mr. Rigoberto Collado, a citizen and resident of Puerto Rico, dies during the
year. Puerto Rico does not impose transfer taxes on properties of decedent not
residing therein. He left the following properties among others:
Shares of stock, San Miguel Corporation, Manila
House and lot, Puerto Rico
Leasehold on a condominium unit, Philippines
Contract for public works, Philippines

The executor of his estate in Philippines asked you what properties are to be
included in his Philippine gross estate. What answer will you give him?
B a. Include all the properties.
b. Include contract for public works and leasehold on condominium unit
only.
c. Include all properties except shares of stock and house and lot
d. Include all properties except house and lot in Puerto Rico.

Page 3 of 14 0915-2303213  [email protected]


TAXATION
ReSA Batch 43 - May 2022 CPALE Batch
09 Feb 2022  6:00 PM to 9:00 PM TAX First Pre-Board Exam
21. A property was transferred mortis causa. The following data were gathered from
the transaction:
Fair market value, time of transfer – P500,000
Fair market value, time of death – P300,000
Consideration received when transferred – P350,000

How much shall be included in the gross estate?


D a. P500,000
b. P300,000
c. P150,000
d. None

22. How much is the allowable standard deduction for non-resident alien decedent?
C a. Five Million Pesos (P5,000,000)
b. Three Million Pesos (P3,000,000)
c. Five Hundred Thousand Pesos (P500,000)
d. None, not allowed to deduct standard deduction

23. The following selected data were taken from the Estate of Ed Sados:
Claim against an insolvent person
(fully uncollectible) P 500,000
Claim against a person who absconded
(fully uncollectible) 300,000
Claim against an insolvent person
(20% collectible) 100,000

How much should be deducted from the gross estate?


B a. P880,000
b. P580,000
c. P100,000
d. P 80,000

24. An unmarried decedent died leaving properties he inherited 4 ½ years ago which
had fair market value of P800,000 at the time of his death (650,000 at the time
of inheritance, and unpaid mortgage of P50,000 paid by the present decedent).
Other properties in his gross estate had fair market value of P1,000,000. The
total expenses, losses indebtedness, taxes and transfer for public purpose
amounted to P300,000.

How much was the vanishing deduction?


D a. P 500,000
b. P 225,000
c. P 200,000
d. P 100,000

25. For purposes of availing of a family home deduction to the extent allowable, a
person may constitute:
B a. as many family homes as possible.
b. only one family home.
c. one family home for each spouse.
d. one family for each child.

26. One of the following is not a distinction between donation inter vivos and
donation mortis causa.
C a. Donation inter vivos takes effect during the lifetime of the grantor
while donation mortis causa takes effect after the death of the
grantor.
b. Donation inter vivos is subject to donor’s tax while donation mortis
causa is subject to estate tax.
c. Donation inter vivos requires a public document while donation mortis
causa may not require a public document.
d. Donation inter vivos is valued at fair market value at the time the
property is given while donation mortis causa is valued at the fair
market value at the time of the death of the grantor.

Page 4 of 14 0915-2303213  [email protected]


TAXATION
ReSA Batch 43 - May 2022 CPALE Batch
09 Feb 2022  6:00 PM to 9:00 PM TAX First Pre-Board Exam
27. Mr. Primitivo Primero died and was survived by his wife and two (2) children,
Faye and Faith. After getting her share in the conjugal property, the surviving
spouse renounced her share in the hereditary estate in favor of Faith to the
exclusion of Faye. Was the renunciation subject to donor’s tax?
A a. Yes, because the renunciation was made categorically in favor of
identified heir to the exclusion or disadvantage of the other co-
heirs.
b. No. because the renunciation was considered a general renunciation.
c. Yes, because , as a rule, renunciation of share in the hereditary
estate is always subject to donor’s tax.
d. No, because, as a rule, the surviving spouse cannot renounce her
share in the hereditary estate.

28. Mr. J. Santos donated P500,000 to AST Foundation, a philanthropic organization.


The organizations total administrative expenses amounted to P5,000,000 out of
its expenses of P20,000,000. How much was the exempt gift to the philanthropic
organization?
C a. P6,000,000
b. P5,000,000
c. P500,000
d. None

29. First statement: For VAT purposes, a taxable person is any person liable to pay
the VAT, whether registered or registrable in accordance with the Tax Code.

Second statement: The status of a “VAT-registered person” as a VAT-registered


person shall continue until the cancellation of such registration.
A a. Both statements are correct
b. Both statements are incorrect
c. Only the first statement is correct
d. Only the second statement is correct

30. Which of the following sales of residential property, shall be subject to VAT
beginning January 1, 2021?
A a. Sale of residential lot the value of which does not exceed P1,500,000
b. Sale of residential house and lot and other residential dwellings
with selling price of not more than P2,000,000.
c. Sale of real properties not primarily held for sale to customers or lease
in the ordinary course of trade or business
d. Sale of real property utilized for socialized housing

31. Which of the following is not included in the term “gross selling price”?
B a. Total amount of money or its equivalent paid by the purchaser
b. Value-added tax passed on by the seller to the buyer
c. Excise tax
d. None of the choices

32. Which statement is incorrect? VAT on importation of goods:


D a. is imposed on an importation for sale or for use in business.
b. is imposed on an importation for personal use.
c. shall be paid prior to removal from customs custody.
d. may not be available as input tax.

33. For VAT purposes, capital goods or properties, also known as depreciable
assets, refer to:
I – goods or properties with estimated useful life greater than one (1)
year
II – treated as depreciable assets under Section 34 (F) of the Tax Code
III – used directly or indirectly in the production or sale of taxable
goods or services
A a. Yes to I, II and III
b. Yes to II and III only
c. Yes to I and II only
d. Yes to I only

Page 5 of 14 0915-2303213  [email protected]


TAXATION
ReSA Batch 43 - May 2022 CPALE Batch
09 Feb 2022  6:00 PM to 9:00 PM TAX First Pre-Board Exam

34. A VAT-registered person shall issue for every sale, barter or exchange of goods
or properties a:
A a. VAT invoice.
b. VAT official receipt.
c. VAT credit certificate.
d. VAT refund.

35. Which of the following shall not be considered livestock for VAT-exempt
transactions purposes?
D a. Cows
b. Bulls and calves
c. Sheep
d. Race horses

36. All of the following cooperatives enjoy VAT exemption on their sales or
receipts except:
D a. agricultural cooperatives.
b. credit and multi-purpose cooperatives.
c. non-agricultural, non-electric and non-credit cooperatives.
d. electric cooperatives.

37. Effective July 1, 2020 to June 30, 2023, the percentage tax rate on persons
exempt from VAT shall be:
A a. 1%.
b. 2%.
c. 3%.
d. 5%.

38. Mr. Jaime Rodriguez is the owner of a small variety store. His gross sales in
any one year do not exceed the VAT threshold amount. He is not VAT-registered.
The following data are taken from the books of the variety store for the quarter
ending March 31, 2020:
Merchandise inventory, December 31, 2019 P 100,000
Gross sales 450,00
Purchases from VAT-registered suppliers 350,000

How much is the percentage tax due and payable?


B a. P22,500
b. P13,500
c. P 9,000
d. P 4,500

39. The following data were provided by an air carrier:


Freight and cargo fees (cargo originating from the
Philippines in a continuous and uninterrupted flight to
Japan, passage documents sold in Japan) P5,000,000
Mail fees (mail originating from Japan in a continuous and
uninterrupted flight to the Philippines, passage documents
sold in the Philippines) 4,000,000
Advance payments for cargo originating from the Philippines
in a continuous and uninterrupted flight to Singapore
(passage documents sold in Singapore) 3,000,000
Receipts from sales of tickets to passengers originating from
the Philippines in a continuous and uninterrupted flight to
Hongkong (tickets sold in the Philippines) 2,000,000
Expenses, Philippines 1,500,000

How much is the common carrier’s tax due from the air carrier assuming it is an
international carrier doing business in the Philippines?
C a. P420,000
b. P300,000
c. P240,000
d. P210,000

Page 6 of 14 0915-2303213  [email protected]


TAXATION
ReSA Batch 43 - May 2022 CPALE Batch
09 Feb 2022  6:00 PM to 9:00 PM TAX First Pre-Board Exam
40. Ms. N is a caterer and a videoke bar operator. In a taxable period, she had the
following data, tax not included:
Sales:
From operations of the Truluv Catering Service:
Cash sales P400,000
Accounts receivable (catering) 250,000
Credit card sales 243,000
From operations of the Dude Videoke Bar:
Cash sales 1,360,000
Credit card sales 624,200
Payments for catering service, based on gross
receipts (80% of which are to VAT taxpayers) 60%

How much is the amusement tax payable?


A a. P357,156
b. P297,630
c. P198,420
d. None of the choices

SITUATIONAL

Ms. Leni operates a convenience store while she offers bookkeeping services to her
clients. In 2018, her gross sales amounted to P800,000.00, in addition to her receipts
from bookkeeping services of P300,000.00. She already signified her intention to be
taxed at 8% income tax rate in her 1st quarter return. How much is the income tax
liability for the yea?

41. Will she qualify to be taxed at 8%?


A a. Yes, because her total gross sales and/or gross receipts and other
non-operating income do not exceed the VAT threshold and she
signifies her intention to be taxed at 8% income tax rate in her
first quarter return.
b. No, because while her total gross sales and/or gross receipts and other
non-operating income do not exceed the VAT threshold and she
signifies her intention to be taxed at 8% income tax rate in her
first quarter return, her income comes from different sources.
c. Yes, because individual taxpayers are always given the option to be
taxed at 8% income tax rate regardless of their total gross sales
and/or gross receipts and other non-operating income.
d. No, because individual taxpayers whose total gross sales and/or
gross receipts and other non-operating income are always subject
to tax under Section 24 (A).

42. How much is her taxable income for the year?


B a. P1,100,000
b. P 850,000
c. P 800,000
d. P 300,000

43. How much is the income tax due?


B a. P88,000
b. P68,000
c. P64,000
d. P10,000

44. Assuming Ms. Leni above, failed to signify her intention to be taxed at 8% income
tax rate on gross sales in her 1st Quarter Income Tax Return, and she incurred
cost of sales and operating expenses amounting to P600,000.00 and P200,000.00,
respectively, or a total of P800,000.00. How much is the income tax?
D a. P88,000
b. P68,000
c. P64,000
d. P10,000

Page 7 of 14 0915-2303213  [email protected]


TAXATION
ReSA Batch 43 - May 2022 CPALE Batch
09 Feb 2022  6:00 PM to 9:00 PM TAX First Pre-Board Exam
45. Using the same information in the immediately preceding number, to what
business tax will she be liable if Ms. Leni failed to signify her intention to
be taxed at 8% income tax rate on gross sales in her 1st Quarter Income Tax
Return?
B a. Value-Added Tax
b. Percentage Tax on Person Exempt From VAT
c. Amusement Tax
d. Excise Tax

SITUATIONAL

A proprietary educational institution has the following data for the calendar year
2021:
Gross receipts, related activities P 15,000,000.00
Cost of services, related activities 6,000,000.00
Allowable deductions from related activities 3,250,000.00
Gross receipts, unrelated activities 18,000,000.00
Cost of services, unrelated activities 5,000,000.00
Allowable deductions from unrelated activities 2,000,000.00
Payments, first three (3) quarters 2,000,000.00

46. How much is net taxable income from related activities?


D a. P16,750,000.00
b. P15,750,000.00
c. P11,000,000.00
d. P 5,750,000.00

47. How much is the net taxable income from unrelated activities?
C a. P16,750,000.00
b. P15,750,000.00
c. P11,000,000.00
d. P 5,750,000.00

48. What corporate tax rate will it be subject to?


A a. 25%
b. 20%
c. 10%
d. 1%

49. How much is the income tax payable?


B a. P4,187,500.00
b. P2,187,500.00
c. P1,675,000.00
d. P1,350,000,00

50. How much is the output VAT, if any?


D a. P3,960,000.00
b. P2,160,000.00
c. P2,160,000.00
d. None, VAT-Exempt

SITUATIONAL

The decedent is a married man with a surviving spouse with the following raw data:
Conjugal real and personal properties (including an automobile
purchased during the marriage using common fund for the
exclusive use of the surviving spouse) P 14,000,000
Conjugal family house 9,000,000
Exclusive family lot 1,000,000
Exclusive properties (including the P3,000,000 exclusive
properties of the surviving spouse) 8,000,000
Conjugal ordinary deductions (including funeral expenses of
P100,000 and judicial expenses of P200,000) 2,300,000
Exclusive ordinary deductions (excluding the P500,000 unpaid
mortgage on the exclusive property of the surviving spouse) 1,000,000
Medical expenses 200,000

Page 8 of 14 0915-2303213  [email protected]


TAXATION
ReSA Batch 43 - May 2022 CPALE Batch
09 Feb 2022  6:00 PM to 9:00 PM TAX First Pre-Board Exam
51. Using BIR Form No. 1801, what value shall be reflected in Schedule 1A (Details
of Family Home)?
A a. P10,000,000
b. P 5,500,000
c. P9,000,000
d. P1,000,000

52. Using BIR Form No. 1801, how much shall be reflected in Line 35 (ordinary
deductions)?
C a. P3,800,000
b. P3,300,000
c. P3,000,000
d. P2,300,000

53. Using BIR Form No. 1801, how much shall be reflected in Line 37D (total special
deductions)?
B a. P15,000,000
b. P10,500,000
c. P10,000,000
d. P 6,000,000

54. Using BIR Form No. 1801, how much shall be reflected in Line 40/16 (net taxable
estate)?
B a. P4,300,000
b. P5,000,000
c. P7,300,000
d. P8,500,000

55. Using BIR Form No. 1801, how much shall be reflected in Line 18 (estate tax due)?
B a. P256,000
b. P300,000
c. P440,000
d. P515,200

SITUATIONAL
Excellent Products, Inc. processes canned fruits and canned sardines. The following
VAT-exclusive selected data for the first quarter of the 2021 calendar year are taken
from its books:
Sale of canned fruits P2,000,000
Sale of canned sardines 3,000,000
Purchase of tomatoes and onions from farmers
(for canned sardines) 500,000
Purchase of corn oil from a supplier
(for canned sardines) 300,000
Purchase of fresh fish from fishermen
(for canned sardines) 400,000
Purchase of fresh fruits from farmers
(for canned fruits) 200,000
Purchase of refined sugar from an agricultural
cooperative that manufactures refined sugar
(for canned fruits) 100,000
Purchase of refined sugar from a refined sugar
refinery (for canned fruits) 50,000
Purchase of packaging materials for the both products 800,000
Purchase of labels (for both products) 150,000
Total bills for trucking services in bringing
canned products to warehouse only P50,000 was paid) 100,000
Monthly VAT payments, previous two months 300,000

56. Using BIR Form No. 2550Q, what shall be reflected in Line 19A (Total
Sales/Receipts)?
A a. P5,000,000
b. P3,000,000
c. P2,000,000
d. None of the choices

Page 9 of 14 0915-2303213  [email protected]


TAXATION
ReSA Batch 43 - May 2022 CPALE Batch
09 Feb 2022  6:00 PM to 9:00 PM TAX First Pre-Board Exam
57. Using BIR Form No. 2550Q, what shall be reflected in Line 19B (output tax due)
using 12% rate?
A a. P600,000
b. P500,000
c. P360,000
d. P240,000

58. Using BIR Form No. 2550Q, what shall be reflected in Line 20D (presumptive
input tax), if any?
C a. P56,000
b. P32,000
c. P20,000
d. None. Not allowed

59. Using BIR Form No. 2550Q, what shall be reflected in Line 24 (total allowable
input tax)?
A a. P182,000
b. P176,000
c. P162,000
d. P156,000

60. Using BIR Form No. 2550Q, what shall be reflected in Line 27 (Tax Still
Payable) using 12% rate?
D a. P418,000
b. P370,000
c. P346,000
d. P118,000

SITUATIONAL

Mr. Nguyen Gandaipen, a non-resident Thai, during the current year donates on January
15, 2022 a brand-new car in the Philippines valued at P1,200,000 and house and lot in
Thailand valued at P2,500,000 to his legitimate son who is getting married in the
Philippines. The son agrees to pay the unpaid tax of P120,000 on the car and the
unpaid mortgage of P500,000 on the house and lot.

61. Using BIR Form No. 1800, what shall be reflected in line 27 (Total Gifts In This
Return)?
C a. P3,700,000
b. P2,500,000
c. P1,200,000
d. None of the choices

62. Using BIR Form No. 1800, what shall be reflected in line 33 (total deductions
allowed)?
B a. P130,000.
b. P120,000.
c. P10,000.
d. none.

63. Using BIR Form No. 1800, what shall be reflected in line 38/14 (total net gift
subject to tax)?
D a. P2,830,000
b. P1,200,000
c. P1,080,000
d. P 830,000

64. Using BIR Form No. 1800, what shall be reflected in line 18 (tax payable)?
D a. P214,200
b. P142,800
c. P64,800
d. P49,800

Page 10 of 14 0915-2303213  [email protected]


TAXATION
ReSA Batch 43 - May 2022 CPALE Batch
09 Feb 2022  6:00 PM to 9:00 PM TAX First Pre-Board Exam
65. Where shall the donor file the donor’s tax return?
C a. Revenue District Office having jurisdiction over the place in the
Philippines where the son is getting married
b. Revenue District Office having the jurisdiction over the place in the
Philippines where the son’s wife resides
c. Philippine Embassy or Philippine Consulate in Thailand
d. None of the choices

SITUATIONAL

The following items (66 – 70) are considered BONUS ITEMS for purposes of determining the score of the
TAX First Pre-Board Exam. The given information for these items were uploaded only around 7:35 PM of
February 9. Some examinees have already submitted the Google Quiz form before 7:35 PM of February 9
while some examinees were able to learn about the uploaded information only AFTER submitting the form.
Ube Paspas Bus is a common carrier by land. It is VAT-registered. During quarter
ending March 31, 2021, it has the following gross receipts:
Transport of passengers P1,000,000
Transport of goods 1,500,000
Transport of cargoes 500,000

66. How much is the common carrier’s tax payable?


D a. P 120,000
b. P 90,000
c. P 60,000
d. P 30,000

67. When shall the Percentage Tax Return be filed?


A a. April 25, 2021
b. April 24, 2021
c. April 21, 2021
d. April 15, 2021

68. How much is the Value-Added Tax?


B a. P 360,000
b. P 240,000
c. P 120,000
d. None. VAT threshold not exceeded

69. When shall the Quarterly VAT be filed?


A a. April 25, 2021
b. April 24, 2021
c. April 21, 2021
d. April 15, 2021

70. Assuming it is not VAT-registered, how much shall be the total percentage
taxes?
C a. P90,000
b. P60,000
c. P50,000
d. P30,000

- END –

Page 11 of 14 0915-2303213  [email protected]


TAXATION
ReSA Batch 43 - May 2022 CPALE Batch
09 Feb 2022  6:00 PM to 9:00 PM TAX First Pre-Board Exam

ANSWERS & SOLUTIONS/CLARIFICATIONS

6. Selling price P500,000


Less: Basis (basis to the last owner who did not acquire the 200,000*
property by donation)
Gain from sale P300,000

23. Claim against an insolvent person (fully uncollectible P500,000


Claim against an insolvent person (80% x 100,000) 80,000
Total P580,000

24. Value to take P 650,000


Less: Mortgage paid 50,000
Initial basis 600,000
Less: Proportional deduction (P600,000/P1,800,000 X P300,000) 100,000
Final basis 500,000
Rate 20%
Vanishing deduction P 100,000

28. The gift was exempted because the administrative expenses of the donee-institution did not
exceed 30% of the total expenses (5,000,000/P20,000,000 = 25%).

38. Gross sales P450,000


Tax rate 3%
Percentage tax 13,500
Effective July 1, 2020 until June 30, 2023, the rate shall be 1%.

39. Freight and cargo fees (cargo originating from the Philippines in a continuous
and uninterrupted flight to Japan, passage documents sold in Japan) P5,000,000
Advance payments for cargo originating from the Philippines in a continuous
and uninterrupted flight to Singapore (passage documents sold in Singapore) 3,000,000
Gross receipts 8,000,000
Tax rate 3%
Common carrier’s tax P 240,000

40. Cash sales P1,360,000


Credit card sales 624,200
Gross receipts (Dude Videoke Bar) P1,984,200
Tax rate 18%
Amusement tax P 357,156

42. Gross sales – Convenience store P800,000.00


Gross receipts - Bookkeeping 300,000.00
Total gross sales/receipts 1,100,000.00
Less: Amount allowed as deduction 250,000.00
Taxable income P850,000.00

43. Taxable income P850,000.00


Tax rate 8%
Tax due P 68,000.00

44. Gross sales – Convenience store P 800,000.00


Gross receipts - Bookkeeping 300,000.00
Total gross sales/receipts 1,100,000.00
Less: Cost of sales 600,000.00
Gross income 500,000.00
Less: Operating expenses 200,000.00
Taxable income P 300,000.00
Tax due 250,000.00 Exempt
50,000.00 x 20% 10,000.00 P 10,000.00

Aside from income tax, Ms. Leni is likewise liable to pay business tax.

Page 12 of 14 0915-2303213  [email protected]


TAXATION
ReSA Batch 43 - May 2022 CPALE Batch
09 Feb 2022  6:00 PM to 9:00 PM TAX First Pre-Board Exam

45. Effective January 1, 2018 to December 31, 2022


If the taxable income is:
Over But not over The tax shall be Plus Of excess over
P 250,000 0%
P 250,000 400,000 20% P 250,000
400,000 800,000 P 30,000 25% 400,000
800,000 2,000,000 130,000 30% 800,000
2,000,000 8,000,000 490,000 32% 2,000,000
8,000,000 2,419,000 35% 8,000,000

Related activities Unrelated activities Total


Gross receipts P15,000,000.00 P18,000,000.00 P33,000,000.00
Less: Cost of services 6,000,000.00 5,000,000.00 11,000,000.00
Gross income 9,000,000.00 13,000,000.00 22,000,000.00
Less: Allowable deductions 3,250,000.00 2,000,000.00 5,250,000.00
Net taxable income (46)P 5,750,000.00 (47) P11,000,000.00 16,750,000.00
Tax rate* (48) 25%
Income tax due 4,187,500.00
Less: Payments, first 3 quarters 2,000,000.00
Income tax payable (49)P2,187,500.00

51. Conjugal family house P9,000,000


Exclusive family lot 1,000,000
Total P10,000,000

52. Conjugal ordinary deductions (2,300,000 – 300,000) P2,000,000


Exclusive deductions 1,000,000
Total P3,000,000

53. Standard deduction P5,000,000


family home
Family house (1/2 x 9,000,000) 4,500,000
Family lot (full) 1,000,000
Total 5,500,000
Maximum 10,000,000
Lower 5,500,000
Total P10,500,000

Exclusive Common Total


34 Gross estate P6,000,000 P23,000,000 P29,000,000
35 Less: Ordinary deductions 1,000,000 2,000,000 3,000,000
36 Estate after ordinary deductions 5,000,000 21,000,000 26,000,000
37 Less: Special deductions
37A Standard deduction 5,000,000
37B Family home 5,500,000
37C Others (specify) -
37D Total special deductions 10,500,000
38 Net estate 15,500,000
39 Less: Share of surviving spouse (1/2 x
21,000,000*) 10,500,000
40/16 Net taxable estate 5,000,000 (54)
17 Applicable tax rate 6%
18 Estate tax due P 300,000 (55)

56. Sale of canned fruits P2,000,000


Sale of canned sardines 3,000,000
Total P5,000,000

57. Gross sales P5,000,000


Tax rate 12%
Output tax P600,000

58. Presumptive input tax on purchase of tomatoes and onions (500,000 x 4% = P20,000))

Page 13 of 14 0915-2303213  [email protected]


TAXATION
ReSA Batch 43 - May 2022 CPALE Batch
09 Feb 2022  6:00 PM to 9:00 PM TAX First Pre-Board Exam

59. Presumptive input tax on purchase of tomatoes and onions (500,000 x 4%) P20,000
Passed-on VAT on purchase of corn oil (300,000 x 12%) 36,000
Passed-on VAT on purchase of refined sugar from sugar refinery (50,000 x 12%) 6,000
Passed-on VAT on purchase of packaging materials (800,000 x 12%) 96,000
Passed-on VAT on purchase of labels (150,000 x 12%) 18,000
Passed-on VAT on trucking services (50,000 x 12%) 6,000
Total P182,000

60. 19B Output tax P600,000


24 Total allowable input tax 182,000
25 Net VAT payable 418,000
Less: 26A Monthly VAT payments, previous two months 300,000
27 Tax Still Payable P118,000

61. Car, Philippines P1,200,000

62. Unpaid tax on car assumed by the donee P120,000

63. 27 Total Gifts In This Return P1,200,000


Less: Deductions
Unpaid tax on car assumed by the donee
33 Total Deductions Allowed 120,000
34 Total net gifts in this return 1,080,000
Add: 35 Total prior net gifts during the calendar year -
36 Total net gifts 1,080,000
37 Less: Exempt gift 250,000
38/14 Total net gift subject to tax P 830,000

64. 38/14 Total net gift subject to tax 830,000


15 Applicable donor’s tax rate 6%
16 Total donor’s tax due 49,800
Less: 17Tax credit/payments -
18 Tax payable P49,800
Add:19 Penalties -
20 Total amount payable P49,800

65. 27 Total Gifts In This Return P1,200,000


Less: Deductions
Unpaid tax on car assumed by the donee
33 Total Deductions Allowed 120,000
34 Total net gifts in this return 1,080,000
Add: 35 Total prior net gifts during the calendar year -
36 Total net gifts 1,080,000
37 Less: Exempt gift 250,000
38/14 Total net gift subject to tax 830,000
15 Applicable donor’s tax rate 6%
16 Total donor’s tax due 49,800
Less: 17Tax credit/payments -
18 Tax payable P49,800
Add:19 Penalties -
20 Total amount payable P49,800

66. Gross receipts (transport of passengers) P1,000,000


Tax rate 3%
Common carrier’s tax P 30,000

68. Gross receipts (transport of goods) P1,500,000


Gross receipts (transport of cargoes 500,000
Total 2,000,000
Tax rate 12%
Value-added tax P 240,000

70. Common carrier’s tax (transport of passengers)


(1,000,000 x 3%) P30,000
Percentage on Persons Exempt from VAT (transport of goods and cargoes)
(2,000,000 x 1%) 20,000
Total P50,000

Page 14 of 14 0915-2303213  [email protected]


RESA
FINAL
PREBOARD
ReSA - THE REVIEW SCHOOL OF ACCOUNTANCY
CPA Review Batch 43  May 2022 CPALE  23 April 2022  8:00 – 11:00 AM

TAXATION FINAL PRE-BOARD EXAMINATION

INSTRUCTIONS: Select the correct answer for each of the questions. Mark only one
answer for each item by shading the box corresponding to the letter of your choice on
the answer sheet provided. STRICTLY NO ERASURES ALLOWED. Use pencil no. 2 only.

1. A citizen who has been previously considered as nonresident citizen and who arrives
in the Philippines at any time during the taxable year to reside permanently in the
Philippines shall likewise be treated as a nonresident citizen for the taxable year
in which he arrives in the Philippines with respect to his:
a. income derived from sources abroad until the date of his arrival in
the Philippines.
b. income derived from sources within the Philippines.
c. income derived from sources abroad until the last day of the calendar
year.
d. income derived from sources within and outside the Philippines.

2. First statement: The husband and wife shall compute their individual income tax
separately based on their respective total taxable income.
Second statement: If any income cannot be definitely attributed to or identified
as income exclusively earned or realized by either of the spouses, the same shall
be divided equally between the spouses for the purpose of determining their
respective taxable income.
a. Both statements are correct
b. Both statements are incorrect
c. Only the first statement is correct
d. Only the second statement is correct

3. There shall be levied, collected and paid for each taxable year upon the income
received by every alien individual employed by regional or area headquarters and
regional operating headquarters established in the Philippines by multinational
companies from such regional or area headquarters and regional operating
headquarters, a tax equal to:
a. Twenty-five percent (25%) of such gross income.
b. Twenty percent (20%) of such gross income.
c. Fifteen percent (15%) of such gross income.
d. Regular income tax rate under Section 24 (A) (2) (a) of the Tax Code.

4. Ms. Cora Fiel, an alien employed in PetrolEx Corporation that is a Petroleum Service
Contractor, received compensation income of P 5,000,000.00 for 2018. She is
married and has four (4) minor children. How much is the tax due?
a. P 1,450,000
b. P 1,402,000
c. P 750,000
d. None of the choices

5. The MCIT shall be imposed upon a domestic corporation or a resident foreign


corporation:
I - whenever such corporation has a zero or negative taxable income;
II - when the amount of minimum corporate income tax is greater than the normal
income tax due from such corporation.
a. Both I and II are correct
b. Neither I nor II is correct
c. Only I is correct
d. Only II is correct

6. Aliw Service Corporation, registered with BIR in 2018, has the following data for
the year 2021:
Gross receipts P1,000,000
Discounts given 100,000
Returns and allowances 150,000
Salaries of personnel directly involved in
the supply of services 300,000
Fees of consultants directly involved in
the supply of services 50,000
Rental of equipment directly used in the
supply of services 70,000
Operating expenses 420,000

Page 1 of 17 0915-2303213  [email protected]


TAXATION
ReSA Batch 43 - May 2022 CPALE Batch
23 April 2022  8:00 AM to 11:00 AM TAX Final Pre-Board Exam

How much is the income tax due and payable?


a. P 27,000
b. P 6,600
c. Zero
d. None of the choices

7. The recovery of the account previously written off would constitute taxable income
only if in the year of recognition of being worthless, the write-off resulted in a
reduction of a taxable income.
a. Reciprocity rule
b. Equity of the incumbent rule
c. Tax benefit rule
d. Materiality rule

8. One of the following is considered an income for income taxation.


a. Advance in the value of the person or a corporation
b. Increase in book value of real property and other fixed assets of a
corporation as a result of their re-appraisal
c. Net increase in the value of livestock or increase of its inventory
d. Items of gross income which have been credited or set apart for the
taxpayer without restriction

9. Excise tax on certain articles is an example of:


a. An indirect tax.
b. A direct tax.
c. A local tax.
d. A transfer tax.

10. An annual tax of P1,000 was imposed upon all residents of the Philippines, who are
above 21 years of age, with a gross income of P250,000, whether or not they send
their children to public schools, for the purpose of raising funds in order to
improve public school buildings. The tax is:
a. violative of the equal protection clause of the Constitution.
b. confiscatory.
c. for public purpose.
d. contradicts the inherent limitations.

11. A law was passed by Congress which granted tax amnesty to those who have not paid
income taxes for a certain year without at the same time providing for the refund
of taxes to those who have already paid them. The law is:
a. valid because there is a valid classification.
b. not valid because those who did not pay their taxes are favored over
those who have paid their taxes.
c. valid because it was Congress which passed the law and it did not
improperly delegate the power to tax.
d. not valid because only the President with the approval of Congress
may grant amnesty.

12. In case of conflict between tax laws and generally accepted accounting principles
(GAAP):
a. both tax laws and GAAP shall be enforced.
b. GAAP shall prevail over tax laws.
c. tax laws shall prevail over GAAP.
d. the issue shall be resolved by the court.

13. The fair market value of real properties located in each zone or area, shall be subject
to automatic adjustment once every how many years through rules and regulations
issued by the Secretary of Finance based on the current Philippine valuation
standards?
a. 3 years
b. 5 years
c. 7 years
d. 10 years

Page 2 of 17 0915-2303213  [email protected]


TAXATION
ReSA Batch 43 - May 2022 CPALE Batch
23 April 2022  8:00 AM to 11:00 AM TAX Final Pre-Board Exam

14. Which of the following cases may not be compromised?


a. Delinquent accounts
b. Cases under administrative protest after issuance of the Final
Notice of Assessment to the taxpayer still pending in the BIR
c. Civil tax cases being disputed before the courts
d. Criminal violations involving criminal tax fraud.

15. The following are instances when penalties and/or interest imposed on the taxpayer
may be abated or cancelled on the ground that the imposition thereof is unjust and
excessive, except when the:
a. Filing of the return or payment of the tax is made at the wrong
venue.
b. Taxpayer’s mistake in payment of his tax is due to erroneous
written official advice of a revenue officer.
c. Assessment is brought about or a result of the taxpayer’s non-
compliance with the law due to a difficult interpretation of the
said law.
d. Taxpayer is declared insolvent or bankrupt.

16. First statement: In civil tax cases involving collection of internal revenue taxes,
prescription is construed strictly against the government and liberally in favor
of the taxpayer.

Second statement: In criminal tax cases involving tax offenses punishable under
the Tax Code, prescription is construed strictly against the government.
a. Both statements are correct
b. Both statements are incorrect
c. Only the first statement is correct
d. Only the second statement is correct

17. If at any time prior to the consummation of the sale all proper charges are paid
to the officer conducting the sale, the goods or effects distrained:
a. shall be restored to the owner.
b. shall no longer be restored to the owner.
c. shall be brought to the Revenue District Officer.
d. shall be brought to the Regional District Officer.

18. If after review and evaluation by the Commissioner or his duly authorized
representative, it is determined that there exists sufficient basis to assess the
taxpayer for any deficiency tax or taxes, the said Office shall issue to the
taxpayer the:
a. Notice of informal conference
b. Notice of discrepancies
c. Preliminary Assessment Notice
d. Formal Assessment Notice

19. For requests for reinvestigation, the taxpayer shall submit all relevant supporting
documents in support of his protest within how many days from date of filing of
his letter of protest, other-wise, the assessment shall become final?
a. Sixty (60) days
b. Thirty (30) days
c. Twenty (20) days
d. None of the choices

20. In 2018, Ms. Glai Espenilla Bangug, a financial comptroller of EB Company,


earns annual compensation of P1,500,000, inclusive of 13th month and other
benefits in the amount of P80,000 and mandatory SSS contribution of P3,500 and
Philhealth contribution of P2,000. Aside from her employment income, she owns
a convenience store, VAT-registered, with gross sales of P3,000,000. Sales
discount amounts to P300,000; sales returns and allowances amount to P150,000.
Her cost of sales and operating expenses are P1,000,000 and P600,000
respectively and with non-operating income of P100,000. Payments for the first
three (3) quarters amount to P300,000.

Page 3 of 17 0915-2303213  [email protected]


TAXATION
ReSA Batch 43 - May 2022 CPALE Batch
23 April 2022  8:00 AM to 11:00 AM TAX Final Pre-Board Exam
Can she avail of the 8% income tax rate?
a. Yes, because her gross sales do not exceed the VAT threshold.
b. No, because she is VAT- registered.
c. Yes, because she is a mixed income earner.
d. No, because her total income including compensation income exceed the
VAT threshold.

21. Under the TRAIN, the books of accounts shall be audited and examined yearly by
independent Certified Public Accountants and their income tax returns
accompanied with a duly accomplished Account Information Form (AIF) which shall
contain, among others, information lifted from certified balance sheets, profit
and loss statements, schedules listing income-producing properties and the
corresponding income therefrom and other relevant statements if the:
a. gross annual sales, earnings, receipts or output exceed Three
million pesos (P3,000,000).
b. gross quarterly sales, earnings, receipts or output exceed Three
million pesos (P3,000,000).
c. gross annual sales, earnings, receipts or output amount to Three
million pesos (P3,000,000)or more.
d. gross quarterly sales, earnings, receipts or output exceed One
Hundred Fifty Thousand (P150,000).

22. Your client dies on July 14, 2018. You are appointed as administrator. You
withdraw from his bank deposit P100,000 because the estate needs cash to
settle some obligations. What is the consequence of your withdrawal from
the decedent’s bank deposit?
a. subject to final withholding tax of 6%
b. subject to creditable withholding tax of 6%
c. Not subject to withholding tax
d. Subject to penalty and interest because such withdrawal is not
allowed.

23 and 24 are based on the following: A resident decedent left the following
properties:
Real properties P 10,000,000
Personal properties excluding bank deposit 15,000,000
Bank deposit (P 940,000 net of final tax, was withdrawn
from the account of the decedent after the decedent 3,000,000
died)

23. The gross estate of the decedent was:


a. P 28,000,000
b. P 27,000,000
c. P 26,000,000
d. P 25,000,000

24. The final withholding tax on amount withdrawn was:


a. P 200,000
b. P 100,000
c. P 60,000
d. None of the choices

An unmarried decedent died leaving properties he inherited 4 ½ years ago which


had fair market value of P800,000 at the time of his death (P650,000 at the time
of inheritance). The present decedent mortgaged the property for P50,000 and
paid P20,000 before he died). Other properties in his gross estate had fair
market value of P1,000,000. The total expenses, losses, indebtedness, taxes and
transfer for public purpose amounted to P300,000.

25. How much was the vanishing deduction?


a. P 225,000
b. P 108,333
c. P 102,900
d. P 100,000

Page 4 of 17 0915-2303213  [email protected]


TAXATION
ReSA Batch 43 - May 2022 CPALE Batch
23 April 2022  8:00 AM to 11:00 AM TAX Final Pre-Board Exam
26 and 27 are based on the following: Transfer Taxes The decedent is a resident
unmarried head of family who died on March 1, 2019. He left the following properties,
obligations and expenses:

Real and personal properties P14,000,000


Family home 30,000,000
Ordinary deductions
Unpaid real estate tax 2,000,000
Medical expenses 600,000

26. The taxable net estate is:


a. P42,000,000.
b. P32,000,000.
c. P27,000,000.
d. P23,000,000.

27. Continuing the preceding number, the estate tax return shall be filed and the
estate tax paid on or before to avoid penalties:
a. March 1, 2020
b. March 1, 2021
c. September 1, 2018
d. None of the choices

28 and 29 are based on the following: During the current year, Jose Tugas who resides
in 158 R. Papa Street, Sampaloc Manila and with TIN 135-567-890-006 made the following
gifts:
Date Donee Amount of donation
June 1, 2018 Anton, his son, on P150,000 cash
account of his
marriage celebrated
June 1, 2018
July 10, 2018 His friend Carlos P400,000 a second hand motor
vehicle
September 30, 2018 His daughter Dana P450,000 cash dowry, on account
of her scheduled marriage on
October 25, 2018
November 23, 2018 His father A parcel of land worth P180,000,
subject to the condition that
his father would assume the
mortgage indebtedness of Jose
in the amount of P40,000;

28. Using Donor’s Tax Return (BIR Form No. 1800), how much will be reflected on line
38 (Total net gifts subject to tax) for the return to be filed on November 23,
2018?
a. P1,140,000
b. P 890,000
c. P 140,000
d. None of the choices

29. Using the Donor’s Tax Return (BIR Form No. 1800), how much will be reflected on
line 18?
a. P53,400
b. P 8,400
c. (P6,600)
d. None of the choices

30 to 33 are based on the following: Your client, Antonio Manuel, is engaged in various
transactions that are subject to percentage taxes. His address is 143 S. Loyola
Street, Sampaloc, Manila. The TIN is 143-678-910-005. He is under RDO 032. Email ad
is [email protected].

The following data for the first quarter of the current year are presented for your
perusal. Answer the questions by filling up the line in the actual Quarterly
Percentage Tax Return (2551Q).

Page 5 of 17 0915-2303213  [email protected]


TAXATION
ReSA Batch 43 - May 2022 CPALE Batch
23 April 2022  8:00 AM to 11:00 AM TAX Final Pre-Board Exam
Gross receipts, sale of gas and water P1,500,000
Expenses, sale of gas and water 200,000
Gross receipts, rentals of office spaces, net of 1% 742,500
withholding tax
Expenses, rental of office spaces 150,000
Gross receipts, life insurance premiums 800,000
Expenses, life insurance business 100,000

30. Using line 14 of BIR Form 2551 Q, how much is total tax due?
a. P91,500
b. P68,500
c. P53,500
d. P30,000

31. Assuming your client filed the return and paid the tax on July 25 and it was found
out that it was due to willful neglect, using line 20, how much is the surcharge?
a. P36,305
b. P34,250
c. P26,750
d. P17,125

32. Using the same information in the preceding number, using line 21, how much is the
interest?
a. P8,220
b. P3,425
c. P1,605
d. P1,600

33. Using line 24, how much is the total amount payable excluding compromise penalty?
a. P104,805
b. P 85,730
c. P 74,355
d. P 74,350

34 and 35 are based on the following: In the year 2021, Mr. Nicko Macariola has the
following transactions:

Gross receipts, trucking business P1,500,000


Gross receipts, lease of residential units
(monthly rental is P20,000 per unit) 2,500,000
Gross receipts, practice of accountancy 1,000,000
Sale of four (4) residential lots at P1,500,000 each to
four (4) individual buyers for socialized housing 6,000,000
Sale of three (3) parking spaces at P500,000 each 1,500,000

34. How much is the VAT-exempt gross receipts, assuming the taxpayer is VAT-registered?
a. P8,500,000
b. P6,000,000
c. P3,500,000
d. None of the choices

35. How much is total VAT-subject amount, assuming the taxpayer is not VAT-registered?
a. P8,500,000
b. P7,500,000
c. P6,500,000
d. None of the choices

36. In 2018 a stockholder of a closely held corporation owns 100,000 shares before the
IPO. The cost of the share is P1,000,000. During the IPO, the shares are selling
at P12 per share. After the IPO, the outstanding shares of the closely held
corporation are 1,000,000 shares and are now selling at P14 per share at the local
stock exchange.

Your friend asks for your advice whether to sell his shares during the IPO through
the local stock exchange or sell after the IPO not through the local stock exchange.

Page 6 of 17 0915-2303213  [email protected]


TAXATION
ReSA Batch 43 - May 2022 CPALE Batch
23 April 2022  8:00 AM to 11:00 AM TAX Final Pre-Board Exam
What advice will you give him to minimize income tax?
a. Sell during the IPO because he will pay lesser tax.
b. Sell after the IPO but directly to a buyer so that he will pay lesser
tax.
c. Sell any time because there is no advantage as far as taxes due are
concerned.
d. Sell it below cost after the IPO directly to a buyer so he will not
be subject to tax on capital gain.

37 and 38 are based on the following: ABC Corporation, VAT-registered real estate
dealer, sold a residential lot on July 2, 2021 for P1,500,000. The down payment was
P200,000. The first installment of P100,000 was made on the year of sale. The zonal
value of the subject property at the time of sale amounted to P2,000,000.

37. Would the sale qualify under installment plan?


a. Yes, because the initial payment did not exceed 25% of the selling
price.
b. No, because the amount of the initial payments were not known.
c. Yes, because the sale had initial payments and, therefore, qualified
under installment plan.
d. No, because, as a rule, sale of real property would always be
considered sale under cash basis.

38. How much was the output tax on the installment payment?
a. P48,000
b. P36,000
c. P32,000
d. Exempt from VAT

39. Mr. Florence Binaluyo donates a piece of land to the City of Masbate. The fair
market value per BIR of the piece of land is P2,000,000 and its value per Tax
Declaration is P1,800,000. How much is the documentary stamp tax, if any?
a. P30,000
b. P30,000
c. None, exempt from DST
d. None of the choices

40. On every original issue, whether on organization, reorganization or for any lawful
purpose, of shares of stock by any association, company or corporation, the
documentary stamp tax is:
a. One peso and fifty centavos (P1.50) on each Two hundred pesos
(P200), or fractional part thereof, of the par value of such stock.
b. Two pesos (P2.00) on each Two hundred pesos (P200), or fractional
part thereof, of the par value, of such shares of stock
c. Three pesos (P3.00) on each Two hundred pesos (P200), or fractional
part thereof, of the par value, of such shares of stock
d. Fifteen pesos (P15.00)on each Two hundred pesos (P200), or fractional
part thereof, of the par value, of such shares of stock

41. These are taxes on goods manufactured or produced in the Philippines for
domestic sales or consumption or for any other disposition and to things imported
as well as services performed in the Philippine, which tax shall be in addition
to the value-added tax.
a. Percentage Taxes
b. Income Tax
c. Documentary Stamp Taxes
d. Excise Taxes

42. Which of the following articles shall be subject to specific excise tax?
a. Excise tax on alcoholic products
b. Excise tax on automobiles
c. Excise tax on non-essential services
d. Excise tax on sweetened beverages

Page 7 of 17 0915-2303213  [email protected]


TAXATION
ReSA Batch 43 - May 2022 CPALE Batch
23 April 2022  8:00 AM to 11:00 AM TAX Final Pre-Board Exam
43. A residential building located in one of the cities in Metro Manila has a fair
market value of P10,000,000. The city ordinance fixed the actual assessment level
at 60%. How much is the basic real property tax, excluding the Special Educational
Fund (SEF)?
a. P120,000
b. P 60,000
c. P30,000
d. None of the choices

44. Unless otherwise provided in the Local Government Code, the tax period of all
local taxes, fees and charges shall be:
a. calendar year.
b. fiscal year.
c. either calendar year or fiscal year at the option of the taxpayer.
d. neither calendar year nor fiscal year at the option of the LGU.

45. Who of the following shall not qualify as senior citizen?


a. Resident Filipino citizen, 60 years old
b. Filipino citizen with dual citizenship, 60 years old, 6 months
residency in the Philippines
c. Resident alien, 65 years old
d. None of the choices

46. A Company that hires a PWD shall be allowed subject to certain conditions:
a. Ten percent (10%) of the total amount paid as salaries and wages to
senior citizens as additional deduction.
b. Fifteen percent (15%) of the total amount paid as salaries and wages
to senior citizens as additional deduction.
c. Twenty-five percent (25%) of the total amount paid as salaries and
wages to senior citizens as additional deduction.
d. Thirty percent (30%) of the total amount paid as salaries and wages
to senior citizens as additional deduction.

47. One of the following is not qualified to register as Barangay Micro Business
Enterprise (BMBE).
a. Bakery with total assets not exceeding P3,000,000
b. Motor shop with total assets not exceeding P3,000,000
c. A CPA practitioner with total assets not exceeding P3,000,000
d. Farm producing agricultural products with total assets not exceeding
P3,000,000

48. Which of the following is not a relief from double taxation?


a. Tax credit
b. Allowance of deduction for foreign taxes
c. Tax treaties
d. Not reporting income or transactions in foreign countries

49. LAB Corporation originally issued stock dividends valued at P2,000,000. How much
is the documentary stamp tax, if any?
a. P30,000
b. P20,000
c. P15,000
d. None of the choices

50. The final income tax return for the taxable year 2018 which was due on April 15,
2019 was filed earlier on March 15, 2019. A substantially amended return was
filed on May 31, 2019. When is the last day to make a valid assessment?
a. March 15, 2022
b. April 15, 2022
c. May 31, 2022
d. May 31, 2025

Page 8 of 17 0915-2303213  [email protected]


TAXATION
ReSA Batch 43 - May 2022 CPALE Batch
23 April 2022  8:00 AM to 11:00 AM TAX Final Pre-Board Exam
SITUATIONAL
During the year 2018, ABC Corporation paid for the monthly rental of a residential
house of its branch manager, Mr. J. de la Cruz, amounting to P 68,000.

51. How much is the value of the fringe benefit?


a. P68,000
b. P34,000
c. P16,000
d. P11,333

52. How much is the monetary value of the fringe benefit?


a. P68,000
b. P34,000
c. P16,000
d. P11,333

53. How much is the fringe benefit tax?


a. P18,308
b. P16,000
c. P 6,000
d. None of the choices
SITUATIONAL

An individual taxpayer holds shares of stock as investment which he bought from a


publicly-listed company for P500,000 (P500,000 par value). The shares are listed
and traded in the local stock exchange. He sells them for P750,000. How much is the
capital gains tax on the sale, if any?

54. How much is the capital gains tax, if any?


a. P112,500
b. P 37,500
c. P 4,500
d. None, not subject to capital gains tax

55. How much is the capital gains tax on the sale assuming the corporation from which
the shares are bought is not compliant with the mandatory minimum public ownership?
a. P112,500
b. P 37,500
c. P 4,500
d. None, not subject to capital gains tax

SITUATIONAL

A resident citizen, 50 years old, married, with three (3) qualified dependent
children asks you to assist him in computing his taxable net income for the year
2018:

Gross professional income, net of 15% withholding tax P680,000


Professional expenses 300,000
Retirement benefits received from his previous
employer that maintained a reasonable private
pension plan (he served the company for 12 years) 250,000
Lump sum benefits received from SSS 200,000
Prize in a sports tournament sponsored by a group
of businessmen promoting health products 50,000
Gain from sale of bonds (maturity period is 4 years) 10,000
Interest income from bonds with a maturity period
of 7 years 20,000
Interest on long term deposits with maturity period
of 5 years 30,000
Philippine Lotto winnings 500,000
Share in the net income of a business partnership 100,000

Page 9 of 17 0915-2303213  [email protected]


TAXATION
ReSA Batch 43 - May 2022 CPALE Batch
23 April 2022  8:00 AM to 11:00 AM TAX Final Pre-Board Exam
56. How much is the total amount of excluded or exempted income?
a. P1,150,000
b. P1,000,000
c. P 500,000
d. P 480,000

57. How much is the final withholding tax on certain income?


a. P130,000
b. P120,000
c. P110,000
d. P 20,000

58. How much is the taxable net income?


a. P651,176
b. P510,000
c. P385,000
d. P375,000

59. How much is the tax due and payable?


a. 81,544
b. 61,544
c. 57,500
d. (62,500)

60. How much is the total income tax expenses for the year?
a. P177,500
b. P167,500
c. P 77,500
d. P 57,500

SITUATIONAL
The decedent was a resident unmarried head of family died on January 6, 2022. The
following data were provided from his estate:

Real and personal properties (excluding a piece of land valued


at P2,000,000 transferred mortis causa to his son, and car
valued at P500,000 which was revocably transferred to a nephew,
decedent waived his right to revoke before he died)
P14,000,000
Family home 30,000,000
The following items were claimed as deductions from the gross estate:
Funeral expenses P 100,000
Judicial expenses 200,000
Unpaid real estate tax 2,000,000
Loss of a personal property due to theft (500,000 recovered
from Insurance) 1,000,000
Transfer for public use 500,000
Medical expenses 300,000
61. How much is to be reflected in line 34 of BIR Form No. 1801 (gross estate)?
a. P46,500,000
b. P46,000,000
c. P42,500,000
d. P42,000,000
62. How much is to be reflected in line 35 of BIR Form 1801 (the total ordinary
deductions)?
a. P3,800,000
b. P3,500,000
c. P3,200,000
d. P3,000,000

63. How much is to be reflected in line 37D of BIR Form 1801 (the total special
deductions)?
a. P15,000,000
b. P11,000,000
c. P 2,000,000
d. P 1,500,000

Page 10 of 17 0915-2303213  [email protected]


TAXATION
ReSA Batch 43 - May 2022 CPALE Batch
23 April 2022  8:00 AM to 11:00 AM TAX Final Pre-Board Exam
64. How much is to be reflected in line 40 of BIR Form 1801 (net taxable estate)?
a. P42,000,000.
b. P32,000,000.
c. P28,000,000.
d. P23,500,000.

65. How much is to be reflected in line 16 of BIR Form No. 1800 (the donor’s tax), if
any, on certain transfer mentioned in the information given above?
a. P30,000
b. P15,000
c. P10,000
d. None, not subject to donor’s tax

SITUATIONAL
A VAT-registered corporation has the following data taken from the books of accounts
for the first calendar quarter of 2018:
Gross sales P150,000,000
Sales returns and allowances 10,000,000
Sales discount (given at the time of sale including P500,000
discount given to senior citizens) 4,500,000
Cost of sales 90,500,000
Office equipment purchased January 1, 2018 1,100,000
Vehicle for land transport imported January 1, 2018 2,500,000
Pre-owned helicopter acquired January 1, 2018 10,000,000
Maintenance expenses, vehicle for land transport and helicopter 150,000
Purchase of goods for sale, (included in the cost of sales above) 30,000,000
Operating expenses (40% with passed-on VAT) 5,000,000
Office supplies purchased (wholly used) 900,000
VAT payments for January 5,000,000
February 2,000,000
66. How much is the output tax?
a. P18,000,000
b. P16,800,000
c. P16,320,000
d. P16,260,000
67. Can the taxpayer claim creditable input tax on importation of vehicle for land
transport?
a. Yes, because the vehicle for land transport is used for the VAT-
subject business.
b. No, because the value of the vehicle exceeded P2,400,000.
c. Yes, because the vehicle is a depreciable asset for income tax
purposes.
d. No, because input tax is not allowed on importation.
68. Can the taxpayer claim input tax on acquisition of helicopter?
a. Yes, because the helicopter is used for the VAT-subject business.
b. No, because the helicopter is a non-depreciable asset, hence, no
input tax shall be allowed.
c. Yes, because the helicopter is a depreciable asset for income tax
purposes, hence, input tax shall be allowed.
d. No, because input tax is not allowed on helicopters without any
exception.
69. How much is the creditable input tax?
a. P4,080,000
b. P3,954,600
c. P3,950,200
d. P3,948,000
70. How much is the VAT payable upon filing of the quarterly return on or before April
25, 2018?
a. P6,920,000
b. P5,845,400
c. P5,370,000
d. P5,305,400
- END –

Page 11 of 17 0915-2303213  [email protected]


TAXATION
ReSA Batch 43 - May 2022 CPALE Batch
23 April 2022  8:00 AM to 11:00 AM TAX Final Pre-Board Exam

ANSWERS & SOLUTIONS/CLARIFICATIONS


1 A 26 C 51 A
2 A 27 A 52 B
3 D 28 B 53 A
4 A 29 B 54 D
5 A 30 C 55 B
6 C 31 C 56 C
7 C 32 D 57 B
8 D 33 D 58 B
9 A 34 B 59 D
10 C 35 C 60 A
11 A 36 D 61 B
12 C 37 A 62 D
13 A 38 A 63 A
14 D 39 C 64 C
15 D 40 B 65 B
16 C 41 D 66 D
17 A 42 D 67 B
18 A 43 A 68 B
19 A 44 A 69 B
20 B 45 C 70 D
21 A 46 C
22 A 47 C
23 B 48 D
24 C 49 B
25 B 50 C

1. Reference: Section 22 (E) (4), NIRC, as amended

2. Reference: Section 24 (A) (1) (a), (b) and (c), NIRC, as amended

3. Reference: Section 25 (C), NIRC, as amended

Section 4 (C), Revenue Regulations No. 8-2018


The preferential income tax rate under subsection (C) (D) and (E) of Section 25 of the
Tax Code, as amended, shall no longer be applicable without prejudice to the application
of preferential tax rates under existing international tax treaties, if warranted. Thus. all
concerned employees of the regional or area headquarters and regional operating
headquarters of multinational companies, offshore banking units and petroleum service
contractor and subcontractor ‘s shall be subject to the regular income tax rate under
Sec. 24(A)(2) ta) of the Tax Code. as amended.

4. Compensation income P5,000,000


Tax due under Section 24 (A)
2,000,000 P490,000
3,000,000 x 32% 960,000 P1,450,000

5. References: Section 2.27 (E) (1), Revenue Regulations No. 9-98, as amended by
Revenue Regulations No. 12-2007
Section 2.28 (A) (2), Revenue Regulations No. 9-98, as amended by
Revenue Regulations No. 12-2007

Page 12 of 17 0915-2303213  [email protected]


TAXATION
ReSA Batch 43 - May 2022 CPALE Batch
23 April 2022  8:00 AM to 11:00 AM TAX Final Pre-Board Exam
6. Gross receipts P1,000,000
Less: Discounts (P 100,000)
Returns and allowances ( 150,000) (250,000)
Net receipts 750,000
Less: Cost of services
Salaries of personnel (P 300,000)
Fees of consultants ( 50,000)
Rental of equipment ( 70,000 ) (420,000)
Gross income 330,000
Less: Operating expenses 420,000
Net operating loss (P 90,000)

RCIT P - ____
MCIT (not subject yet) P___-____
Tax due and payable Zero

8. Reference: Section 52, Revenue Regulations No. 2

13. Reference: Section 6 (E), NIRC, as amended under TRAIN

14. Reference: Section 2, Revenue Regulations No. 30-2002

16. References: CIR vs. Goodrich Phils., Inc. G.R. 104171, Feb. 24, 1999
Lim vs. CA, G.R. 48134-37, Oct. 18, 1990

In criminal tax cases involving tax offenses punishable under the Tax Code, prescription is
construed strictly against the taxpayer.

17. Reference: Section 210, NIRC, as amended

SEC. 210. Release of Distrained Property Upon Payment Prior to Sale. - If at any
time prior to the consummation of the sale all proper charges are paid to the officer
conducting the sale, the goods or effects distrained shall be restored to the owner.

18. An order the to expedite the processing of Letter Notice (LN) cases, the issuance of Notice
for Informal Conference may immediately commence, even without prior issuance of Letters
of Authority, as required in certain situations, as prescribed in the existing RMOs on the LN
system.

19. References: Section 228, NIRC, as amended


Section 3.1.5, Revenue Regulations No. 12-99
as amended under Revenue Regulations No. 18-2013

The term “relevant supporting documents” refer to those documents necessary to support
the legal and factual bases in disputing a tax assessment as determined by the taxpayer.

The sixty (60)-day period for the submission of all relevant supporting documents shall not
apply to requests for reconsideration.

Furthermore, the term “the assessment shall become final” shall mean the taxpayer is
barred from disputing the correctness of the issued assessment by introduction of newly
discovered or additional evidence, and the FDDA shall consequently be denied.

23. Real properties, Philippines P10,000,000


Personal properties excluding bank deposit 15,000,000
Bank deposit (P3,000,000 – P1,000,000) 2,000,000
Gross estate P27,000,000

24. Amount withdrawn P1,000,000


Rate 6%
Final withholding tax P 60,000

Page 13 of 17 0915-2303213  [email protected]


TAXATION
ReSA Batch 43 - May 2022 CPALE Batch
23 April 2022  8:00 AM to 11:00 AM TAX Final Pre-Board Exam

25. Value to take P 650,000


Less: Mortgage paid -
Initial basis 650,000
Less: Proportional deduction (650,000/1,800,000 x 300,000) 108,333
Final basis 541,667
Rate 20%
Vanishing deduction P 108,333

26. Reference: Section 2, Revenue Regulations No. 30-2002

Gross estate P44,000,000


Less: Deductions 2,000,000
Estate after deductions 42,000,000
Less: Family home (10,000,000)
Standard deduction ( 5,000,000)
Taxable net estate P27,000,000

28. 25 Personal property (From Part V Schedule A) P -


26 Real property (From Part V Schedule B) 180,000
27 Total gifts in this return 180,000
Less: Deductions allowed (Sum of items 28 to 32)
28 Mortgage assumed by the donee 40,000
34 Total net gifts in this return (Item 27 less Item 28) 140,000
35 Add: Total net gifts during the calendar year (Item 36 of return
previously filed with the year)
June 1, 2018 150,000
July 10, 2018 400,000
September 30, 2018 450,000 1,000,000
36 Total net gifts (Sum of Items 34 and 35) 1,140,000
37 Less: Exempt gift 250,000
38/14 Total net gifts subject to tax (Item 36 less Item 37) P 890,000

29. 14 Total net gifts subject to tax (From Part IV Item 38) P890,000
15 Applicable donor’s tax rate 6%
16 Total donor’s tax due (Item 14 x Item 15) 53,400
17 Less: Tax credit/payments
17A Payments for prior gifts during the taxable year
(1,000,000 – 250,000 x 6%) 45,000
18 Tax payable/Overpayment ((Item 16 less Item 17A) P 8,400

30. Alpha numeric


tax code (ATC) Taxable amount Rate Tax due
PT 060 P1,500,000 2% P30,000
PT 010 750,000 1% 7,500
PT 120 800,000 2% 16,000
Total tax due P53,500

31. Total tax due P53,500


Surcharge rate 50%
Surcharge for willful neglect P26,750

32. Interest on delinquency (April 25 to July 25)


(53,500 x 12% x 91/365) P1,600

Page 14 of 17 0915-2303213  [email protected]


TAXATION
ReSA Batch 43 - May 2022 CPALE Batch
23 April 2022  8:00 AM to 11:00 AM TAX Final Pre-Board Exam

33. 14 Total tax due (From schedule 1 item 7) P53,500


Less: Tax credit/payment (attach proof)
15 Creditable percentage tax per BIR Form No. 2307
7,500
Tax still payable 46,000
Add: Penalties
20 Surcharge 26,750
21 Interest 1,600
Total amount payable P74,350

34. Sale of four (4) residential lots at P1,500,000 each to four (4) individual
buyers for socialized housing P6,000,000

35. Gross receipts, trucking business P1,500,000


Gross receipts, lease of residential units 2,500,000
Gross receipts, practice of accountancy 1,000,000
Sale of three (3) parking spaces at P500,000 each 1,500,000
Vatable gross amount P6,500,000

36. Sell directly to


Sell during IPO buyer after IPO
Gross selling price P1,200,000 P1,400,000
Less: Cost - 1,000,000
Gross selling price/net capital gain P1,200,000 P 400,000
Rate 4% 15%
Tax due P 48,000 P 60,000

Ratio = 100,000 shares/1,000,000 shares 10%

37. Initial payment/selling price (300,000/1,500,000 = 20%) does not exceed 25%.

38. 300,000/1,500,000 x 2,000,000 = 400,000 x 12% = 48,000

39. Transfers exempt from donor’s tax under Section 101(a) and (b) of the Tax Code shall be
exempt from the tax imposed under this Section.

43. Fair market value P10,000,000


Multiplied by assessment level 60%
Assessed Value 6,000,000
Tax rate 2%
Basic real property tax P 120,000

46. Conditions:
a. That such entities present proof as certified by the Department of Labor and
Employment that disabled persons are under their employ:
b. The disabled employee is accredited with the Department of Labor and Employment
and the Department of Health as to his disability, skills and qualifications.

47. BMBE refers to any business entity or enterprise engaged in the production, processing or
manufacturing of products or commodities, including agro-processing, trading and services,
whose total assets including those arising from loans but exclusive of the land on which the
particular business entity's office, plant and equipment are situated, shall not be more than
Three Million Pesos (P3,000,000.00).

“Services" shall exclude those rendered by any one, who is duly licensed by the government
after having passed a government licensure examination, in connection with the exercise of
one's profession.

Page 15 of 17 0915-2303213  [email protected]


TAXATION
ReSA Batch 43 - May 2022 CPALE Batch
23 April 2022  8:00 AM to 11:00 AM TAX Final Pre-Board Exam

49. Documentary stamp tax (P2,000,000/200 x P2) = P20,000

50. If the return is amended substantially different from the original return, the three (3) year
prescriptive period shall be counted from the filing of the amended return.

There is substantial amendment when a new return is filed declaring more losses, which
can only be done either:
1) In reducing gross income, or
2) In increasing the items of deductions claimed.

If the amendment is minimal, the counting of the prescriptive period is still the original
period. (CIR vs. Phoenix Assurance, Inc.’ G.R. L-19727, May 20, 1965, 14 SCRA 52

52. P68,000 x 50% P34,000

53. P34,000 divided by 65% x 35 P18,308

55. Selling price P750,000


Less: Cost 500,000
Capital gain P250,000
Capital gains tax 250,000 x 15% P 37,500

56. Retirement benefits received from his previous employer that maintained
a reasonable private pension plan (he served the company for 12 years)
P250,000
Lump sum benefits received from SSS 200,000
Interest income from bonds with a maturity period of 7 years 20,000
Interest on long term deposits with maturity period of 5 years 30,000
Total P500,000

57. Prize in a sports tournament sponsored by a group of businessmen


promoting health products (P50,000 X 20%) P 10,000
Share in the net income of a business partnership (P100,000 X 10%) 10,000
Philippine Lotto winnings (500,000 x 20%) 100,000
Total P120,000

58. Gross professional income, net of 15% withholding tax (P680,000/85%) P800,000
Less: Professional expenses 300,000
Net income from operation 500,000
Add: Non-operating income
Gain from sale of bonds (maturity period is 4 years) 10,000
Taxable income P510,000

60. Regular income tax P 57,500


Final tax on passive income 120,000
Total P177,500

Note: The income tax expenses include the regular income tax, final tax on passive income
and capital gains tax, if any.

61. Real and personal properties (14,000,000 + 2,000,000) P16,000,000


Family home 30,000,000
Gross estate P46,000,000

62. Unpaid real estate tax P2,000,000


Loss of a personal property due to theft 500,000
Transfer for public use 500,000
Total P3,000,000

Page 16 of 17 0915-2303213  [email protected]


TAXATION
ReSA Batch 43 - May 2022 CPALE Batch
23 April 2022  8:00 AM to 11:00 AM TAX Final Pre-Board Exam

63. Family home P 10,000,000


Standard deduction 5,000,000
Total P 15,000,000

64. 34 Gross estate P 46,000,000


35 Less: Ordinary deductions 3,000,000
36 Estate after ordinary deductions 43,000,000
37D Less: Total special deductions 15,000,000
40 Net taxable estate P 28,000,000

65. 36 Total net gifts P 500,000


37 Less: Exempt gift 250,000
38 Total net gifts subject to tax 250,000
15 Applicable donor’s tax rate 6%
16 Total donor’s tax due P 15,000

66. Gross sales P 150,000,000


Less: Sales returns and allowances 10,000,000
Sales discount 4,500,000 14,500,000
Net sales P 135,500,000
Tax rate 12%
Output tax P 16,260,000

67. Reference: Section 3 B, Revenue Regulations No. 12-2012


Only one vehicle for land transport is allowed for the use of an official or employee, the
value of which should not exceed P2,400,000.

68. Reference: Section 3 C, D and E, Revenue Regulations No. 12-2012

C. No depreciation shall be allowed for yachts, helicopters, airplanes and/or aircrafts, and
land vehicles which exceed the threshold amount, unless the taxpayer’s main line of
business is transport operations or lease of transportation equipment and the vehicles
purchased are used in said operations.
D. All maintenance expenses on account of non-depreciable vehicles for taxation
purposes are disallowed in its entirety.
E. The input taxes on the purchase of non-depreciable vehicles and all input taxes on
maintenance expenses are likewise disallowed for taxation purposes.

69. Office equipment (1,100,000 x 12% divided 60 x 3) P 6,600


Purchase of goods for sale (30,000,000 12%) 3,600,000
Operating expenses (5,000,000 x 40% x 12%) 240,000
Office supplies purchased (900,000 x 12%) 108,000
Total P3,954,600

70. Output tax P16,260,000


Less: Input taxes 3,954,600
VAT payable 12,305,400
Less: VAT payments for January and February 7,000,000
Tax payable P5,305,400

Page 17 of 17 0915-2303213  [email protected]


CPAR
FIRST
PREBOARD
CPA Review School of the Philippines
Manila
First Preboard Examination February 2022
TAXATION De Vera / Llamado

1. A tax is classified as ____________________, if the same is based on the value of the


property subject matter of the tax, and therefore may require the expert judgment of assessors
and appraisers.

a. ad valorem
b. specific
c. direct
d. indirect
e. None of the above

2. Statement 1: One of the sumptuary/secondary purposes of the power to tax is to raise


revenues for governmental expenses.
Statement 2: A revenue bill may originate from the Senate as long as the House of
Representatives is given the opportunity to study the bill upon three full readings
of the same.
a. Both are true.
b. Both are false.
c. Only Statement 1 is true.
d. Only Statement 2 is true.

3. A non-resident alien not engaged in trade or business (NRANETB) shall generally be subject
to a 25% final tax on gross income received from all sources within the Philippines.
However, a NRANETB shall be exempt from tax on:
a. Philippine Charity and Sweepstakes Office (PCSO) winnings less than ₱10,000.
b. De minimis prizes of less than ₱10,000.
c. Interest income paid by a depositary bank under the foreign currency deposit
system.
d. Interest income from a long-term deposit or investment certificate issued by a bank in the
Philippines, and held for a period exceeding 5 years.
4-9) Christina, single, Filipino with 1 dependent child, received in taxable year 2021 the
following professional fees as an independent consultant:

Monthly Fees Less 5% CWT Net Monthly Fees

₱240,000 ₱12,000 ₱228,000

The monthly cost of her professional services amounted to ₱20,000, gross of the CWT.
The rental expenses related to her consultancy business amounted to:

Monthly rental Monthly rental expense


CWT
expense paid

Office space ₱30,000 ₱1,500 ₱28,500

Computer ₱5,000 None ₱5,000

Note: The lessor of the office space is Christina’s brother, Christopher.


Her car fuel expenses amounted to ₱10,000 per month. She used the car for her consultancy
business 40% of the time.
She is not VAT-registered and instead pays 3% OPT on her gross receipts under Section 116
of the Tax Code.
She has excess tax credits from the prior year of ₱15,500.
In her 1st Quarterly ITR, she chose to be taxed under the graduated rates with OSD as
method of deduction.

4. Calculate her tax payable in the 2nd Quarterly ITR.


a. ₱0
b. ₱75,200
c. ₱61,700
d. None of the above.

5. For her annual ITR, she chose to file BIR Form No. 1701A (for Individuals Earning Income
Purely from Business/Profession). What is the correct amount to be placed on Part IVC
Line 58 of BIR Form No. 1701A?

a. ₱108,000
b. ₱144,000
c. ₱155,300
d. None of the above.
6. Calculate her tax payable in the Annual ITR.
a. ₱93,600
b. ₱334,400
c. ₱78,100
d. None of the above.

7. If Christina, in her 1st Quarterly ITR, chose to be taxed under the graduated rates with
itemized deduction as method of deduction, what would be her tax payable in her 3rd
Quarterly ITR?
a. ₱124,920
b. ₱120,420
c. ₱147,420
d. None of the above.

8. If Christina chose to be taxed under the graduated rates with itemized deduction as method
of deduction, what BIR Form should she file as her annual ITR?
a. BIR Form No. 1701A
b. BIR Form No. 1701
c. Either (A) or (B)
d. None of the above.

9. Calculate her tax payable in the Annual ITR if she chose to be taxed under the graduated
rates with itemized deduction as method of deduction.
a. ₱127,832
b. ₱162,032
c. ₱156,332
d. None of the above.
10 -16) Carissa, resident citizen, single, 45 years of age, is an employee of GMA Inc. from which
she received the following employment income:

Less SSS premiums


Gross pay Less CWT Net Pay
and union dues

₱850,000 ₱60,000 ₱6,000 ₱784,000

The amount she received from GMA is inclusive of 13th Month Pay and other benefits of
₱165,000.
Carissa, aside from her employment, is also registered with the BIR as engaged in real estate
leasing and management consultancy.
She has the following financial information in the current taxable year:
(a) Interest income on receivable from her mother ₱ 25,000
(b) Interest expense on loan owed to grandfather 10,000
(c) Royalty income from book, net of FT 15,000
(d) Loss on sale of rental property to brother 350,000
(e) Gain on sale of personal car bought in 2018 90,000
(f) Rental receipts from real property, gross of 5% CWT 600,000
(g) Cost of revenues related real property leasing 100,000
(h) Distributive share in net income of a GPP, net of 15% WT 187,000
(i) Dividend received from Microsoft Corporation with
headquarters in Seattle, Washington, USA 75,000
(j) Dividend from Jollibee Corporation 5,000
(k) Expenses for dancing lessons with Zeus Cohen 50,000
Because she is a mixed income earner and is not qualified for the 8% tax option, she will be
filing BIR Form No. 1701 as her annual ITR.

10. What is the correct amount of Non-Taxable/Exempt Compensation to be placed on Line 5 in


Schedule 2 (Part V) in page 2 of BIR Form No. 1701?
a. ₱60,000
b. ₱90,000
c. ₱96,000
d. None of the above.
11. What is the correct amount of Taxable Compensation Income to be placed on Line 6 in
Schedule 2 (Part V) in page 2 of BIR Form No. 1701?
a. ₱790,000
b. ₱754,000
c. ₱760,000
d. None of the above.

12. What is the correct amount of Net Sales/Revenues/Receipts/Fees to be placed on Line 10 in


Schedule 3.A (Part V) in page 2 of BIR Form No. 1701?
a. ₱ 600,000
b. ₱ 850,000
c. ₱1,354,000
d. None of the above.

13. What is the correct amount of Total Allowable Itemized Deductions of to be placed on Line
16 in Schedule 3.A (Part V) in page 2 of BIR Form No. 1701?
a. ₱410,000
b. ₱360,000
c. ₱350,000
d. None of the above.

14. What is the correct amount of Optional Standard Deduction to be placed on Line 17 in
Schedule 3.A (Part V) in page 2 of BIR Form No. 1701?
a. ₱240,000
b. ₱250,000
c. ₱580,000
d. None of the above.

15. What is the correct amount of Total Other Non-Operating Income to be placed on Line 22 in
Schedule 3.A (Part V) in page 2 of BIR Form No. 1701?
a. ₱340,000
b. ₱295,000
c. ₱365,000
d. None of the above.

16. Calculate her tax payable in the Annual ITR.


a. ₱252,700
b. ₱285,700
c. ₱315,700
d. None of the above.
17. The following are the features of the 8% optional income tax in the ITR, except:
a. The qualified individual taxpayer is not required to file his financial statements with
his Annual ITR.
b. The qualified taxpayer is still required to comply with bookkeeping and invoicing
requirements.
c. The qualified taxpayer, who is also a partner in a general professional
partnership, must signify his choice of the 8% tax rate in the first quarterly
ITR.
d. The qualified taxpayer is exempt from paying the 3% OPT under Section 116 of the
Tax Code.

18. DND Corporation leased a residential house for the use of its CEO. The monthly rent
agreed upon under the lease contract was ₱156,750, net of the 5% CWT. Compute the
quarterly FBT to be paid by DND Corporation.
a. ₱126,606
b. ₱126,923
c. ₱133,269
d. None of the above

19. In number 18, what is the total deduction allowable to DND Corporation for the quarter?
a. ₱628,269
b. ₱380,769
c. ₱513,038
d. None of the above

20. Lee Min Ho, a non-resident alien, came to the Philippines to perform project management
services for Green View Corporation, a domestic corporation engaged in mining. The
contract fee for Lee’s services was ₱600,000. He stayed in the Manila Mandaring Hotel for
10 days, and upon completion of the service, left for Korea. Green View paid for Lee’s
hotel bills in the amount of ₱75,000.
Compute the final taxes to be remitted by Green View to the BIR.
a. ₱168,750
b. ₱150,000
c. ₱175,000
d. None of the above
21. Which of the following benefits received by an employee may be included in the employee’s
ITR and be subject to the regular income tax?
(1) Use of employer’s Mercedes Benz by CEO for to attend business meeting
(2) Uniform and clothing allowance of ₱6,000 per year
(3) Payment of tuition fee of eldest child of a rank-and-file employee
(4) Housing provided to a rank-and-file employee
a. (1) and (2)
b. (2) and (3)
c. (3) and (4)
d. None of the above

22. Alberto fell in love with Beatrice. However, Alberto’s father disapproved of the
relationship. Against his father’s wishes, Alberto and Beatrice eloped, got married, and had
4 children.
In spite of his disapproval of his son’s marriage, Alberto’s father created a revocable trust
with Atty. Salvador Sanchez as trustee. Alberto’s father transferred a 10-door apartment
where rent income of P190,000 per month (net of 5% withholding tax) was received by the
trust with an expense of P980,000 during the year. 30% of the net income was given to
Alberto.
Determine the income tax still due from the trust.
a. ₱170,000
b. ₱166,500
c. ₱156,680
d. None of the above

23-24) A Co. and B Co., domestic corporations, both in the construction business, formed a
joint venture (JV) to construct apartment complexes in Laguna, with an agreed equal
sharing in net income. Since both corporations were both licensed by the Philippine
Contractors’ Accreditation Board (PCAB), the management of the JV deemed it
unnecessary to get a PCAB license for itself.
Data on income and expenses for the calendar year 2021 show:

Joint Venture A Co. B Co.


Gross Income P 80,000,000 P 2,000,000 P 3,000,000
Expenses 60,000,000 1,200,000 2,000,000

23. Determine the income tax due of the JV for 2021:


a. ₱5,000,000
b. ₱4,000,000
c. ₱4,500,000
d. None of the above
24. The income tax due of A Co for 2021:
a. ₱200,000
b. ₱700,000
c. ₱ 75,000
d. None of the above

25. Who among the following individuals is subject to the preferential 15% tax rate on gross
compensation income under the Tax Code as amended by the TRAIN?

i) Foreign managers in Philippine regional area headquarters of multinational companies;


ii) Aliens employed with foreign petroleum service contractors engaged in petroleum
operations in the Philippines;
iii) Aliens employed with offshore banking units in the Philippines; or
iv) Filipinos occupying non-managerial technical positions in Philippine regional operating
headquarters of multinational companies,
a. All except (iv).
b. All of them.
c. Only (i) and (ii).
d. None of them.

26-29) Ginger Licaros, single, resident citizen, had the following financial information in 2021:
Compensation income, Philippines, net of CWT 2,500,000
Business income, Philippines 3,895,000
Business income, abroad 1,300,000
Business expenses, Philippines 878,000
Business expenses, abroad 340,000
Dividend income from a domestic corporation 40,000
Dividend income from a foreign corporation* 30,000
Interest income, BPI Manila 50,000
Interest income, Citibank New York 25,000
Interest income, BPI FCDU 34,000
Royalty income, copyright (book) 450,000
Royalty income, patent 1,350,000
Raffle draw winnings 25,000
Philippine lottery winnings 35,000
CWT 194,750
Taxes withheld from compensation income 220,000
Household expenses 2,500,000

*Note: 2/3 of the foreign corporation’s income in the last 3 years was earned in the
Philippines.
26. What is the total amount of final taxes on Ginger’s income in 2021?
a. ₱346,100
b. ₱391,100
c. ₱339,100
d. None of the above

27. Compute Ginger’s income tax payable for CY 2021.


a. ₱1,595,890
b. ₱1,587,810
c. ₱1,592,690
d. None of the above

28. What is the total amount of final taxes on Ginger’s income in 2021 if she was a non-resident
alien engaged in trade or business (NRAETB)?
a. ₱345,000
b. ₱350,100
c. ₱341,000
d. None of the above

29. Compute Ginger’s income tax payable for CY 2021 if she was a non-resident alien engaged
in trade or business (NRAETB).
a. ₱1,277,490
b. ₱1,274,290
c. ₱1,280,690
d. None of the above

30. The Municipality of Oslob in the Province of Cebu conducts Butanding Watching Activity
operations for visiting tourists. It also provides guides to assist the tourists. For such
services, the municipality charges and collects fees from the tourists. Are these fees
taxable?

(a) No. The municipality is a political subdivision of the National Government and thus
exempt from taxes.
(b) Yes. The income derived by the municipality is not in the exercise of its essential
government functions, but pursuant to its proprietary function. The income
derived therefrom is subject to both income tax and VAT. (BIR Ruling No. 471-
2018, March 13, 2018).
(c) It depends whether the fees received shall be used for the benefit of the residents of the
municipality.
(d) None of the above.
31. The board of directors of CPAR noticed the tremendous performance of Christian Cruz who
teaches taxation. The board voted to take a life insurance policy covering Christian’s life.
Statement 1: The premium paid by CPAR on the abovementioned life insurance is taxable
income to Christian Cruz, whoever is the designated beneficiary of the policy.
Statement 2: If the designated beneficiary of the policy is CPAR, the premiums paid
thereon are deductible by CPAR.
a) Statement 1 is true. c) Both statements are false.
b) Statement 2 is true. d) Both statements are true.

32. Statement 1: Property dividend received by a resident foreign corporation from a non-
resident foreign corporation is not subject to final tax but is includible in the income tax
return of the resident foreign corporation.
Statement 2: Dividends received by individuals from domestic corporations are subject to
creditable withholding taxes.
a) Statement 1 is true. c) Both statements are false.
b) Statement 2 is true. d) Both statements are true.

33. The following are the requirements in the Tax Code for retirement benefits to be exempt,
except:
a) There must be a reasonable private benefit plan.
b) The plan must be approved by the BIR.
c) The retiree must have been in the service of the employer for at least 10 years.
d) The retiree must at least be 55 years old at the time of retirement.
e) The exemption shall be availed of only once.

34-35) Fernando Macasaet, a CPA, is employed as an accounting professor in a university. On


the side, he also provides accounting services for select clients. One of these clients was so
impressed with him that he was made a consultant for one of his companies engaged in agri-
business.
For 2021, he received the following incomes:
Salary from university, net of CWT of ₱45,000 ₱ 268,000
13th month pay received from the university 95,000
Monetized unused vacation leave credits (10 days) 50,000
Employee longevity award with monetary value of 10,000
Fees received as consultant, gross of 10% CWT 80,000
Professional fees from clients, net of 15% CWT 850,000
Damages won in a defamation suit filed against ex-wife 8,000,000
Capital gains from sale of jewelry bought in 2000 100,000
34. What is his gross taxable compensation income for 2018?
a) ₱318,000 c) ₱315,000
b) ₱313,000 d) None of the above

35. In number 34, what is his total gross (taxable) income (in the ITR)?

a) ₱1,448,000 c) ₱1,285,000
b) ₱1,498,000 d) None of the above

Atty. Idol Gadon with total assets of ₱60,000,000, excluding the land on which its business is
situated, has the following data for the year 2021:

Gross Receipts, Philippines P1,000,000


Gross Receipts, USA 500,000
Gross Receipts, Japan 500,000
Expenses, Philippines 300,000
Expenses, USA 200,000
Expenses, Japan 100,000

Other Income:
Dividend from San Miguel Corp. 70,000
Dividend from Ford Motors, USA 120,000
Gain, sale of San Miguel shares directly to buyer 150,000
Royalties, Philippines 50,000
Royalties, USA 100,000
Interest income (other than from bank deposit) 60,000
Rent, land in USA 250,000
Other rental income (Phils.) 100,000
Prize, contest in Manila 200,000
Interest income ($ deposit in BDO) 50,000

36. If Atty. Gadon is Resident Citizen, how much is his income tax?
a. P486,000 c. P499,600
b. P411,500 d. P563,600

37. How much is his final taxes on passive income?


a. P64,500 c. P57,500
b. P24,500 d. P87,000
38. How much is his capital gains tax?
a. P10,000 c. P22,500
b. P15,000 d. 30,000
39. How much is his total tax liability?
a. P486,600 c. P661,100
b. P586,600 d. P579,100

40. If he opted for Gross Receipt Tax, how much is his tax payable?
a. P0 c. P210,400
b. P190,400 d. P142,400

41. If Atty. Gadon is Non-Resident Alien Engaged in Business, how much is his income tax?
a. P186,000 c. P148,000
b. P148,600 d. P163,600

42. How much is his final taxes on passive income?


a. P64,000 c. P57,500
b. P64,500 d. P87,000

43. How much is his capital gains tax?


a. P10,000 c. P22,500
b. P15,000 d. 30,000

44. How much is his total tax liability?


a. P235,000 c. P235,600
b. P234,500 d. P228,00

45. If he opted for Gross Receipt Tax, how much is his tax payable?
a. P0 c. P92,800
b. P72,800 d. P68,800

46. If Atty. Gadon is Non-Resident Alien not Engaged in Business, how much is his income
tax?
a. P386,000 c. P348,000
b. P348,600 d. P370,000

47. How much is his final taxes on passive income?


a. P64,000 c. P57,500
b. P64,500 d. none
48. How much is his capital gains tax?
a. P10,000 c. P22,500
b. P15,000 d. 30,000

49. How much is his total tax liability?


a. P335,000 c. P335,600
b. P334,500 d. P392,500

50. If he opted for Gross Receipt Tax, how much is his tax payable?
a. none c. P92,800
b. P72,800 d. P68,800
RESIDENT CITIZEN

ITR
Gross income: Philippines 1,000,000
USA 500,000
Japan 500,000
Gross income from operations 2,000,000
Add: Other taxable income not subject
to final tax:
Divided from Ford 120,000
Royalties (USA) 100,000
Interest income (not from
bank deposit) 60,000
Rentals (USA) 250,000
Rentals (Phils.) 100,000
Prize - Manila 630,000
Total Gross Income 2,630,000
Less: Allowable deductions:
Expenses: Phils. 300,000
USA 200,000
Japan 100,000 (600,000)
Net taxable income 2,030,000

Tax Due 499,600

Final Tax
(a)Dividends from SMC
70,000
10% 7,000

(b) Royalties (Phils.) 50,000


20% 10,000

40,000
(c)Prizes
200,000
x 20%

(d) Interest ($ deposit - BDO) 5


50,000
15% 7,500

Total Final Tax from Passive


income 64,500
CAPITAL GAINS TAX

Gains from sale of San Miguel Shares direct to buyer

150,000 x 15% 22,500

GROSS RECEIPT TAX 8% X (2,630,000-250,000) 190,400


NON RESIDENT ALIEN ENGAGED IN BUSINESS

ITR
Gross income from operations: Philippines 1,000,000
Add: Other taxable income not subject
to final tax:
Interest income (not from
bank deposit) 60,000
Rentals (Phils.) 100,000
Prize - Manila 160,000
Total Gross Income 1,160,000
Less: Allowable deductions:
Expenses: Phils. (300,000)
Net taxable income 860,000

Tax Due 148,000

CAPITAL GAINS TAX


(a) Gain from sale of SMC shares
directly to buyers 150,000
15% 22,500

FINAL TAX ON PASSIVE


INCOME
(a) Royalties (Phils.) 50,000
20% 10,000

(b) Interest ($ deposit - BDO) 50,000


exempt 0

70,000
X
(c) Dividends SMC 20% 14,000

(d)Prizes 40,000
200,000
X 20%

Total Final taxes 64,000

GROSS RECEIPTS TAX

(1,160,000-250,000)x 8% = 72,800
NON RESIDENT ALIEN
NOT ENGAGED IN
BUSINESS

(a) Income subject to 25% FT:


Gross income (Phils.) 1,000,000
Dividends 70,000
Royalties, Phils 50,000
Interest income 60,000
Rental income (Phils.) 100,000
Prize (Manila) 200,000
1,480,000
25% 370,000

(b) Final tax Interest BDO 50,000


exempt 0

(c) Capital Gain Tax from sale of SMC shares


directly to buyers 150,000
15% 22,500

Total Tax Liability 392,500

GROSS RECEIPTS TAX none, disqualified


51. Statement 1: A person may refuse to pay a tax on the ground that he receives no personal
benefit from it.
Statement 2: A taxpayer has a right to question illegal expenditures of public funds.
a. if only the first statement is correct.
b. if only the second statement is correct.
c. if both statements are correct.
d. if both statements are incorrect.

52. Statement 1: One of the essential characteristics of a tax is it is unlimited in amount.


Statement 2: A tax is generally unlimited because it is based on the needs of the state.
a. If only the first statement is correct.
b. If only the second statement is correct.
c. If both statements are correct.
d. If both statements are incorrect.

53. Statement 1: A License Fee is a charge imposed under the police power of the state.
Statement 2: Penalty is imposed by the state only.
a. If only the first statement is correct.
b. If only the second statement is correct.
c. If both statements are correct.
d. If both statements are incorrect.

54. Statement 1: Imposition of taxes is a legislative act.


Statement 2: Collection of taxes is an administrative act.
a. If only the first statement is correct.
b. If only the second statement is correct.
c. If both statements are correct.
d. If both statements are incorrect.

55. Statement 1: A state has the power to tax even if not granted by the Constitution.
Statement 2: A state cannot exercise police power if not granted by the Constitution.
a. If only the first statement is correct.
b. If only the second statement is correct.
c. If both statements are correct.
d. If both statements are incorrect.

56. Statement 1: There can only be a tax if there is a law imposing the tax.
Statement 2: The power to tax is inherent.
a. If only the first statement is correct.
b. If only the second statement is correct.
c. If both statements are correct.
d. If both statements are incorrect.
57. Statement 1: A tax is based on law while debt is based on contract.
Statement 2: A tax is also a custom’s duty.
a. If only the first statement is correct.
b. If only the second statement is correct.
c. If both statements are correct.
d. If both statements are incorrect.

58. Statement 1: A person cannot be imprisoned for non-payment of property tax.


Statement 2: A law may be passed violating uniformity of taxation.
a. If only the first statement is correct.
b. If only the second statement is correct.
c. If both statements are correct.
d. If both statements are incorrect.

59. Statement 1: The RDO is known as the alter ego of the BIR Commissioner.
Statement 2: The BIR Commissioner is directly under the President’s Office.
a. If only the first statement is correct.
b. If only the second statement is correct.
c. If both statements are correct.
d. If both statements are incorrect.

60. Statement 1: Provisions in the Philippine Constitution on Taxation are grants of power.
Statement 2: Due process of law in taxation in the Constitution is a grant of power.
a. If only the first statement is correct.
b. If only the second statement is correct.
c. If both statements are correct.
d. If both statements are incorrect.

61. Statement 1: An excise tax is also called privilege tax.


Statement 2: A tax which is neither personal nor property, is an excise tax.
a. If only the first statement is correct.
b. If only the second statement is correct.
c. If both statements are correct.
d. If both statements are incorrect.

62. Statement 1: Estate tax is a proportional tax.


Statement 2: A progressive tax is a tax, the rate of which is directly proportional to tax base.
a. If only the first statement is correct.
b. If only the second statement is correct.
c. If both statements are correct.
d. If both statements are incorrect.
63. Statement 1: Taxation without representation is not tyranny.
Statement 2: In the exercise of taxation, the state can tax anything at anytime and at any
amount.
a. If only the first statement is correct.
b. If only the second statement is correct.
c. If both statements are correct.
d. If both statements are incorrect.

64. Statement 1: Taxation and power of Eminent Domain may be exercised simultaneously.
Statement 2: Taxation and Police Power may be exercised simultaneously.
a. If only the first statement is correct.
b. If only the second statement is correct.
c. If both statements are correct.
d. If both statements are incorrect.

65. Statement 1: All of our tax laws are statutory laws.


Statement 2: The non-impairment clause is a statutory law on taxation.
a. If only the first statement is correct.
b. If only the second statement is correct.
c. If both statements are correct.
d. If both statements are incorrect.

66. Statement 1: Non-payment of tax makes the business illegal.


Statement 2: The non-payment of license fee makes the business illegal.
a. If only the first statement is correct.
b. If only the second statement is correct.
c. If both statements are correct.
d. If both statements are incorrect.

67. Statement 1: Margin fee is a tax.


Statement 2: Custom’s duty is a tax.
a. If only the first statement is correct.
b. If only the second statement is correct.
c. If both statements are correct.
d. If both statements are incorrect.

68. Statement 1: Territoriality is one of the constitutional limitations on the power of taxation.
Statement 2: International comity is an inherent limitation in taxation.
a. If only the first statement is correct.
b. If only the second statement is correct.
c. If both statements are correct.
d. If both statements are incorrect.
69. Statement 1: Taxes may be used for sectarian purposes if allowed by an ordinance.
Statement 2: The President can refuse to implement a tax law if it appears to be
unconstitutional.
a. If only the first statement is correct.
b. If only the second statement is correct.
c. If both statements are correct.
d. If both statements are incorrect.

70. Statement 1: There can be double taxation in the Philippines.


Statement 2: Double taxation is illegal if it violates the uniformity of taxation.
a. If only the first statement is correct.
b. If only the second statement is correct.
c. If both statements are correct.
d. If both statements are incorrect.

Beginning 1 January 2018:

Taxable Income (PHP)


Tax on column 1 (PHP) Tax on excess (%)
Over Not Over

0 250,000 - 0

250,000 400,000 - 20

400,000 800,000 30,000 25

800,000 2,000,000 130,000 30

2,000,000 8,000,000 490,000 32

8,000,000 2,410,000 35
DC RFC NRFC
Dividend from San Miguel Corp. Exempt Exempt FT (15%)
Dividend from Ford Motors (USA) ITR Exempt Exempt
Gain from sale of San Miguel shares directly to buyer FT (15%) FT(15%) FT(15%)
Royalties (Phils.) FT(20%) FT(20%) FT(25%)
Royalties (USA) ITR Exempt Exempt
Interest income (other than from banks) ITR ITR FT(25%)
Rentals of land (USA) ITR Exempt Exempt
Other rental income (Phils) ITR ITR FT(25%)
Prize, contest in Manila ITR ITR FT(25%)
Interest income ($ Deposit in BDO) FT(15%) FT(15%) Exempt
CPAR
FINAL
PREBOARD
CPA Review School of the Philippines
Manila

FINAL PRE-BOARD EXAMINATION


TAXATION

Instructions: Choose the BEST answer for each of the following items. Mark only one answer for
each item on the Special Answer Sheet provided. Strictly no erasure allowed.

1. Statement 1: The CIR is authorized to inquire into the bank deposit of a taxpayer who has filed
an application for compromise of his tax liability by reason of financial incapacity to pay.
Statement 2: The CIR is authorized to look into the bank deposit of a decedent to determine his
gross estate.
a. True, true c. False, true
b. True, false d. False, false

2. Atty. Idol Gadon, failed to report and file estate tax return of his late father, Ferdinand Gadon
who died September 19, 2020. The BIR assess Atty. Gadon amounting to Php23 billion in
October 30, 2021. Atty. Idol Gadon, protested the assessment with the Supreme Court for
failure of the BIR to observe due process. Which of the following is correct?
a. A protest should be filed by the taxpayer within the BIR, otherwise the assessment
becomes final and can no longer be questioned in court.
b. A protest may be filed by the taxpayer any time before the BIR collects the tax.
c. The assessment should be made by the BIR within five years from the filing of the return
d. The Supreme Court may reverse the assessment and shall include only tax proper.

3. Which of the following is not a requisite to toll the collection of taxes to be made by the BIR,
upon proof of the following to the Court of Tax Appeals
a. It will jeopardize the interest of the taxpayer
b. It will jeopardize the interest of the government
c. Filing of a bond for at least double the amount of the tax assessed
d. The case is not dilatory

4. ABC Corporation won a tax refund case with the Supreme Court for P50 Million. Upon
execution of the judgment and when trying to get the Tax Credit Certificates (TCC) representing
the refund, the Bureau of Internal Revenue (BIR) refused to issue the TCC on the basis of the
fact that the corporation is under audit by the BIR and it has a potential tax liability. Is there a
valid justification for the BIR to withhold the issuance of the TCC?
a. Yes, power of the BIR to make assessment and collection of all national internal revenue
taxes, fees and charges.
b. Yes, power of the BIR to enforce all national taxes, fees and charges including
forfeitures, penalties and fines connected therewith.
c. No, BIR is only in the assessment stage and not yet in the collection stage.
d. No, the BIR is inferior to the courts.
5. On June 2, 2018 Global Bank received a final notice of assessment from the BIR for deficiency
documentary stamp tax in the amount of P5million. On June 30, 2018, Global Bank filed a
request for reconsideration with the Commissioner of Internal Revenue. The Commissioner
denied the request for reconsideration only on March 30, 2022, at the same time serving on
Global Bank a warrant of distraint to collect the deficiency tax. What are the remedies of Global
Bank?
a. Appeal to the Court of Tax of Appeals within 30 days from receipt of the denial.
b. File for injunctive relief to restrain the distraint.
c. Enter into a compromise agreement with the Commissioner of Internal Revenue.
d. All of the above.

6. Gerry was being prosecuted by the BIR for failure to pay his income tax liability for Calendar
Year 2018 despite several demands by the BIR in 2021. The information was filed with the
Regional Trial Court only last January 2022. What are the remedies of Gerry?
a. Gerry should file a motion to quash the Information on the ground of prescription.
b. Gerry should answer the Information.
c. Gerry should file a protest with the Commissioner of BIR.
d. Gerry should appeal to the CTA.

7. A’s income tax liability for 2020 was P75,000. She filed the return and paid the total amount
due on July 15, 2021 but not with the proper internal revenue district office. The amount payable
by A is
a. P96,000 c. P114,750
b. P116,250 d. P112,500

Tax proper 75,000


Surcharge: 25% wrong venue 18,750
Late filing 18,750
Interest (75,000x12%x3/12mos) 2,250
TOTAL 114,750

8. Jane filed his annual income tax return on May 22, 2016 for income earned during 2015. The
BIR examiner assessed Jane for tax delinquency on October 15, 2017. Jane filed a protest
plus all documentary evidence on November 8, 2017 with the Commissioner of the Internal
Revenue. The Commissioner rendered its unfavorable judgment on May 10, 2018. When is
the last day to appeal the case to the Court of Tax of Appeal?
a. June 8, 2008 c. May 25, 2008
b. June 10, 2008 d. May 22, 2008
9. Commissioner of Bureau of Internal Revenue may abate any internal revenue tax when, except
one:
a. The tax `or any portion thereof appears to be unjustly or excessively assessed
b. The administration and collection costs involved do not justify the collection of the
amount due.
c. Where on the face of the return upon which payment was made such payment appears
clearly to have been erroneously paid.
d. The delinquent account or disputed assessment is one resulting from a jeopardy
assessment.

10. Under the eFPS a return is deemed filed with the BIR after a sequence Number is generated.
What is this sequence Number?
a. Filing Reference Number
b. Confirmation Number
c. Acknowledgement Number
d. Computerized Generated Number

Buenas Corporation, a real estate dealer and lessor, on its first year of operation, has received
a Letter of Authority from the BIR. As a revenue officer, you were assigned to audit the
Corporation’s income tax for taxable year 2018. The audited financial statements show the
Corporation’s income statement as follow:
Revenues:
Rental income 2,800,000
Interest from bank 24,000
Gain on sale of land 500,000
Total 3,324,000
Expenses:
Depreciation 800,000
Salaries and wages 200,000
Taxes and licenses 180,000
Entertainment and representation 40,000
Interest expense 60,000
Office supplies 20,000
Total 1,300,000

Net income before income tax 2,024,000


Income tax expense
450,000
Net income 1,574,000

Income tax due for the year is computed as follows:

Net income before income tax 2,024,000


Less : Interest from bank 24,000
Gain on sale of land 500,000
524,00
Net taxable income 1,500,000
Multiply by: corporate income tax
rate 30%
Income tax expense 450,000

After a detailed examination of the Corporation’s books, you have noted the following:
• Balance sheet as of December 31, 2018 showed an outstanding balance of deferred rent
income amounting to P200,000, representing advance rent payments by tenants.
• Interest from bank is net of 20% final withholding tax
• Gain on sale of land, represents gain on sale of residential lot with selling price, amounting
to P2,000,000
• Taxes and licenses in the Notes to Financial Statements
Showed the following breakdown
• Withholding tax on sale of land, P120,000
• Documentary stamp tax on sale of land, P30,000
• Surcharge and penalties on late payment of tax, P10,000
• Permits and licenses, P20,000
Interest expense includes interest on late payment of tax, amounting to P5,000.

11. How much should be reflected as net sales and revenue in the annual income tax return of the
Corporation?
a. P3,000,000
b. P5,000,000
c. P4,800,000
d. P3,300,000

Rent 2,800,000
ADD: Advance Rent 200,00
Sale of real property 2,000,000
Total REVENUE 5,000,000

12. How much should be the deductible taxes and licenses?


a. P20,000
b. P180,000
c. P30,000
d. P50,000

Documentary Stamp Tax 30,000


ADD: Permits 20,000
TAXES 50,000
13. How is the deductible entertainment and representation expense?
a. P40,000
b. P33,333
c. P38,000
d. P34,000
Actual EAR Ceiling Prevail
2/5 x 40,000 16,000 10,000 10,000
3/5 x 40,000 24,000 30,000 24,000
Allowed EAR 34,000
14. How much is the basic deficiency income tax due?
a. P136,475
b. P252,200
c. P253,770
d. P250,800

Gross Income (5,000,000-1,500,000) 3,500,000


Allowable Deductions (1,300,000-130,000-6,000- (1,154,100)
9900)
Taxable Income 2,345,900
Tax Due 25% 586,475
Tax Credit (450,000)
Tax Payable 136, 475

BTS CORPORATION, is a registered enterprise with the Board of Investments and was
granted an Income Tax Holiday (ITH) Incentives for the first 4 years of operations as a non-
pioneer firm. The Corporation has not applied for an extension of its ITH incentives on its 5th
year 2019. On its 5th year of operations, it reported the following:
Export Sales P120,000,000
Cost of goods manufactured and sold 70,000,000
Operating expenses 20,000,000
Additional information were made available as follows:
➢ Cost of goods manufactured and sold include, imported raw materials with dutiable
value of P1,400,000. Pre-computed customs duties and other charges are as follows:
(a) Customs duties – P200,000
(b) Insurance – P10,000
(c) Arrastre charges – P5,000
(d) Wharfage dues – P7,000.
➢ Included under operating expenses are expenses incurred on the importation of the
said equipment as follows:
(a) Facilitation fee – P100,000
(b) Delivery expense to warehouse after release from Customs P11,200.
➢ Cost of goods manufactured and sold include direct labor skilled and unskilled
workers amounting to P10,000,000 and P20,000,000, respectively. The project meets
the prescribed ration of capital equipment to number of workers set by the BOI.
➢ The Company incurred an accumulated net operating loss on the first 4 years of
operations as follows:
(a) 1st year – P2,000,000
(b) 2nd year – P1,600,000
(c) 3rd year – P800,000
(d) 4th year – P400,000.

15. How much is the deductible direct labor cost?


a. P35,000,000
b. P45,000,000
c. P40,000,000
d. P30,000,000

Total compensation 30,000,000


Preferential 150% 45,000,000

16. How much is the deductible net operating loss carry over?
a. None
b. P2,800,000
c. P400,000
d. P4,800,000

17. How much is the income tax due of the Company for the year?
a. P4,500,000
b. P1,750,000
c. P4,530,000
d. P1,869,000

Gross Income 50,000,000


Allowable Deductions (35,000,000–100,000) 34,900,000
Taxable income 15,100,000
TAX DUE 4,530,000
John and Marcia inherited a commercial lot and building from their parents 2 years ago. In 2018
(current year), the inherited property realized rental income of P300,000 per month, in which
both shared the revenues equally. The co-ownership remained unregistered with the Securities
and Exchange Commission. In addition, Jose and Maria reported their following personal
income and expenses.

➢ John, gross compensation income (inclusive of 13th month and other benefits
amounting to P100,000), P1,300,000

➢ Marcia, net income from her laundry service business (net of cost and expenses of
P600,000), P200,000.

18. How much final withholding income tax should the co-ownership remit representing the
revenue distribution to the co-owners?
a. P360,000
b. P540,000
c. None
d. P180,000

19. How much is the income tax due of John in his annual income tax return, assuming optional
standard deduction is used in determining his expenses?
a. P582,800
b. P431,800
c. P253,000
d. P611,600
Share in the partnership 1,800,000

Taxable portion 60% 1,080,000


ADD: Taxable Compensation 1,210,000
TOTAL TAXABLE INCOME 2,290,000
TAX DUE 582,800

20. How much is the income tax due of Marcia in his annual income tax return, assuming optional
standard deduction is used in determining his expenses.
a. P22,000
b. P322,000
c. P358,000
d. P250,000
Share in the partnership 1,800,000
ADD: taxable business income 800,000
TOTAL 2,600,00
Taxable income 60% 1,560,000
TAX DUE 358,000

21. When is the deadline for filling of the annual income tax return of the co-ownership for taxable
year 2018?
a. April 15, 2018
b. May 15, 2019
c. Not applicable
d. April 15, 2019

ATTY. PAPA, is a practicing lawyer, is also a licensed real estate broker. Both businesses
were registered with the BIR and had the following data made available for year 2018 (months
are gross of withholding tax):
Revenues from his profession 3,000,000
Commission received 600,000
Cost of services 700,000
Business related expenses 400,000
Gain on sale of real property held for investment 500,000
Selling price of real property 2,000,000
Wagering gain 150,000
Wagering losses 100,000
Royalties from books published 150,000
Interest Income from banks 30,000

22. Compute Tax due, assuming ATTY. opted to use itemized deduction in computing his income
tax.
a. P810,000
b. P698,000
c. P666,000
d. P826,000

Gross Income 2,950,000


Allowable Deductions (400,000)
Taxable income 2,550,000
TAX DUE 666,000
23. Compute OSD. Optional Standard Deductions
a. P2,240,000
b. P2,300,000
c. P1,460,000
d. P1,440,000

Gross Receipts 3,600,000


OSD 40% 1,440,000

24. How much is the final withholding income tax of ATTY., assuming 2/3 of interest income
from banks were realized from time deposits with maturity of 5 years?
a. P47,000
b. P17,000
c. P32,000
d. P137,000

Capital Gains Tax 6% 120,000


Final Tax on Royalty 10% 15,000
Final Tax on Interest 2,000
TOTAL FINAL TAX 137,000

25. 1st Statement: The term “goods” for value added tax purposes shall mean all tangible and
intangible objects which are capable of pecuniary estimation and shall include, but not limited
to radio, television, satellite transmission and cable television time.
2nd Statement: The term “export sales” for VAT purposes shall include, but not be limited to
sale of gold to the Bangko Sentral ng Pilipinas.
a. True, False c. False, True
b. True, True d. False, False

26. 1st Statement: Any VAT registered person whose sales are zero-rated or effective zero-rated
may, within two (2) years after the close of the taxable year when the sales were made may
apply for tax credit certificate or refund of the creditable input tax due or paid attributable to
such sales.
2nd Statement: A VAT-registered person may apply for the issuance of tax credit certificate or
refund of input taxes paid on capital goods imported or locally purchased to the extent that
such input taxes have not been applied against output taxes. The application may be made
only within two years after the close of the taxable quarter when the importation was made.
a. True; False c. False; True
b. True; True d. False; False
27. 1st Statement: Excise taxes imposed and based on weight or volume capacity or any other
physical unit of measurement shall be referred to as ad valorem tax.
2nd Statement: Excise taxes imposed and based on selling price or other specified value of
the good shall be referred to as specific tax.
a. True; False c. False; True
b. True; True d. False; False

28. A merchant of stock or securities, whether an individual, partnership or corporation, with an


established place of business, regularly engaged in the purchase of securities and resells them
to customers; and who, as a merchant, buys securities and resells them to customers with a
view to the gains and profits that may be derived therefrom.
a. Dealer in securities c. Underwriters
b. Stock brokers d. Stock merchants

29. Mr. Robin Tulfo is a manufacturer of fermented liquors. In making sales, all taxes on the
products and transactions are passed on to the buyers. For purposes of the value-added tax,
which of these taxes mentioned here forms part of the gross selling price?
a. Excise tax c. Percentage tax
b. Value-added tax d. Both A and B

30. 1st statement: America Corporation is a merchandising concern and has one (P1,000,000.00)
million worth of inventories. Brazil Corporation, a real estate company, exchanged its real
properties for shares of stock of America Corporation resulting to acquisition of corporate
control. The one million (1,000,000.00) worth of inventories of America Corporation, which
were transferred to Brazil Corporation by virtue of corporate take-over is not subject to VAT.
2nd statement: The exchange of real property by Brazil Corporation resulting to corporate
control is subject to VAT.
a. True, true c. False, false
b. True, false d. False, true

31. ERA Corporation has the following sales during the month:
Sale to private entities 224,000.00
Sale to export-oriented enterprise 100,000.00
Sale of exempt goods 100,000.00

The following input taxes were passed on by its VAT suppliers during the month:
Input tax on taxable goods 5,000.00
Input tax on zero-rated sales 3,000.00
Input tax on sale of exempt goods 2,000.00
Input tax on depreciable capital good not 20,000.00
attributable to any specific activity
The VAT payable for the month:
a. 1,000 c. 9,000
b. 7,200 d. 16,000

Output VAT 24,000


Input VAT (5,000)
Input VAT from zero-rated (3,000)
Allocated Input VAT from depreciable goods (15,0000
(300,000/400,000)* 20,000
VAT PAYABLE 1,000

32. Mr. Jose, Filipino, married died leaving the following estate:
Car acquired before marriage by Mr. Jose - P 300,000
Car acquired before marriage by Mrs. Jose – P 450,000
House and lot acquired during marriage P 1,500,000
Jewelries of Mrs. Jose, acquired
before marriage 100,000
Personal properties inherited by Mr. Jose
during marriage 250,000
Benefits from SSS 50,000
Retirement benefits from a private firm 150,000
Proceeds of group insurance taken by his employer 75,000
Land inherited by the wife
during marriage 1,000,000
Income earned from the land inherited
By wife above (25% of which was
Earned after death) 200,000

The gross estate under Conjugal Partnership of Gains is:


a. P 2,600,000 c. P 1,950,000
b. P 3,600,000 d. P 2,200,000

Exclusive Common Total Gross Estate


Car 300,000 House 1,500,000
Personal Properties 250,000 Income 150,000
550,000 1,650,000 2,200,000
33. Under the same problem, the gross estate under Absolute Community of property is:
a. P 2,600,000 c. P 1,950,000
b. P 3,600,000 d. P 2,500,000

Exclusive Common Total Gross Estate


Cars 750,000
House 1,500,000
Personal Properties 250,000 Jewelries 100,000
250,000 2,350,000 2,600,000

34. A decedent died single, leaving a family home which consists of a piece of land that he
inherited 3 – ½ years ago (with a value at the time of P600,000) with a fair value of P800,000
at the time of his death, and a house thereon which he built at a cost of P650,000, and a fair
market value at the time of his death of P450,000. Other properties in his gross estate have
a fair market value of P550,000. Unpaid obligations at the time of his death amounted to
P300,000.
The vanishing deduction is:
a. 200,000
b. 500,000
c. 40,000
d. 225,000

Initial Basis 600,000


New Initial Basis 600,000
Basis (600,000- 600,000/1,800,000*300,000) 500,000
Vanishing Deduction 40% 200,000

35. The Congregation of the Mary Immaculate donated a land and a dormitory building located
along España Street in favor of the Sisters of the Holy Cross, a group of nuns operating a
free clinic and high school teaching basic spiritual values. Is the donation subject to donor’s
tax?
a. Yes, the donation is subject to donor’s tax because the donee is a stranger.
b. Yes, the donation is subject to donor’s tax because it failed to mention the purpose
of the donation.
c. No, the donation is not subject to donor’s tax because the donee is a non-stock, non-
profit organization engaged principally for religious and educational vocation.
d. No, the donation is exempt from donor’s tax being charitable in nature.
Beginning 1 January 2018:

Taxable Income (PHP)


Tax on column 1 (PHP) Tax on excess (%)
Over Not Over

0 250,000 - 0

250,000 400,000 - 20

400,000 800,000 30,000 25

800,000 2,000,000 130,000 30

2,000,000 8,000,000 490,000 32

8,000,000 2,410,000 35

36. The following are excluded from the gross estate of a decedent except:
a. Transfers before the decedent’s death, not in contemplation of death.
b. Bona fide sale for an adequate and full consideration in money or money’s worth before the
decedent’s death.
c. Exclusive properties or capital of the surviving spouse.
d. Proceeds of life insurance where the designation of the third party beneficiary is revocable.

37. One of the assets left by J. Cruz when he died is 100,000 shares of stock of Walt Stick
Company with a par value of ₱4.00 per share. The company has an authorized capital stock
of 1,200,000 shares. Unissued shares total 400,000 shares. Retained earnings amount to
₱600,000. The shares of stock of the company are not traded in the stock exchange. How
much will be included in the gross estate of J. Cruz?
a. ₱475,000
b. ₱316,667
c. ₱400,000
d. None of the above.
38. Mr. A died leaving a gross estate of ₱870,000. Among the assets are: (1) a receivable from B,
businessman, in the amount of ₱40,000; and (2) another receivable from C, a cousin, in the
amount of ₱30,000.
B was declared an insolvent by a court. The court determined that B’s assets that were
available for payment to his creditors totalled ₱60,000, while his total liabilities totalled
₱100,000.
On the other hand, C, without properties, was able to pay only ₱10,000.
How much can the estate deduct as claims against insolvent persons for purposes of
computing the net estate?
a. ₱70,000
b. ₱36,000
c. ₱58,000
d. None of the above.

39-40 J. Cruz, Filipino, single, died on December 24, 2018 leaving the following properties and expenses:

Apartment and lot ₱ 500,000


Toy collection 300,000
Shares of San Miguel Corp. 300,000
Land 100,000
Jeep 20,000

The last 3 properties, namely the SMC shares, the land, and the jeep were acquired by the
decedent by gratuitous title with the following fair market values:
Date FMV on FMV in estate
Acquired date acquired of J. Cruz

San Miguel shares (donated) July 1, 2012 ₱ 400,000 ₱ 300,000


Land (inherited) October 1, 2016 50,000 100,000
Jeep (inherited) October 1, 2016 100,000 20,000
Expenses:
Funeral expenses ₱ 50,000
Mortgage on apartment and lot 300,000
Loss of jeep 20,000
Other information:
a. The land which J. Cruz inherited had a mortgage of ₱30,000 which he paid before he
died.
b. The amount of ₱70,000 of the mortgage on the apartment and lot remained unpaid upon
his death.
c. The jeep was carnapped on July 5, 2019.
39. Compute the vanishing deduction that may be availed of in computing the estate tax.
a. ₱22,230
b. ₱11,115
c. ₱18,525
d. None of the above.

40. Compute the vanishing deduction assuming J. Cruz paid only the amount of ₱20,000 of the
mortgage on the inherited land.
a. ₱27,541
b. ₱16,525
c. ₱18,361
d. None of the above.

41. Which of the following does not escape the estate tax?
a. Devise of real property in favor of St. Francis Church
b. Merger of usufruct in the owner of the naked title to the property
c. Transmission via a special power of appointment of personal property to the City of Manila.
d. Legacy transmitted by the fiduciary heir to the second heir in a fideicommissary substitution.

42-43. J. Cruz, Filipino, married, died testate on January 10, 2018. Records show the following
assets and deductions on the date of death:

Condominium unit in Makati (family home) ₱ 40,000,000


Apartment in HongKong 300,000
Domestic shares 250,000
Note receivable from insolvent debtor residing in Manila 60,000
Note receivable from insolvent debtor residing in HongKong 40,000

Deductions claimed:
Cost of cemetery lot 35,000
Expenses of internment 25,000
Note payable, contracted 2017, not notarized 30,000
Note payable, contracted 2016, notarized, paid on Jan. 22, 2018 100,000
Note payable, contracted 2016, notarized, paid on Jan. 4, 2018 50,000
Mortgage on condominium unit in Makati 100,000
Mortgage on apartment in HongKong 50,000
Legacy to J. Cruz's brother 40,000
Legacy to Municipality of Orani, Bataan for schoolhouse 100,000
Legacy to social welfare organization 30,000
42. Determine the gross estate of J. Cruz.
a. ₱40,400,000
b. ₱40,250,000
c. ₱40,350,000
d. None of the above.

43. Determine the net estate of J. Cruz subject to the estate tax.
a. ₱40,050,000
b. ₱ 5,100,000
c. ₱ 5,145,000
d. None of the above.

44-46. J. Adams Mansanas, single, died on December 6, 2018, leaving the following assets,
expenses, charges, and obligations:
Assets:
Domestic shares ₱ 5,000,000
Foreign shares, U.S. Company has 40% business in the Phils. 700,000
Investment in a domestic partnership 1,000,000
Cash in bank, Manila 1,200,000
Account receivable from a debtor residing in the U.S. (The
account is worthless. Debtor is fully insolvent.) 100,000
Car, model 2017 700,000

Expenses, charges, and obligations:


Funeral expenses 150,000
Judicial expenses incurred in settlement of the estate 100,000
Unpaid income tax for 2016 160,000
Loss through robbery 2 months after death 200,000
Loss in business 80,000
Legacy in favor of Philippine National Red Cross ("PNRC") 50,000
Legacy to City of Makati for children's playground. 100,000

44. Compute the taxable net estate and the estate tax due if decedent was a resident of the
Philippines at the time of his death.
a. ₱3,090,000; ₱185,400
b. ₱3,790,000; ₱227,400
c. ₱2,840,000; ₱170,400
d. None of the above.
45. Compute the taxable net estate and the estate tax due if decedent was a non-resident alien,
and there was no reciprocity.
a. ₱7,723,103: ₱463,386
b. ₱3,090,000; ₱185,400
c. ₱6,923,103; ₱415,386
d. None of the above.

46. Compute the taxable net estate and the estate tax due if decedent was a non-resident alien, and
there was reciprocity.
a. ₱21,034; ₱1,262
b. ₱521,034; ₱31,262
c. (₱310,000); ₱0
d. None of the above.

47. In which of the following cases will filing an estate tax return be unnecessary?
a. When the gross estate does not include any registrable property and is valued at ₱640,000. Total
deductions amount to ₱530,000.
b. When the gross estate consists of the following properties: Cash, ₱40,000; Second hand
jeep parked in Manila with FMV of ₱90,000; and jewelry worth ₱130,000. Standard
deduction of ₱5,000,000 was claimed.
c. When the gross estate, which does not include any registrable property, is valued at
₱450,000. Standard deduction is claimed.
d. None of the above.

48. Where property is transferred for less than an adequate and full consideration in money or
money’s worth, then the amount by which the fair market value of the property exceeded the
value of the consideration shall be deemed a gift, and shall be included in computing the
amount of gifts made during the calendar year. The legislative intent of the deemed gift
provision under Section 100 of the Tax Code is to discourage parties in a transaction from
controlling their selling price in order to reduce their income taxes to be paid to the
government.
However, the deemed gift provision does not apply to the following:
a. Sale by individual taxpayers of real property located in the Philippines and classified as capital
asset.
b. Sale thru the stock exchange of listed domestic shares.
c. Property dividends.
d. All of the above.
49. In 2018, Alberto sold a printing press to Bimbo, his brother, the particulars of which are as
follows:
Market value of the printing press ₱ 230,000
Selling price 170,000
Cost in 20015 160,000
Book value in 2018 140,000
Which statement/s is/are true?
a. Seller Alberto is deemed a donor and buyer Bimbo a donee by operation of law.
b. The printing press being an ordinary or business asset, the entire gain on the sale is taxable
income to Alberto and must be included in his income tax return.
c. All of the above.
d. None of the above.

50. Sarah Joy Bumanglag believed in the ability of Chris Llamado, a candidate in the May 2022
elections for Senator. Sarah donated the amount of ₱500,000 to the campaign of Llamado.
a. The donation is subject to donor’s tax, but the same can be used as a deduction in Sarah Joy’s ITR.

b. The donation is not subject to donor’s tax, and the same cannot be used as a
deduction in Sarah Joy’s ITR.
c. The donation is not subject to donor’s tax, and the same can be used as a deduction in
Sarah Joy’s ITR.
d. Sarah Joy should be reprimanded for supporting an obviously incompetent candidate.

51-54. Donor gave the following donations in 2021:


April 12 To M, legitimate child, a lot with an
unpaid mortgage of ₱30,000 assumed
by M, and with a fair market value of ₱ 600,000

June 6 To B, brother, on account of marriage


on October 8, 2021 20,000

July 30 To Y, brother-in-law 40,000

November 24 To X, legitimate child, as birthday gift 10,000

To the Philippine National Red Cross ("PNRC") 50,000


51. Compute the donor's tax and the donor’s tax payable on April 12, 2021.
a. ₱19,200; ₱19,200
b. ₱34,200; ₱19,200
c. ₱19,200; ₱0
d. None of the above.

52. Compute the donor's tax and the donor’s tax payable on June 6, 2021.
a. ₱20,400; ₱1,200
b. ₱35,400; ₱1,200
c. ₱20,400; ₱20,400
d. None of the above.

53. Compute the donor's tax and the donor’s tax payable on July 30, 2021.
a. ₱37,800; ₱20,400
b. ₱22,800; ₱2,400
c. ₱2,400; ₱590,000
d. None of the above.

54. Compute the donor's tax and the donor’s tax payable on November 24, 2021.
a. ₱38,400; ₱600
b. ₱23,400; ₱600
c. ₱22,800; ₱600
d. None of the above.

55-56. J. Cruz, resident citizen, gave the following donations to his son:
August 1, 2021:
Land, Philippines ₱ 1,500,000
Land, U.S.A. 4,500,000
Gift tax paid in U.S.A. 450,000

November 1, 2021:
Land, Philippines 2,000,000

55. Compute the donor’s tax payable on August 1, 2021.


a. ₱345,000
b. ₱ 86,250
c. ₱258,750
d. None of the above.
56. Compute the donor’s tax payable on November 1, 2021.
a. ₱465,000
b. ₱117,188
c. ₱115,234
d. None of the above.

57. Statement 1: A separate return shall be filed by each donor for each gift or donation made on
different dates during the year. Any previous gifts made in the same calendar year shall be
reflected in each return. The tax due shall be computed based on the total net gifts made
during the calendar year.
Statement 2: Only one return shall be filed for several gifts or donations by a donor made on
the same date to different donees.
a. Both statements are true.
b. Statement 1 is true. Statement 2 is false.
c. Statement 1 is false. Statement 2 is true.
d. Both statements are false.

58. Statement 1: If the gift or donation involves conjugal/community property, each spouse shall
file a separate return corresponding to his/her respective share in the conjugal/community
property.
Statement 2: If the gift or donation involves co-owned property, each co-owner shall file a
separate return corresponding to his/her respective share in the co-owned property.
a. All statements are true.
b. Statement 1 is true. Statement 2 is false.
c. Statement 1 is false. Statement 2 is true.
d. All statements are false.

59. E-Regalo Corporation, a domestic corporation, is a service provider that operates a platform
for paperless gift cards, also known as electronic gift cards. It also runs digital card
storefronts for various merchants in the Philippines.
In the operation of the platform, E-Regalo earns revenues by charging service fees to the
merchants, and not from the issuance of electronic gift cards to individual clients. Whatever
amounts it receives from the buyers of electronic gift cards are held in trust for
reimbursement to its accredited merchants.

What amounts received by E-Regalo are subject to VAT?


a. Only the service fees paid by the merchants are subject to VAT.
b. Only the amounts representing the value of electronic gift cards are subject to VAT.
c. Both are subject to VAT.
d. Both are VAT-exempt.
60. Ready Wear Go Corporation is a VAT-registered domestic corporation engaged in the
manufacturing and selling of t-shirts in Manila. It owns cars which are used by its
executives for business travel.
In 2021, it sold two of its fully-depreciated cars: one to its manager, and the other to its
CFO.
a. Only the sale of t-shirts are subject to output VAT.
b. Both the sale of t-shirts and sale of the 2 cars are subject to output VAT.
c. Both the sale of t-shirs and the sale of the 2 cars are VAT-exempt.
d. None of the above.

61. The following are the characteristics of VAT:

a. Indirect tax
b. Proportional tax
c. Consumption or end-user tax
d. All of the above.

62. Statement 1: A person who imports VATable goods, whether or not in the course of trade or
business, is subject to VAT.
Statement 2: If the importer is a non-VAT person, and the importation of VATable goods is
not in the course of trade or business, such person is not required to register for VAT.
a. Both statements are true.
b. Statement 1 is true. Statement 2 is false.
c. Statement 1 is false. Statement 2 is true.
d. Both statements are false.

63. J. Parada operates 9 passenger jeeps, a limousine service, a car detailing shop, and a store
selling car and jeep parts and accessories. His sales and receipts during the year are
shown below:
Receipts, passenger jeeps ₱ 2,500,000
Receipts, limousine service 1,560,000
Receipts, car detailing shop 1,000,000
Sales, store 400,000
Total ₱ 5,460,000
a. Mr. Parada is required to VAT-register.
b. Mr. Parada is not required to VAT-register.
c. Mr. Parada may optionally register his VAT-exempt businesses under the VAT system.
d. None of the above.
64. Mr. Parada, in the preceding number, is also engaged in the exportation of toy cars with total
export sales during the year of ₱1,700,000.
a. Mr. Parada is required to VAT-register.
b. Mr. Parada is not required to VAT-register.
c. Mr. Parada may optionally register his VAT-exempt businesses under the VAT system.
d. None of the above.

65. Sweet Foot Massage Parlor is not VAT-registered and its annual gross receipts do not
exceed ₱2,500,000. Lolo Chris, a senior PWD, went to Sweet Foot Massage to augment his
physical therapy. After the massage, he presented his PWD ID card to get the 20% discount.
When he was shown the bill, he was surprised to see that while the establishment gave him a
20% discount, it was charging him the 3% other percentage tax (“OPT”).
a. Lolo Chris should present his Senior Citizen ID Card to be exempted from the 3% OPT.
b. Lolo Chris may refuse to pay as he is also entitled to exemption from payment of the
OPT.
c. Lolo Chris is liable to pay the OPT as the exemption granted by the law from
payment of indirect business taxes only covers VAT.
d. None of the above.
66. Statement 1: The importation of a cargo vessel destined for domestic or international
transport operations shall be exempt from VAT provided the importation complies with the
requirements on restriction on vessel importation and the mandatory vessel retirement
program of the Maritime Industry Authority (MARINA).
Statement 2: The sale or importation of fresh mushrooms is exempt from VAT.
a. Only Statement 1 is true.
b. Only Statement 2 is true.
c. Both statements are true.
d. Both statements are false.
67. Statement 1: The sale of services, including lease or property, to persons
engaged in international shipping or international air transport operations is 0-rated, provided
the seller is VAT-registered and such services are exclusively for international shipping or
air transport operations.
Statement 2: Services provided by a motel-restaurant to flight attendants of an
international airline shall be 0-rated.
a. Only Statement 1 is true.
b. Only Statement 2 is true.
c. Both statements are true.
d. Both statements are false.
68-70. MEGABIRD, Inc. is a VAT-registered breeder which sells white leg horn hens and
fighting cocks from its lone mega-farm in Alfonso, Cavite. White leg horn is a type of
chicken bred for meat production.
During the first quarter of 2022, total sales of fighting cocks amounted to ₱336,000, gross
of VAT, and total sales of white leg horn hens amounted to ₱200,000. MEGABIRD
enjoys a gross margin of 25% on the sale of all its products.
During the same quarter, the farm underwent repairs amounting to ₱67,200, gross of
VAT. Purchase of supplies directly attributable to the sale of fighting cocks amounted to
₱15,680, gross of VAT.
During the same quarter, MEGABIRD entered into the following transactions affecting
the inventory accounts:

Fighting cock inventory White leg horn inventory


Paid to veterinarian ₱ 382,000 ₱ 45,500
Paid to shareholders as 480,000 45,000
property dividends
Paid to creditors 525,000 56,250
Donation to Alfonso city 128,000 7,000
during fiesta
Total ₱ 1,515,000 ₱153,750

68. Compute the output VAT of MEGABIRD for the 1st quarter of 2022.
a. ₱217,800
b. ₱263,250
c. ₱ 36,000
d. None of the above.

69. Compute the available input tax credits of MEGABIRD for the 1st quarter of 2022.
a. ₱8,880
b. ₱7,788
c. ₱5,898
d. None of the above.

70. Compute the VAT payable of MEGABIRD for the 1st quarter of 2022.
a. ₱208,920
b. ₱255,462
c. ₱ 30,102
d. None of the above.

THE END.

You might also like